Patho final

Ace your homework & exams now with Quizwiz!

Following a fracture, the nurse is educating the client and his family about bone healing. The nurse begins by stating: A) "In order to initiate the cellular events essential to bone healing, within a day or so, your body will develop a blood clot at the fracture site." B) "The first thing that will happen is your body will form a soft tissue callus around the fracture site." C) "Your body will first absorb any excess bone fragments that occurred as a result of the break." D) "Just like on your hands after hard work, your body will begin the healing process by forming a callus at the fracture site."

A) "In order to initiate the cellular events essential to bone healing, within a day or so, your body will develop a blood clot at the fracture site."

A child in gymnastics class has fallen off the balance beam and hurt her ankle. X-rays are negative for fracture, so the health care provider has diagnosed a severe sprain. Which of the following treatment measures should be taught to the family and child? Select all that apply. A) Immobilization for several weeks B) Elevate the ankle on pillows C) Apply ice packs to the ankle D) Perform active and passive range-of-motion exercises daily

A) Immobilization for several weeks B) Elevate the ankle on pillows C) Apply ice packs to the ankle

The nurse is caring for of a 6-month-old infant diagnosed with iron deficiency anemia. The nurse determines further teaching is necessary when the mother states: A) "I must stop breast-feeding my baby because of the anemia." B) "I need to feed my baby cereals with fortified iron." C) "I know this anemic condition can be resolved with good nutrition." D) "I need to decrease my baby's intake of whole milk to provide the best nutrition."

A. "I must stop breast-feeding my baby because of the anemia."

A client is being taught how to use a TENS unit. The nurse determines that teaching was effective when the client states: A) :I should take my medication and apply the TENS unit to the painful area as soon as I feel the pain." B) "The TENS unit is the easiest and most cost-effective way to manage my pain at home." C) "I will need to return to the doctor's office to have this TENS unit surgically implanted under my skin for the best pain relief." D) "Anytime that the setting on my TENS unit needs to be changed, I will have to return it to the company and wait for them to deliver a new one."

A. "I should take my medication and apply the TENS unit to the painful area as soon as I feel the pain."

A client presents to the orthopedic clinic for evaluation since the primary care provider thinks the client may have rheumatoid arthritis (RA). Which statement by the client correlates with the diagnosis of RA? Select all that apply. A) "I'm having a hard time opening doors since it hurts so bad." B) "Look, I didn't button all my shirt buttons...it just hurts too much and look at the swelling in my hands." C) "Look how my hand is deformed. My doctor calls it 'hyperextension.'" D) "Just look at my face. It looks like I have varicose veins on my cheeks." E) "Every time I get something out of the freezer, my hands turn reddish purple in color."

A. "I'm having a hard time opening doors since it hurts so bad." B. "Look, I didn't button all my shirt buttons...it just hurts too much and look at the swelling in my hands." C. "Look how my hand is deformed. My doctor calls it 'hyperextension.'"

Which of the following statements by the husband of a client with Alzheimer disease demonstrates an accurate understanding of his wife's medication regimen? A) "I'm really hoping these medications will slow down her mental losses." B) "We're both holding out hope that this medication will cure her Alzheimer's." C) "I know that this won't cure her, but we learned that it might prevent a bodily decline while she declines mentally." D) "I learned that if we are vigilant about her medication schedule, she may not experience the physical effects of her disease."

A. "I'm really hoping these medications will slow down her mental losses."

A client presented to the emergency department after getting "hit in the head with a baseball" while watching his grandson play. An x-ray of the head reveals poor quality of bone. The ED physician suspects the client has Paget disease. Which of the following signs/symptoms helps confirm this diagnosis? Select all that apply. A) "I've had a lot of headaches lately." B) "Every now and then, I get a ringing in my ears." C) "Do you see my knuckles...they have big growths on them." D) "My thumb joint has been cracking every time I rotate it." E) "I have gotten dizzy and had to sit down while shopping."

A. "I've had a lot of headaches lately." B. "Every now and then, I get a ringing in my ears." E. "I have gotten dizzy and had to sit down while shopping."

A client with a recent diagnosis of renal failure who will require hemodialysis is being educated in the dietary management of the disease. Which of the client's following statements shows an accurate understanding of this component of treatment? A) "I've made a list of high-phosphate foods, so that I can try to avoid them." B) "I'm making a point of trying to eat lots of bananas and other food rich in potassium." C) "I'm going to try to maintain a high-fiber, low-carbohydrate diet." D) "I don't think I've been drinking enough, so I want to include 8 to 10 glasses of water each day."

A. "I've made a list of high-phosphate foods, so that I can try to avoid them."

A client who had a pulmonary embolism is receiving IV heparin and has just begun taking his first dose of warfarin (Coumadin). The client asks the nurse, "How long will this pill take in order to prevent me from developing more clots. I would like to go home soon." The nurse responds: A) "It usually takes 2 to 3 days for warfarin to become therapeutic, meaning your blood will be thin enough to prevent further clot formation." B) "I don't really know. Right now you are on two blood thinners, so we won't be able to tell if it's working until the doctor discontinues your heparin drip." C) "That's a good question. If warfarin has a long half-life, it could take many days. I will have to ask the pharmacist this question." D) "First, we have to have your health care provider switch you from IV heparin to a low molecular weight heparin, which you need to inject in your belly."

A. "It usually takes 2 to 3 days for warfarin to become therapeutic, meaning your blood will be thin enough to prevent further clot formation."

A 20-year-old male athlete presents with a soft tissue contusion on the right ankle. The most appropriate information for the nurse to provide would be: A) "Keep the ankle elevated and apply ice for 15 minutes at a time for the first 24 hours." B) "Surgery will be the expected treatment, and consent will need to be signed immediately." C) "Pain medications will be the only treatment needed." D) "Apply heat compresses for 15 minutes to open up the venues of the soft tissue."

A. "Keep the ankle elevated and apply ice for 15 minutes at a time for the first 24 hours."

The nurse has just finished teaching a client newly diagnosed with rheumatoid arthritis about the disease. The nurse determines that teaching is effective when the client states: A) "Rheumatoid arthritis includes joint involvement that is usually symmetric and polyarticular." B) "The disease will not fluctuate in occurrence." C) "Rheumatoid arthritis is a chronic autoimmune system disease that affects a single joint." D) "Only upper extremity diathrodial joint can be involved."

A. "Rheumatoid arthritis includes joint involvement that is usually symmetric and polyarticular."

A client presents to a health clinic complaining of several vague symptoms. As the history/physical continues, the health care provider clearly thinks the client may have myasthenia gravis. Which statements by the client would correlate with this diagnosis? Select all that apply. A) "Sometimes I have double vision." B) "I have more energy in the morning but get worse as the day goes by." C) "Sometimes I have numbness/tingling on my face." D) "I had what felt like an electric shock race down the back of my leg when I bend my neck." E) "I feel like I don't have enough energy to chew my food sometimes."

A. "Sometimes I have double vision." B. "I have more energy in the morning but get worse as the day goes by." E. "I feel like I don't have enough energy to chew my food sometimes."

A beta-adrenergic blocker has been prescribed for a client diagnosed with migraines. The most important information for the nurse to teach the client would be: A) "Take the medication daily as you have been directed." B) "Stop this medication immediately if you have dizziness after taking it." C) "Take the medication only when feeling a migraine start." D) "This medication will prevent you from having any more migraines."

A. "Take this medication daily as you have been directed."

A client refuses to take the 81 mg of aspirin ordered by the physician, stating, "I do not have any pain." The best response by the nurse would be: A) "The 81 mg of aspirin daily will help protect you from a stroke or a heart attack." B) "This dose of aspirin will break the blood clot that you have in your leg." C) "Low-dose aspirin will help prevent you from having increased bleeding after surgery." D) "The doctor wants you to take the medication to prevent you from experiencing pain."

A. "The 81 mg of aspirin daily will help protect you from a stroke or a heart attack."

A client's primary care provider has ordered an oral glucose tolerance test (OGTT) as a screening measure for diabetes. Which of the following instructions should the client be given? A) "The lab tech will give you a sugar solution and then measure your blood sugar levels at specified intervals." B) "You'll have to refrain from eating after midnight and then go to the lab to have your blood taken first thing in the morning." C) "They'll take a blood sample and see how much sugar is attached to your red blood cells." D) "You can go to the lab at any time; just tell the technician when you last ate before they draw a blood sample."

A. "The lab tech will give you a sugar solution and them measure your blood sugar levels at specified intervals."

A nurse is assessing the pain level fo a client who has just had surgery. The client tells the nurse he does not think he should take any medication until the pain becomes extreme. The best response would be: A) "The use of pain medication before the pain becomes extreme will help prevent hyper excitability of the CNS." B) "If you wait longer, you will receive stronger medication." C) "Asking for pain medication with the early onset of pain will cause addiction." D) "If you wait for a longer period of time, the pain will decrease in intensity."

A. "The use of pain medication before the pain becomes extreme will help prevent hyper excitability of the CNS."

A couple has just learned that their newborn infant has been diagnosed with osteogenesis imperfecta, and they have responded by seeking out as much information as possible about their child's diagnosis. What should the clinician teach the couple about their child's health problem? A) "This is something that your child may have inherited from one or both of you." B) "This might have been caused by something you were exposed to during the early part of your pregnancy." C) "You'll have to be vigilant of your child's safety for the next few years, but the disease often resolves spontaneously." D) "With aggressive treatment, most children with osteogenesis imperfecta are cured within several months."

A. "This is something that your child may have inherited from one or both of you."

A bladder cancer client asks the nurse, "What did the doctor mean by intravesicular chemotherapy? Am I going to lose all my hair and have to do for treatments over months and months?" The best response would be: A) "This is when they put the chemotherapy directly into the bladder to kill any cancer cells." B) "They will take you to radiology and inject some chemotherapy through your abdomen into your bladder." C) "The doctor will place a scope up your urethra, into the bladder, and burn the lining of the bladder with a laser and then inject some tuberculosis bacillus into the lining." D) "This is when they use a CyberKnife to cut off any lesions and then inject chemotherapy into the remaining portion of the bladder."

A. "This is when they put the chemotherapy directly into the bladder to kill any cancer cells."

A nurse is monitoring the intracranial pressure (ICP) of a client. The nurse would consider the client to have a normal reading when the results identify: A) 0-15 mm Hg B) 30-45 mm Hg C) 15-30 mm Hg D) 45-60 mm Hg

A. 0-15 mm Hg

When educating a client with possible glucocorticoid dysfunction, the nurse will explain that the CRH controls the release of ACTH. The best time to perform the blood test to measure peak ACTH levels would be: A) 06:00 to 08:00 AM B) 10:00 to 12:00 AM C) 04:00 to 6:00 PM D) 09:00 to 11:00 PM

A. 06:00 to 08:00 AM

The nurse is planning care for a client with chronic kidney disease (CKD). The nurse determines that an allowable fluid intake would be: A) 500 to 800 mL/day B) 400 to 600 mL/day C) 1050 to 1200 mL/day D) 850 to 1000 mL/day

A. 500 to 800 mL/day

The nurse is reviewing assessment data on four clients. Select the client at highest risk for developing type 2 diabetes. A) A 45 year-old obese female with a sedentary lifestyle B) A 10-year-old male whose grandmother has type 2 diabetes C) A 60-year-old female with a history of gestational diabetes D) A 40-year old male who has an active lifestyle

A. A 45-year-old obese female with a sedentary lifestyle

The nurse is reviewing the health histories of four clients. Select the client most at risk for developing secondary osteoporosis. A) A 60-year-old female taking prednisone for asthma B) A 22-year-old female taking oral contraceptives C) An 80-year-old male who resides in an assisted living facility D) A 42-year-old male with a healing leg fracture

A. A 60-year-old female taking prednisone for asthma

Which of the following clients are at high risk for developing anemia? Select all that apply. A) A HIV-positive client who is experiencing frequent infection and elevated CD4 + counts B) A breast cancer client undergoing chemotherapy and radiation therapy C) A client who had a colectomy 3 weeks ago to remove adhesions and fecal impaction D) A COPD client with acute exacerbation requiring O 2 therapy via C-PAP E) A middle-aged renal failure client who has hemodialysis three times/week

A. A HIV-positive client who is experiencing frequent infection and elevated CD4+ counts B. A breast cancer client undergoing chemotherapy and radiation therapy E. A middle-aged renal failure client who has hemodialysis three times/week

Which of the following clients are at risk for developing hypothyroidism? Select all that apply. A) A client who is prescribed amiodarone for frequent dysrhythmias B) A client who has bulging eyeballs being treated with β-adrenergic blockers C) A client who has precancerous thyroid lesions who underwent ablation with radiation D) A female experiencing an autoimmune disorder called thyroiditis E) A bipolar client prescribed lithium carbonate

A. A client who is prescribed amiodarone for frequent dysrhythmias C. A client who has precancerous thyroid lesions who underwent ablation with radiation D. A female experiencing an autoimmune disorder called thyroiditis E. A bipolar client prescribed lithium carbonate

Which of the following clients are at risk for developing blood clots and should be assessed frequently? Select all that apply. A) A college-aged female taking oral contraceptives for irregular menstrual periods B) A middle-aged male who smokes three packs of cigarettes/day C) A young adult who has a history of asthma and utilizes steroid inhalers daily D) A marathon runner who exercises more than 4 hours/day E) A diabetic client who is also greater than 100 pounds over the ideal body weight

A. A college-aged female taking oral contraceptives for irregular menstrual periods B. A middle-aged male who smokes three packs of cigarettes/day E. A diabetic client who is also greater than 100 pounds over the ideal body weight

Of the following clients, which would be at highest risk for developing hyperkalemia? A) A male admitted for acute renal failure following a drug overdose B) A client diagnosed with an ischemic stroke with multiple sensory and motor deficits C) An elderly client experiencing severe vomiting and diarrhea as a result of influenza D) A postsurgical client whose thyroidectomy resulted in the loss of some of the parathyroid glands

A. A male admitted for acute renal failure following a drug overdose

Which of the following clients would be considered to be exhibiting manifestations of "prediabetes"? A) A middle-aged overweight adult with a fasting plasma glucose level of 122 with follow- up OGTT of 189 mg/dL. B) A school-aged child who had a blood glucose level of 115 following lunch. C) A retired female registered nurse with a fasting plasma glucose level of 92 mg/dL. D) An elderly client who got "light-headed" when he skipped his lunch. Blood glucose level was 60 mg/dL at this time.

A. A middle-aged overweight adult with a fasting plasma glucose level of 122 with follow-up OGTT of 189 mg/dL

Which of the following scenarios place the client at a high risk for developing hypoparathyroidism and require close supervision for assessing for development of muscle cramps, carpopedal spasm, convulsions, and paresthesia in the hands and feet? Select all that apply. A) A neck cancer client returning from OR after having a radical neck dissection B) A hyperthyroid client experiencing a "thyroid storm" requiring urgent thyroidectomy C) A client with seizure experiencing some anoxic deficits and memory loss D) A client with a history of human papillomavirus (HPV) in the uvula

A. A neck cancer client returning from OR after having a radical neck dissection B. A hyperthyroid client experiencing a "thyroid storm" requiring urgent thyroidectomy

A client with a diagnosis of liver cirrhosis secondary to alcohol abuse has a distended abdomen as a result of fluid accumulation in his peritoneal cavity (ascites). Which of the following pathophysiologic processes contributes to this third spacing? A) Abnormal increase in transcellular fluid volume B) Increased capillary colloid osmotic pressure C) Polydipsia D) Impaired hormonal control of fluid volume

A. Abnormal increase in transcellular fluid volume

While comparing and contrasting joints, the instructor mentions that the primary role of fibrocartilage includes: Select all that apply. A) Absorption of physical shock B) Provision of flexibility C) Facilitation of long bone growth D) Tissue hydration E) Rigidity

A. Absorption of physical shock B. Provision of flexibility

While discussing the role of parathyroid glands in bone formation, the instructor will mention which of the following statements? To maintain adequate serum calcium levels, parathyroid hormone: Select all that apply. A) Activates vitamin D to increase intestinal absorption of calcium B) Reduces serum phosphate levels C) Facilitates release of calcium from the bone D) Stimulates the kidneys to hold on to more sodium E) Reduces tubular reabsorption of phosphates

A. Activates vitamin D to increase intestinal absorption of calcium B. Reduces serum phosphate levels C. Facilitates release of calcium from the bone

While explaining the physiology behind systemic sclerosis (scleroderma), the instructor states, "One of the hallmarks of scleroderma is: A) Activation of fibroblasts, resulting in fibrosis B) The development of thin fragile skin C) Development of a collagen deficiency D) Avascular necrosis of the femoral head

A. Activation of fibroblasts, resulting in fibrosis

Hemoglobin solubility results and hemoglobin electrophoresis have resulted in a diagnosis of sickle cell anemia in an African American infant. The parents of the child should be aware that their child is at a significant risk for which of the following health problems? Select all that apply. A) Acute pain B) Stroke C) Respiratory disease D) Autoimmune diseases E) Fractures

A. Acute pain B. Stroke C. Respiratory disease

If a client with a kidney stone has the "classic" ureteral colic, the client will describe his pain as: Select all that apply. A) Acute, intermittent B) Diffuse over the entire lower back and legs C) Excruciating D) In the flank and upper outer quadrant of the abdomen

A. Acute, intermittent C. Excruciating D. In the flank and upper outer quadrant of the abdomen

The signs and symptoms of abrupt cessation of pharmacologic glucocorticoids closely resemble those of: A) Addison disease B) Cushing disease C) Cushing syndrome D) Graves disease

A. Addison disease

A client has been diagnosed with dysfunction of the anterior pituitary gland. The nurse is aware that which of the following hormones may be affected? Select all that apply. A) Adrenocorticotropic hormone (ACTH) B) Antidiuretic hormone (ADH) C) Growth hormone (GH) D) Norepinephrine E) Thyroid-stimulating hormone (TSH) F) Luteinizing hormone (LH)

A. Adrenocorticotropic hormone (ACTH) C. Growth hormone (GH) E. Thyroid-stimulating hormone (TSH) F. Luteinizing hormone (LH)

A patient is concerned that medication may damage his kidneys. Which factor(s) place the patient at most risk for developing a drug-related nephropathy? Select all that apply. A) Age/elderly B) Age/young adult C) Diuretics D) Prescription methicillin and other synthetic antibiotics E) Nonsteroidal anti-inflammatory drugs (NSAIDs) F) Procedures involving radiocontrast media

A. Age/elderly C. Diuretics D. Prescription methicillin and other synthetic antibiotics E. Nonsteroidal anti-inflammatory drugs (NSAIDs) F. Procedures

A client with a history of osteoarthritis in his hip, which he refers to as "bad hip," is also complaining of knee pain. The physiological principle behind this would include the fact that: A) All joints of an extremity are innervated by the same peripheral nerves as they travel down the limb. B) If the hip is dislocated, then the knee is not staying in the correct alignment. C) There is more than likely a tear in the synovial membrane, which is impacting the amount of plasma diffusing between the vessels and the joint. D) Synovial tissue has received an injury that is referring pain to the knee.

A. All joints of an extremity are innervated by the same peripheral nerves as they travel down the limb

A client who has developed stage 3 renal failure has been diagnosed with high phosphate levels. To avoid the development of osteodystrophy, the physician may prescribe a phosphate-binding agent that does not contain: A) Aluminum B) Calcium carbonate C) Calcium acetate D) Sevelamer hydrochloride

A. Aluminun

Which of the following residents of a long-term facility is exhibiting clinical manifestations of hypothyroidism? A) An 80-year-old woman who has uncharacteristically lost her appetite and often complains of feeling cold B) A 90-year-old woman with a history of atrial fibrillation whose arrhythmia has recently become more severe C) An 88-year-old man with a history of Alzheimer disease who has become increasingly agitated and is wandering around the facility more frequently D) A 91-year-old man with a chronic venous ulcer and a sacral ulcer who has developed sepsis

A. An 80-year-old woman who has uncharacteristically lost her appetite and often complains of feeling cold

Which of the following individuals displays the precursors to acromegaly? A) An adult with an excess of growth hormone due to an adenoma B) A girl who has been diagnosed with precocious puberty C) An adult who has a diagnosis of Cushing syndrome D) A client who has recently developed primary adrenal carcinoma

A. An adult with an excess of growth hormone due to an adenoma

Which of the following clients are at risk for developing osteomalacia? Select all that apply. A) An elderly female who "can't stand to drink milk" and refuses calcium supplements since she has a history of kidney stones B) A person who lives in a colder region of the northwest who doesn't get out much during the winter months for fear of falling C) A middle-aged adult with acute renal insufficiency caused by decreased cardiac output prior to having a coronary bypass graft surgery D) A young 30-year-old African American diagnosed with hypertension who is noncompliant related to taking medication and prescribed diet restrictions E) A female client whose father had a genetic predisposition to primary hyperparathyroidism resulting in increased calcium resorption from the bone

A. An elderly female who "can't stand to drink milk" and refuses calcium supplements since she has a history of kidney stones B. A person who lives in a colder region of the northwest who doesn't get out much during the winter months for fear of falling E. A female client whose father had a genetic predisposition to primary hyperparathyroidism resulting in increased calcium resorption from the bone

All but which of the following are true about platelets? A) An enzyme called erythropoietin stimulates their production B) They are made from megakaryocytes C) They originate from the bone marrow D) They are stored in the spleen

A. An enzyme called erythropoietin stimulates their production

A client with a diagnosis of chronic kidney disease (CKD) may require the administration of which of the following drugs to treat coexisting conditions that carry a high mortality? A) Antihypertensive medications B) Antiarrhythmic medications C) Opioid analgesics D) Nonsteroidal anti-inflammatory drugs (NSAIDs)

A. Antihypertensive medications

The nurse is teaching a client with diabetes about medications that will increase the blood glucose level. The most important information for the nurse to provide would be: Select all that apply. A) Antipsychotics B) Loop diuretics C) Insulin D) Vitamins E) Total parenteral nutrition F) Oral contraceptives

A. Antipsychotics B. Loop diuretics E. Total parenteral nutrition F. Oral contraceptives

For which of the following health problems is stem cell transplantation likely to be of therapeutic benefit? A) Aplastic anemia B) β-Thalassemias C) Chronic disease anemias D) Secondary polycythemia

A. Aplastic anemia

A client with a spinal cord injury at T8 would likely retain normal motor and somatosensory function of her: A) Arms B) Bowels C) Bladder D) Perineal musculature

A. Arms

A health care provider suspects a female client (who has had vague complaints over the past several months) may be developing systemic lupus erythematosus (SLE). Which clinical manifestations would correlate with this diagnosis? Select all that apply. A) Arthralgia B) Tendon rupture C) Facial hair growth D) Uncontrolled hypertension related to pyelonephritis E) Chest pain that increases with each deep breath

A. Arthralgia B. Tendon rupture E. Chest pain that increases with each deep breath

Which of the following pain medications should not be administered to a client who has hemophilia A? A) Aspirin B) Acetaminophen C) Morphine D) Oxycodone

A. Aspirin

Following a stroke, injury to nerve cells within the central nervous system needs to be repaired. The health care provider knows that which of the following processes explains how this occurs? A) Astrocytes fill up the space to form a glial scar, repairing the area and replacing the CNS cells that cannot regenerate. B) The microglia are responsible for cleaning up debris after cerebral infection, or cell death. C) Ependymal cells are responsible for phagocytosis. D) Oligodendrocytes are responsible for integrative metabolism.

A. Astrocytes fill up the space to form a glial scar, repairing the area and replacing the CNS cells that cannot regenerate

A client fell off a ladder and sustained a spinal cord injury that has resulted in bladder dysfunction. During the period immediately after the spinal injury, spinal shock develops and the bladder displays what type of function? A) Atonic B) Spasmodic C) Uninhibited D) Hyperactive

A. Atonic

The most important information for a nurse to give a patient who has been diagnosed and treated for a calcium oxalate stone would be to: A) Avoid spinach and peanuts B) Increase intake of citrus juices C) Take a thiazide diuretic D) Avoid milk and dairy products

A. Avoid spinach and peanuts

The results of a 44-year-old obese man's recent diagnostic workup have culminated in a new diagnosis of type 2 diabetes. Which of the following pathophysiologic processes underlies the client's new diagnosis? A) Beta cell exhaustion due to long-standing insulin resistance B) Destruction of beta cells that is not attributable to autoimmunity C) T-lymphocyte-mediated hypersensitivity reactions D) Actions of insulin autoantibodies (IAAs) and islet cell autoantibodies (ICAs)

A. Beta cell exhaustion due to long-standing insulin resistance

In type 1 diabetes mellitus the cause is: A) Beta cells fail to secrete insulin B) Glucose cannot enter cells C) Hypersecretion of pancreatic enzymes D) Increased blood glucose

A. Beta cells fail to secrete insulin

Which of the following comorbidities represent the greatest risk for the development of foot ulcers in a diabetic client? Select all that apply. A) Bilateral distal loss of pain sensation B) Previous incidents of diabetic ketoacidosis with blood glucose levels of 400 mg/dL C) Diabetic renal problems with severely decreased GFR D) Motor neuropathy related to improperly fitted shoes E) Smoking history averaging 2 packs/day

A. Bilateral distal loss of pain sensation D. Motor neuropathy related to improperly fitted shoes E. Smoking history averaging 2 packs/day

Although urinary obstruction and urinary incontinence have almost opposite effects on urination, they can both result from: A) Bladder structure changes B) Bladder wall atrophy C) Micturition reflex spasms D) Bladder distensibility loss

A. Bladder structure changes

A nurse is informing a client diagnose with a neurogenic bladder about treatment options. Which options should be included in client teaching? Select all that apply. A) Bladder training B) Pharmacologic intervention C) Catheterization D) Weight reduction E) Surgery

A. Bladder training B. Pharmacologic intervention C. Catheterization E. Surgery

When explaining to the client diagnosed with gout how the xanthine oxidase inhibitors work to help treat gout, the health care provider would include which of the following data? Allopurinol: A) Blocks the production of uric acid by the body B) Increases elimination of uric acid by the kidneys C) Prevents flare-up during the first few months of starting medication D) Can be given intravenously to rapidly decrease serum uric acid levels

A. Blocks the production of uric acid by the body

A diabetic client presents to the clinic. He is concerned his lower legs are "feeling funny." Which of the following assessment findings lead the health care provider to suspect the client may have developed somatic neuropathy? Select all that apply. A) Both legs appear to be the same as far as numbness is involved. B) Bilateral cool ankles and feet. C) Right foot has a diminished perception of vibration; left foot is normal. D) With eyes closed, the client cannot identify where the HCP is touching his feet. E) One leg has a reddened area in the calf and has a positive Homan sign.

A. Both legs appear to be the same as far as numbness is involved B. Bilateral cool ankles and feet D. With eyes closed, the client cannot identify where the HCP is touching his feet

A teenager has been in a car accident and experienced acceleration-deceleration head injury. Initially, the client was stable but then started to develop neurological signs/symptoms. The nurse caring for this client should be assessing for which type of possible complication? A) Brain contusions and hematomas B) TIAs and cerebrovascular infarction C) Momentary unconsciousness D) Status epilepticus

A. Brain contusions and hematomas

Which area of the brain is responsible for respiration? A) Brain stem B) Midbrain C) Diencephalon D) Frontal lobe

A. Brain stem

A client is experiencing bladder hyperactivity. The nurse should be prepared to educate the client about which of the following medications that may be injected to help decrease the bladder hyperactivity? A) Capsaicin, a specific C-fiber afferent neurotoxin B) Botulinum toxin type A C) Oxybutynin, an antimuscarinic agent D) Urecholine, a cholinergic agonist

A. Capsaicin, a specific C-fiber afferent neurotoxin

The nurse is caring for a client who is being treated for HHS (hyperosmolar hyperglycemic state). The nurse is aware that the client should be assessed for: A) Cerebral edema B) Polyuria C) Glycosuria D) Dehydration

A. Cerebral edema

Select the body systems most commonly affected by chronic kidney disease in the elderly. select all that apply. A) Cerebrovascular systems B) Hematologic C) Cardiovascular D) Immune E) Skeletal F) Reproductive systems G) Gastrointestinal systems H) Integumentary systems

A. Cerebrovascular systems C. Cardiovascular E. Skeletal

A woman who is exhibiting clinical manifestations of a pituitary adenoma will likely complain of: Select all that apply. A) Cessation of menses B) Unusual milk secretion unrelated to pregnancy C) Enlargement of the abdomen D) Pelvic pain E) Infertility

A. Cessation of menses B. Unusual milk secretion unrelated to pregnancy E. Infertility

The parents of a child who is diagnosed with short stature ask what may have caused the abnormality. The best response are: Select all that apply. A) Chromosomal abnormalities B) Acute kidney disease C) Hyperthyroidism D) Protein-calorie malnutrition E) Malabsorption syndromes F) Panhyperpituitarism G) GH deficiency

A. Chromosomal abnormalities D. Protein-calorie malnutrition E. Malabsorption syndromes G. GH deficiency

An adult client presents to the emergency department with manifestations of acute cystitis. For which would the nurse assess the client? Select all that apply. A) Cloudy urine B) Back discomfort C) Urinary infrequency D) Burning on urination E) Chest pain F) Fever

A. Cloudy urine B. Back discomfort D. Burning on urination

A client has just been diagnosed with Hashimoto thyroiditis and is asking the health care provider for information about the condition. The most appropriate information to provide would be: Select all that apply. A) Common in females B) Hyperthyroidism develops at disease onset C) Autoimmune disorder D) Prone to goiter development E) Rare cause of hypothyroidism F) Occurs only in adults

A. Common in females C. Autoimmune disorder D. Prone to goiter development

Which statements identify bladder function? Select all that apply. A) Control of function involves the autonomic and somatic nervous systems B) The sympathetic nervous system promotes bladder filling C) Storage and filtration of urine D) Striated muscles provide involuntary control of urine E) The parasympathetic nervous system promotes emptying

A. Control of function involves the autonomic and somatic nervous systems B. The sympathetic nervous system promotes bladder filling E. The parasympathetic nervous system promotes emptying

A client diagnosed with urinary obstruction exhibits a weak, small urinary stream and hesitancy. The client most likely is experiencing: A) Decompensation B) Spasmodic response C) Early obstruction D) Compensation

A. Decompensation

As chronic kidney disease progresses, the second stage (renal insufficiency) is identified by: A) Decrease in GFR of 60 to 89 mL/minute/1.73 m 2 B) Decrease in GFR to 30 to 59 mL/minute/1.73 m 2 C) GFR decrease to 15 to 29 mL/minute/1.73 m 2 D) Diminished GFR to less than 15 mL/minute/1.73 m 2

A. Decrease in GFR of 60 to 89 mL/minute/1.73 m2

A very ill client has been admitted to the hospital for testing for possible septic shock. The client reports lightheadedness, dizziness, and tingling/numbness of the fingers and toes. The nurse understands that this is likely due to which physiological phenomenon? A) Decrease in cerebral blood flow B) Impaired alveolar ventilation C) Gain in bicarbonate D) Inability of the kidney to excrete the body's fixed acids

A. Decrease in cerebral blood flow

If a CKD client is developing uremic encephalopathy, the earliest manifestations may include: Select all that apply. A) Decreased alertness B) Delirium and hallucinations C) New-onset seizures D) Diminished awareness

A. Decreased alertness D. Diminished awareness

The health care provider is concerned that a client may be at risk for problems with cerebral blood flow. The most important data to assess would be: A) Decreased level of oxygen B) Decreased level of PCO2 C) Decreased level of carbon dioxide D) Decreased hydrogen ions

A. Decreased level of oxygen

A traumatic brain injury client has developed extreme cerebral edema. The nurse is monitoring the client closely for signs of brain herniation. Which clinical manifestations would correlate to upward herniation of the midbrain from the infratentorial compartment? Select all that apply. A) Deep coma B) Rhythmic movement of arms and legs C) Respiratory rate of 8 with intermittent sighs D) Intracranial bleeding from nose and ears E) Bilateral small, fixed pupils

A. Deep coma C. Respiratory rate of 8 with intermittent sighs E. Bilateral small, fixed pupils

A client has progressed to the stage of advanced syphilis (tertiary). Assessment findings of the client may include: A) Dementia B) Stomatitis C) Nervousness D) Tremors

A. Dementia

The family of a multiple sclerosis client asks, "What psychological manifestations may we expect to see in our mother?" The health care provider informs them to expect which of the following? Select all that apply. A) Depression B) Hallucinations C) Delirium D) Inattentiveness E) Forgetfulness

A. Depression D. Inattentiveness E. Forgetfulness

A client works as a data entry worker for a large company. The client goes to employee health with pain in the wrist/hand. The nurse suspects that it is carpal tunnel syndrome based on which of the following assessment findings? Select all that apply. A) Describes numbness/tingling in the thumb and first digit B) States his forearm feels funny (paresthesia) C) Loss of tendon reflexes on the affected extremity D) Precision grip weakness in the affected hand E) Pain interferes with sleeping

A. Describes numbness/tingling i the thumb and first digit D. Precision grip weakness in the affected hand E. Pain interferes with sleeping

A client sustained a fall that resulted in an injury to the right shoulder, and the emergency doctor suspects a rotator cuff injury. The client would most likely manifest: Select all that apply. A) Difficulty abducting the affected arm B) Increased active range of motion C) Muscle atrophy D) Pain E) Increased ability to rotate the arm

A. Difficulty abducting the affected arm C. Muscle atrophy D. Pain

An infant (5 days old) has lab results revealing an elevated level of unconjugated bilirubin, due to hemolysis of RBCs. Because of this, which assessment findings would correlate with this abnormal lab result? Select all that apply. A) Difficulty to arouse (lethargy) B) Cyanosis in hands and feet C) Click in the right hip area when adducted D) Jaundice E) Rigidity and tremors

A. Difficulty to arouse (lethargy) D. Jaundice

A client has developed osteomyelitis and asks the health care provider how the problem occurred. The best response would be: A) Direct contamination of an open wound B) Rheumatoid disease C) Excessive vitamin intake D) Deficiency of calcium

A. Direct contamination of an open wound

A healthy, primiparous (first-time) mother delivered a healthy infant several hours ago, but the mother has experienced postpartum hemorrhage. Which of the following disorders is most likely to underlie the client's excessive bleeding after delivery? A) Disseminated intravascular coagulation B) Hemophilia A C) Von Willebrand disease D) Thrombotic thrombocytopenic purpura (TTP)

A. Disseminated intravascular coagulation

Which of the following interventions would be considered a nonpharmacologic method of pain control? Select all that apply. A) Distraction by knitting B) Guided imagery C) Biofeedback D) OTC acetaminophen

A. Distraction by knitting B. Guided imagery C. Biofeedback

A client's recent diagnosis of Parkinson disease has prompted his care provider to promptly begin pharmacologic therapy. The drugs that are selected will likely influence the client's levels of: A) Dopamine B) Acetylcholine C) Serotonin D) Adenosine

A. Dopamine

The public health nurse is presenting a program on the prevalence of STIs, which should include: Select all that apply. A) Drug can manage infections but do not control the spread B) Partners of infected persons are easy to find and treat C) STIs may be asymptomatic D) Condoms do not prevent the spread of STIs E) STIs are only bacterial

A. Drug can manage infections but do not control the spread C. STIs may be asymptomatic

The nurse is providing dietary instruction for a client diagnosed with folic acid deficiency. The nurse should provide the client with what information to help assure the client ingests sufficient amounts of folic acid? Select all that apply. A) Eat green leafy vegetables daily. B) Cook all vegetables to increase their absorption C) Increase consumption of carbohydrates D) Alcohol consumption is a risk factor for folic acid deficiency E) Include a variety of cereals in the daily diet

A. Eat green leafy vegetables daily. D. Alcohol consumption is a risk factor for folic acid deficiency E. Include a variety of cereals in the daily diet

A patient who developed acute pyelonephritis asks the nurse what caused the infection. Which should be included in the nurse's response? Select all that apply. A) Escherichia coli is the causative agent in about 80% of cases B) Reflux is the most common cause C) Severe hypertension often is a contributing factor in the progress of the disease D) Acute pyelonephritis is caused by bacterial infection E) Outflow obstruction, catheterization, and urinary instrumentation

A. Escherichia coli is the causative agent in about 80% of cases D. Acute pyelonephritis is caused by bacterial infection E. Outflow obstruction, catheterization, and urinary instrumentation

Which of the following clinical manifestations would support the medical diagnosis of Cushing syndrome? Select all that apply. A) Excessive facial hair growth B) Muscle hypertrophy C) Blood glucose level in 200 mg/dL range D) "Buffalo hump" on back E) Blood pressure reading less than 90/70

A. Excessive facial hair growth C. Blood glucose level in 200 mg/dL range D. "Buffalo hump" on back

Which of the following is a nonsurgical method of treatment for renal calculi (kidney stones)? A) Extracorporeal shock wave lithotripsy (ESWL) B) Retrograde ureteroscopy C) Percutaneous ureterolithotomy D) Percutaneous nephrolithotomy

A. Extracorporeal shock wave lithotripsy (EWSL)

The health care provider for a client with degenerative Alzheimer-type dementia recognizes the client has moved from the initial stage to the moderate stage based on which of the following clinical manifestations? Select all that apply. A) Extreme confusion and disorientation B) Incontinence of urine and bowel C) Need for direct supervision for ADLs D) Social withdrawal from all family and friends E) Inability to problem solve simple tasks

A. Extreme confusion and disorientation C. Need for direct supervision for ADLs E. Inability to problem solve simple tasks

Select the most appropriate pain assessment tool for a health care provider to use when caring for children between the ages of 3 to 7 years old. A) Faces pain scale B) Numeric pain scale C) Behavioral distress scale D) Word graph scale

A. Faces pain scale

Hemophilia A is a hereditary blood disorder caused by inadequate activity or absence of: A) Factor VIII B) Prothrombin C) von Willebrand complex D) Intrinsic factor

A. Factor VIII

Which of the following food items are considered an important dietary source of vitamin D? Select all that apply. A) Fish B) Cottage cheese C) Irradiated milk D) Mozzarella cheese sticks E) Apples

A. Fish C. Irradiated milk

A sudden traumatic complete transection of the spinal cord results in what type of injury below the site? A) Flaccid paralysis B) Vasoconstriction C) Deep visceral pain D) 3+ tendon reflexes

A. Flaccid paralysis

A client presents with copious, frothy, malodorous green/yellow vaginal discharge. A diagnosis of Trichomoniasis is made. The client tells the nurse that she has not been sexually active and is concerned. Select the possible modes of transmission. Select all that apply. A) Fomites B) Drinking glasses C) Respiratory droplets D) Sexual contact E) Swimming pools F) Hot tubs

A. Fomites D. Sexual contact E. Swimming pools F. Hot tubs

Disorders that affect cortical bone typically result in: A) Fractures of long bones B) Impaired collagen synthesis C) Infection D) Vertebral fractures

A. Fractures of long bones

While explaining the somatosensory cortex to a group of nursing students, the instructor asks, "What is involved in the final processing of somatosensory information?" The correct response includes: Select all that apply. A) Full localization B) Autonomic nervous system C) Discrimination of intensity D) Interpretation of somatosensory stimuli E) Pacinian corpuscle receptors

A. Full localization C. Discrimination of intensity D. Interpretation of somatosensory stimuli

A client has been recently undergone diagnostic testing for possible Berger disease. The nurse caring for this client would anticipate the primary clinical manifestations include which of the following? Select all that apply. A) Gross hematuria B) Recent upper respiratory infection C) Elevated ketone levels in the urine D) Fever, chills, and general body aches

A. Gross hematuria B. Recent upper respiratory infection D. Fever, chills, and general body aches

Loss of pituitary function can result in deficiencies/loss of which of the following hormones' secretions? Select all that apply. A) Growth hormone B) Luteinizing hormone C) Follicle stimulating hormone D) Corticotropin-releasing hormone E) Prolactin

A. Growth hormone B. Luteinizing hormone C. Follicle stimulating hormone E. Prolactin

The parents of a child who has been diagnosed with CKD ask the nurse about manifestations of the disease. Which is the most appropriate information for the nurse to provide? Select all that apply. A) Growth impairment B) Developmental delay C) Early-onset puberty D) Accelerated sexual maturation E) Secondary hyperparathyroidism F) Delayed cognitive development

A. Growth impairment B. Developmental delay E. Secondary hyperparathyroidism F. Delayed cognitive development

Which of the following clients likely faces the highest risk of an acquired hypocoagulation disorder and vitamin K deficiency? A client who: A) Has a diagnosis of liver failure secondary to alcohol abuse B) Has chronic renal failure as a result of type 1 diabetes mellitus C) Is immunocompromised as a result of radiation therapy for the treatment of lung cancer D) Has dehydration and hypokalemia resulting from Clostridium difficile-associated diarrhea

A. Has a diagnosis of liver failure secondary to alcohol abuse

Which of the following would be considered physiologic characteristics of immature or woven bone? It: Select all that apply. A) Has low tensile strength B) Serves as temporary scaffolding for support C) Serves as weight-bearing pillar D) Is only found in mature adults E) Is found in parts of a healing fracture

A. Has low tensile strength B. Serves as temporary scaffolding for support E. Is found in parts of a healing fracture

A high school senior sustained a concussion during a football game. Which of the following signs and symptoms would indicate the presence of postconcussion syndrome in the days and weeks following his injury? A) Headaches and memory lapses B) Recurrent nosebleeds and hypersomnia C) Unilateral weakness and decreased coordination D) Neck pain and decreased neck range of motion

A. Headaches and memory lapses

A client presents to the emergency department with severe menstrual bleeding where she is soaking three to 4 pads/hour. Following assessment, which of the following findings indicates that her body is trying to increase its cardiac output? Select all that apply. A) Heart rate 120 beats/minute B) Deep respirations with expiratory wheezes C) Light pink mucous membranes D) Complaints of chest "palpitations" E) Pale bluish nail beds

A. Heart rate 120 beats/minute D. Complaints of chest "palpitations"

While taking a history from an adult client newly diagnosed with renal cell cancer, the nurse can associate which of the following high-risk factors with the development of this cancer? A) Heavy smoking B) Inherited renal disease C) Adrenal medulla tumors D) Anorexia/bulimia disorder

A. Heavy smoking

A client has fractured his sternum when his chest hit the steering wheel during an accident. Which of the following statements most accurately describes the physiologic function of bone marrow? A) Hematopoiesis takes place in red bone marrow. B) Yellow bone marrow predominates in infants. C) By adulthood, all red bone marrow has been replaced by yellow bone marrow. D) Yellow bone marrow is hematopoietically active in infants, but not in adults.

A. Hematopoiesis takes place in red bone marrow

Select the manifestations of renal cell carcinoma. Select all that apply. A) Hematuria B) Does not metastasize C) Often silent in the early stages D) Palpable flank mass

A. Hematuria C. Often silent in the early stages D. Palpable flank mass

Anemia refers to a deficiency of: A) Hemoglobin B) Blood plasma C) Platelet D) Folic acid

A. Hemoglobin

A client has been told she is at risk for the development of osteogenesis imperfects. Select the statement that best describes the diagnosis. A) Hereditary disorder caused by deficiencies in the synthesis of type I collagen B) Of all hereditary bone diseases, it is not common C) Characterized by thick, well-developed bones that are prone to multiple fractures D) Can be corrected with definitive treatment E) May manifest as thick skin, hypertonic muscles, and tight pointedness

A. Hereditary disorder caused by deficiencies in the synthesis of type I collagen

The client with a suspected diagnosis of osteoarthritis asks the health care provider how the diagnosis will be confirmed. The best response would be: A) History and physical examination, x-ray studies, and laboratory findings that exclude other diseases B) Biopsy C) Bone marrow aspiration D) Exercise trials

A. History and physical examination, x-ray studies, and laboratory findings that exclude other diseases

A nurse is assessing a client for early manifestations of chronic kidney disease (CKD). Which would the nurse expect the client to display? A) Hypertension B) Asterixis C) Impotence D) Terry nails

A. Hypertension

An elderly client is admitted with elevated magnesium level related to a history of renal insufficiency and excess use of antacids and laxatives containing magnesium. On admission assessment, the nurse notes which clinical manifestations that correlate to hypermagnesemia? Select all that apply. A) Hyporeflexia B) Blood pressure 180/90 C) Tetanic muscle contractions D) Muscle weakness causing shallow breathing E) Paresthesia of the lips

A. Hyporeflexia D. Muscle weakness causing shallow breathing

The nurse is caring for a client with isotonic fluid volume deficit. The nurse would anticipate the client to manifest: Select all that apply. A) Hypotension B) Increased hematocrit and BUN C) Decreased body temperature D) Tachycardia and weak pulse E) Decreased urine output F) Weight gain

A. Hypotension B. Increased hematocrit and BUN D. Tachycardia and weak pulse E. Decreased urine output

A clinician is assessing the muscle tone of a client who has been diagnosed with a lower motor neuron (LMN) lesion. Which of the following assessment findings is congruent with the client's diagnosis? A) Hypotonia B) Spasticity C) Tetany D) Rigidity

A. Hypotonia

A nurse is assessing an elderly woman diagnosed with chronic hypothyroidism who has developed myxedematous coma. The nurse is aware that the client may manifest: Select all that apply. A) Hypoventilation B) Lactic acidosis C) Hyperthermia D) Hyponatremia E) Hypoglycemia F) Increased cardiac stimulation

A. Hypoventilation B. Lactic acidosis D. Hyponatremia E. Hypoglycemia

In third-world countries, many young children with severe vitamin C deficiency will exhibit: A) Impaired formation of the organic matrix B) Calcification of new bone growth C) Growth plate separation D) Widening of the cortex

A. Impaired formation of the organic matrix

The pathologic effects of the thalassemias are primarily due to which of the following pathophysiologic processes? A) Impaired hemoglobin synthesis B) Impaired folic acid absorption C) Erythropoietin deficiency D) Loss of iron

A. Impaired hemoglobin synthesis

The nurse is assessing a patient who has unilateral obstruction of the urinary tract. The nurse anticipates the patient may develop: A) Increase in blood pressure B) Inability to control urination C) Increased urine output D) Excretion of dilute urine

A. Increase in blood pressure

A client has recently undergone successful extracorporeal shock wave lithotripsy (ESWL) for the treatment of renal calculi. Which of the following measures should the client integrate into his lifestyle to reduce the risk of recurrence? A) Increased fluid intake and dietary changes B) Weight loss and blood pressure control C) Regular random blood glucose testing D) Increased physical activity and use of over-the-counter diuretics

A. Increased fluid intake and dietary changes

A patient has been diagnosed with a simple cyst of the kidney and is presenting with symptoms. Which does the nurse anticipates the patient will display? Select all that apply. A) Infection B) Hematuria C) Hypertension D)Less than 50 years of age E) Compromised renal function

A. Infection B. Hematuria C. Hypertension

The client has had prolonged urethral outlet obstruction. The nurse knows that physiologically, the client may likely develop small pockets of mucosal tissue, called cellulae, which can ultimately cause: Select all that apply. A) Infections due to stasis B) Backpressure on the ureters C) Development of hydroureters D) Sphincter dystonia

A. Infections due to stasis B. Backpressure on the ureters C. Development of hydroureters

A newly diagnosed type 2 diabetic client has been prescribed metformin. When explaining the actions of this medication, the nurse should include which statement? This medication: A) Inhibits hepatic glucose production and increases the sensitivity of peripheral tissues to the actions of insulin B) Blocks the action of intestinal brush border enzymes that break down complex carbohydrates C) Increases insulin sensitivity in the insulin-responsive tissues—liver, skeletal muscle, and fat—allowing the tissues to respond to endogenous insulin more efficiently D) Acts like a hormone released into the circulation by the gastrointestinal tract after a meal, especially one high in carbohydrates, which amplify the glucose-induced release of insulin

A. Inhibits hepatic glucose production and increases the sensitivity of peripheral tissues to the actions of insulin

The diagnosis of type 1 diabetes would be confirmed by: A) Insulin is not available for use by the body B) Insulin is present in large amounts for use by the body C) Small amounts of insulin are produced daily D) Insulin is produced but unavailable for use in the body

A. Insulin is not available for use by the body

Following a head injury, a client is diagnosed with a possible intracranial epidural hematoma. During the initial assessment, the client suddenly becomes unconscious. Other clinical manifestations that correlate with this diagnosis may include: A) Ipsilateral pupil dilation B) Ipsilateral hemiparesis C) Diffuse venous bleeding from the nose D) Increased head circumference with hydrocephalus

A. Ipsilateral pupil dilation

A client has received too much morphine (narcotic) in the postsurgical recovery room. Blood gas results reveal the patient has developed respiratory acidosis. Which of the following assessment findings correlate with acute primary respiratory acidosis? Select all that apply. A) Irritability B) Tingling/numbness in the fingers and toes C) Muscle twitching D) Respiratory depression E) Cardiac palpitations

A. Irritability C. Muscle twitching D. Respiratory depression

Select the assessment data that place a client at most risk for the development of an opportunistic infection. A client with/who: Select all that apply. A) Is currently receiving chemotherapy B) Has normal body function and weight C) Has a compromised immune system D) Just completed 6 weeks of radiation therapy E) Has a diagnosis of malnutrition

A. Is currently receiving chemotherapy C. Has a compromised immune system D. Just completed 6 weeks of radiation therapy E. Has diagnosis of malnutrition

Following a lengthy series of diagnostic tests, a client's chronic hip pain has been attributed to advanced osteonecrosis. What treatment is this client most likely to require? A) Joint replacement surgery B) Intravenous antibiotics C) Injections of corticosteroids into the synovial space D) Transfusion of packed red blood cells

A. Joint replacement surgery

The nurse is educating a pregnant client about the importance of folic acid. Select the food that has the highest levels of folic acid. A) Kale B) Carrots C) Apples D) Eggs

A. Kale

The parents are ready to take their newly diagnosed hemophilia child home. Which of the following teaching aspects should the nurse discuss with them prior to discharge? Select all that apply. A) Keep the child away from contact sports like football and wrestling. B) Give ibuprofen (an NSAID) if the child runs a fever. C) Aspirin should only be given for severe pain in the joints. D) Administration of factor VIII at home when bleeding occurs. E) The signs of a MI related to bleeding in the heart vessels.

A. Keep the child away from contact sports like football and wrestling D. Administration of factor VIII at home when bleeding occurs

A female client asks the nurse if there is any noninvasive treatment to help with the involuntary loss of urine that occurs when she coughs or sneezes. Which is the best response by the nurse? A) Kegel exercises B) Periurethral injection of a bulking agent C) Implanted artificial sphincter D) Self-catheterizatoin

A. Kegel exercises

A client with a history of heart and kidney failure is brought to the emergency department. Upon assessment/diagnosis, it is determined the client is in decompensated heart failure. Of the following assessment findings, which are associated with excess intracellular water? Select all that apply. A) Lethargy B) Confusion C) Hyperactive deep tendon reflexes D) Seizures E) Firm, rubbery tissue when palpating lower extremities

A. Lethargy B. Confusion D. Seizures

The region of the brain involved in emotional experience and control of emotional behavior is the: A) Limbic system B) Occipital lobe C) Parietal lobe D) Cerebral hemisphere

A. Limbic system

The nurse is developing a plan of care for the management of rheumatic diseases for an elderly client. The most important interventions for the nurse to include would be: Select all that apply. A) Local heat B) Muscle-building exercises C) Frequent rest and decreased ambulation D) Splints and walking aids

A. Local heat B. Muscle-building exercises D. Splints and walking aids

Select the treatment of choice for syphilis. A) Long-acting penicillin in a single injection B) Oral amoxicillin for 10 days C) UV radiation to the genital area D) Topical clotrimazole cream

A. Long-acting penicillin in a single injection

The iatrogenic form of Cushing syndrome is caused by: A) Long-term cortisone therapy B) Pituitary tumor secreting ACTH C) Benign or malignant adrenal tumor D) Ectopic ACTH-secreting lung tumor

A. Long-term cortisone therapy

A client is brought to the emergency department with a suspected diagnosis of DKA (diabetic keto acidosis). Select the assessment/diagnostic data to confirm the diagnosis. Select all that apply. A) Low serum bicarbonate B) Positive urine ketones C) Hypoglycemia D) Negative serum ketones E) High arterial pH

A. Low serum bicarbonate B. Postive urine ketones

A nurse is caring for a patient diagnosed with nephrotic syndrome. The nurse is aware that the patient may manifest which of the following? Select all that apply. A) Massive proteinuria (>3.5 g/day) B) Generalized edema C) Lipiduria D) Hyperalbuminemia E) Hyperlipidemia

A. Massive proteinuria (>3.5 g/day) B. Generalized edema C. Lipiduria E. Hyperlipidemia

Arterial blood gases of a client with a diagnosis of acute renal failure reveal a pH of 7.25, HCO 3 - level of 21 mEq/L, and decreased PCO 2 level accompanied by a respiratory rate of 32. This client is most likely experiencing which disorder of acid-base balance? A) Metabolic acidosis B) Metabolic alkalosis C) Respiratory acidosis D) Respiratory alkalosis

A. Metabolic acidosis

A diabetic client was visiting the endocrinologist for annual checkup. The client's blood work reveals an increased level of which lab result that reveals early signs of diabetic nephropathy? A) Microalbuminuria B) Oliguria C) Hypokalemia D) Hyperlipidemia

A. Microalbuminuria

The most recent assessment of a client with a diagnosis of type 1 diabetes indicates a heightened risk of diabetic nephropathy. Which of the following assessment findings is most suggestive of this increased risk? A) Microalbuminuria B) Hematuria C) Orthostatic hypotension D) Diabetic retinopathy

A. Microalbuminuria

Following the diagnosis of nephrotic syndrome, the nurse knows the clinical manifestations occur as a result of a decreased plasma colloidal osmotic pressure. Therefore, the nurse should assess the client for: Select all that apply. A) Moist crackles in both lung fields B) Areas of diminished breath sounds due to pleural effusions C) Liver enlargement D) Kidneys palpable to deep palpation E) Increased circumference in the abdomen related to fluid excess

A. Moist crackles in both lung fields B. Areas of diminished breath sounds due to pleural effusions E. Increased circumference in the abdomen related to fluid excess

Select the most common symptom a female diagnosed with chlamydial infection will develop. A) Mucopurulent cervical discharge B) Urinary frequency C) Dysuria D) Vaginal discharge

A. Mucopurulent cervical discharge

The somatosensory system consists of three types of sensory neurons. The special somatic type of afferent sensory neurons has receptors that sense: A) Muscle position B) Visceral fullness C) Temperature D) Painful touch

A. Muscle position

A chronic kidney disease client who has renal osteodystrophy should be assessed for which of the following complications? Select all that apply. A) Muscle weakness B) Kidney stones C) Bone pain D) Stress fractures E) Urosepsis

A. Muscle weakness C. Bone pain D. Stress fractures

The nurse on the pediatric unit is implementing distraction strategies for a child who is experiencing pain. The best strategies for the nurse to implement would be: select all that apply. A) Music B) Games C) Bubbles D) Silence E) Television F) Acetaminophen

A. Music B. Games C. Bubbles E. Television

The nurse is caring for a client who has multi nodular goiter. The nurse is aware that the client may be at risk for: Select all that apply. A) Neck vein distention B) Expiratory stridor C) Syncope when coughing D) Relaxing of the trachea E) Choking F) Edema of the eyelids

A. Neck vein distention C. Syncope when coughing E. Choking F. Edema of the eyelids

The health care provider has prescribed an amino glycoside (gentamicin) for a client. The nurse is aware that the client is at risk for: A) Nephrotoxic acute tubular necrosis B) Postrenal failure C) Chronic kidney disease D) Ischemic acute tubular necrosis

A. Nephrotoxic acute tubular necrosis

Which of the following peripheral nerve injuries will likely result in cellular death with little chance of regeneration? A) Nerve fibers destroyed close to the neuronal cell body B) Crushing injury where the nerve is traumatized but not severed C) Cutting injury where slow-regeneration axonal branches are located D) Incomplete amputation where tubular implants are used to fill in the gaps of nerves

A. Nerve fibers destroyed close to the neuronal cell body

While explaining the role of skeletal muscle relaxants, such as succinylcholine, used during anesthesia, the faculty mentions that these effects are caused by blocking: A) Nicotinic acetylcholine receptors, muscle-type receptor (N M ) B) Norepinephrine C) Muscarinic acetylcholine receptors D) N 2 , neuronal-type receptor, a type of nicotinic acetylcholine receptor

A. Nicotinic acetylcholine receptors, muscle-type receptor (NM)

A client diagnosed with Parkinson disease is displaying the following manifestations: tremor, rigidity, and slowness of movement. The nurse would interpret these as: A) Normal manifestations of Parkinson disease B) Signs of clinical improvement C) Signs of clinical deterioration D) Manifestations of another disease process

A. Normal manifestations of Parkinson disease

In persons with a bleeding disorder caused by vascular defects, platelet counts and INR results will most often reveal: A) Normal values B) Hypocalcemia C) Polycythemia D) Thrombocytopenia

A. Normal values

As nitrogenous wastes increase in the blood, the CKD client may exhibit which of the following clinical manifestations? Select all that apply. A) Numbness in lower extremities B) Photophobia C) Extremely low platelet counts D) Restless leg syndrome E) Pruritus

A. Numbness in lower extremities D. Restless leg syndrome E. Pruritus

A drug abuser was found unconscious after shooting up heroin 2 days prior. Because of the pressure placed on the hip and arm, the client has developed rhabdomyolysis. The nurse knows this can: A) Obstruct the renal tubules with myoglobin and damage tubular cells B) Be cured by administering an anticoagulant immediately C) Cause the kidney to develop renal stones due to stasis D) Cause compartment syndrome in the lower extremities

A. Obstruct the renal tubules with myoglobin and damage tubular cells

Which are contributing factors of acute postinfectious glomerulonephritis? Select all that apply. A) Occurs after infection of group A B-hemolytic streptococci B) Primarily infects the pharynx C) May result from impetigo D) Polyuria is the first symptom E) Increased GFR F) Antibodies develop in 2 to 5 days after infection

A. Occurs after infection of group A B-hemolytic streptococci B. Primarily infects the pharynx C. May result from impetigo

The nurse is performing an admission assessment on a client with the diagnosis of primary polycythemia. The nurse anticipates that the assessment will include: Select all that apply. A) Pain B) Hypertension C) Hypotension D) Itching E) Plethora F) Pale skin

A. Pain B. Hypertension D. Itching E. Plethora

A client asks the nurse what the most common sign/symptom of bladder cancer is. Which is the best response by the nurse? A) Painless hematuria B) Intermittent urgency C) Continual dysuria D) Urinary frequency

A. Painless hematuria

A feature of rheumatoid arthritis that differentiates it from other forms of inflammatory arthritis is the development of: A) Pannus tissue B) Tophus deposits C) Subluxations D) Autoantibodies

A. Pannus tissue

Disorders of the pyramidal tracts, such as a stroke, are characterized by: A) Paralysis B) Hypotonia C) Muscle rigidity D) Involuntary movements

A. Paralysis

A newly diagnosed paraplegic client who suffered an automobile accident appears to have control of bladder emptying. The health care provider explains this process to the client/family stating, "This function is allowing the motor component of the neural reflex to assist with bladder emptying and is primarily controlled by the: A) Parasympathetic division of the ANS." B) Sympathetic division of the ANS." C) Somatic nervous system." D) Hypogastric nervous system."

A. Parasympathetic division of the ANS."

What pain theory proposes that pain receptors share pathways with other sensory modalities and that different activity patterns of the same neurons can be used to signal painful or nonpainful stimuli? A) Pattern B) Specificity C) Gate control D) Neuromatrix

A. Pattern

Risk factors of bacterial vaginosis include: Select all that apply. A) Pelvic inflammatory disease (PID) B) Postpartum depression C) Preterm labor D) Premature rupture of membranes E) Postpartum endometritis

A. Pelvic inflammatory disease (PID) C. Preterm labor D. Premature rupture of membranes E. Postpartum endometritis

A nurse is assessing a client diagnosed with CKD for neuromuscular manifestation. Select the manifestations the nurse may expect to find. Select all that apply. A) Perceptual errors B) Increased muscle strength C) Restless leg syndrome D) Loss of recent memory E) Peripheral neuropathy F) Increased alertness

A. Perceptual errors C. Restless leg syndrome D. Loss of recent memory E. Peripheral neuropathy

Following an episode of strep throat, the school nurse notices the fourth grade child has not recovered from this illness a week later. Upon further investigation, the nurse notices that the child has developed water retention. Which of the following assessments support this conclusion? Select all that apply. A) Periorbital edema B) BP 100/70 C) Swelling of the hands and fingers D) Vomiting after intake of any solid food E) Dizziness and right ear pain

A. Periorbital edema B. BP 100/70

While assessing a peritoneal dialysis client in his or her home, the nurse notes that the fluid draining from the abdomen is cloudy, is white in color, and contains a strong odor. The nurse suspects this client has developed a serious complication known as: A) Peritonitis B) Bowel perforation C) Too much sugar in the dialysis solution D) Bladder erosion

A. Peritonitis

A mother has brought her 2-week-old infant to the emergency department due to the baby's persistent and increasing jaundice. Blood testing reveals that the infant's unconjugated bilirubin level is 28 mg/dL, and assessment does not reveal neurologic deficits. The infant's weight is normal, and the mother claims to have had no significant difficulty feeding the infant. The most likely treatment for this infant will be: A) Phototherapy B) Packed red blood cell transfusion C) Phlebotomy D) Intravenous antibiotics

A. Phototherapy

A child has developed a brain tumor in the walls of the cerebellum. The health care provider would document this as: A) Pilocytic astrocytomas B) Fibrillary astrocytomas C) Oligodendrogliomas D) Ependymomas

A. Pilocytic astrocytomas

An elderly female complains about waking up one morning with pain/stiffness in her neck/shoulders. Lab work reveals an elevated erythrocyte sedimentation rate (ESR). The physician gives the client a 3-day trial of prednisone, which significantly improves the pain. The health care provider correlates this information and diagnoses which of the following disorders? A) Polymyalgia rheumatica B) Psoriatic arthritis C) Reiter syndrome D) Ankylosing spondylitis

A. Polymyalgia rheumatica

Select the most common symptoms of diabetes. Select all that apply. A) Polyphagia B) Polydipsia C) Polyhydramnios D) Polyuria E) Polycythemia

A. Polyphagia B. Polydipsia D. Polyuria

A heart failure client has gotten confused and took too many of his "water pills" (diuretics). On admission, his serum potassium level was 2.6 mEq/L. Of the following assessments, which correlate to this hypokalemia finding? Select all that apply. A) Polyuria B) Constipation C) Bradycardia D) Paresthesia with numbness of the lips/mouth E) ECG showing short runs of ventricular fibrillation

A. Polyuria B. Constipation D. Paresthesia with numbness of the lips/mouth

Which of the following diagnostic and assessment results support the diagnosis of chronic pyelonephritis? Select all that apply. A) Polyuria (excess urine output) B) Nocturia (voiding at night) C) Bilateral flank pain D) Blood pressure 140/92 E) Severe pain in upper outer quadrant of the abdomen

A. Polyuria (excessive urine output) B. Nocturia (voiding at night)

The school nurse is performing annual screenings of the adolescent students. An adolescent is noted to have a lateral curvature in the spine in the upright position; however, the curvature corrects with bending. The nurse would interpret this as: A) Postural scoliosis B) Congenital scoliosis C) Neuromuscular scoliosis D) Structural scoliosis

A. Postural scoliosis

More complex patterns of movements, such as throwing a ball or picking up a fork, are controlled by which portion of the frontal lobe? A) Premotor cortex B) Primary motor cortex C) Reflexive circuitry D) Supplementary motor cortex

A. Premotor cortex

A 69-year-old client who is obese and has a diagnosis of angina pectoris has been prescribed clopidogrel (Plavix) by his primary care provider. The client asks, "Why do I need this medication? It won't help my chest pain." The best response would be this medication: A) Prevents the blood cells from forming a clot in your heart vessels B) Activates plasminogen, which converts to plasmin to digest clots in your heart vessels C) Inhibits the intrinsic clotting pathway to keep RBCs from clumping together in the heart vessels D) Inactivates calcium ions, thereby preventing blood clotting

A. Prevents the blood cells from forming a clot in your heart vessels

Which clients have an increased risk for developing UTIs? A client who is or is diagnosed with: Select all that apply. A) Prostate disease B) Not sexually active C) Elderly D) Neurogenic disorders E) Urinary obstruction F) A premenopausal woman

A. Prostate disease C. Elderly D. Neurogenic disorders E. Urinary obstruction

When acute tubular necrosis (ATN) is suspected, the nurse will likely see which of the following laboratory findings on the urinalysis report? Select all that apply. A) Protein B) Glucose C) Red blood cells D) Sodium excess E) Cast cells

A. Protein C. Red blood cells E. Cast cells

Which symptom occurs in client with chronic kidney disease (CKD) as a result of elevated serum phosphate levels and the development of phosphate crystals that occur with hyperparathyroidism? A) Pruritus B) Azotemia C) Asterixis D) Uremia

A. Pruritus

What is the effect of glucagon? A) Raise blood sugar B) Lower blood sugar C) Raise sodium levels in the blood D) Lower sodium levels in the blood

A. Raise blood sugar

Following a car accident that has resulted in partial amputation of the lower limbs, the client's body has implemented a compensatory mechanism releasing antidiuretic hormone (ADH) into the blood stream, causing retention of water and vasoconstriction of blood vessels. This is accomplished as a result of: A) Rapid axonal transport of ADH from the hypothalamus into the posterior pituitary B) Extension of Nissl bodies and free ribosomes carrying ADH into the dendrites C) Dendrites conducting information and ADH toward the cell body D) Afferent, or sensory, neurons of the PNS transmitting information to the CNS

A. Rapid axonal transport of ADH from the hypothalamus into the posterior pituitary

Following his annual influenza vaccination, a client begins to feel achy, like he has developed the flu. An hour later, the client is rushed to the emergency department. Diagnosis of Guillain-Barré syndrome was made based on which of the following assessment findings? Select all that apply. A) Rapid deterioration of respiratory status B) Lack of any physical pain C) Flaccid paralysis of limbs D) BP 90/62 E) Pale, cool, dry skin

A. Rapid deterioration of respiratory status C. Flaccid paralysis of limbs D. BP 90/62

Knowing that thrombotic thrombocytopenic purpura (TTP) results in thrombi in the microcirculation system, the health care worker should assess the client for which of the following manifestations? Select all that apply. A) Red/purple skin discoloration that does not blanch when pressure is applied (purpura) B) Jaundice noted on skin and sclera of the eyes C) Petechiae over the entire body D) Erythema around all orifices E) Confusion or seizures caused by decreased circulation to the brain from vascular occlusion

A. Red/purple skin discoloration that does not blanch when pressure is applied (purpura) C. Petechiae over the entire body E. Confusion or seizures caused by decreased circulation to the brain from vascular occlusion

A client's primary care provider has prescribed a β-adrenergic receptor blocker. Which of the following therapeutic effects do the client and care provider likely seek? A) Reduction in heart rate and blood pressure B) Slowing of gastrointestinal motility C) Increase in mental acuity D) Decreased production of gastric acid

A. Reduction in heart rate and blood pressure

While walking down the street, a cat jumps out of an alley into the pathway of an adult. They note that his heart is "racing." This response is primarily due to: A) Reflex circuitry produced by the ANS reflexes B) Parasympathetic effects on the vagus nerve C) Secretions of sympathetic neurotransmitters produced in the adrenal medulla D) The cell body of the first motor neuron that lies in the brain stem

A. Reflex circuitry produced by the ANS reflexes

A woman has sought care because of recurrent urinary tract infections, which have been increasing in both frequency and severity. The health care worker should explain which of the following physiological factors to the client that is likely contributing to recurrent UTIs? A) Reflux flow of urine that can occur from coughing or sneezing B) Fluctuations in urine pH related to beverage consumption C) Urethral trauma that occurs during sexual intercourse D) Inadequate intake of water

A. Reflux flow of urine the can occur from coughing or sneezing

Coagulation activated by the extrinsic pathways initiated when: A) Release of thromboplastin from the injured vessel wall. B) Blood is exposed to tissue extracts C) Factor X is activated D) An increase in blood volume occurs in the vascular system

A. Release of thromboplastin from the injured vessel wall

A nurse is caring for a client diagnosed with amyotrophic lateral sclerosis (ALS). The nurse determines that the client is at greater risk for the development of: A) Respiratory impairment B) Generalized fatigue C) Severe dysarthria D) Weakness in one leg

A. Respiratory impairment

A client diagnosed with chronic kidney disease (CKD) is experiencing nausea and vomiting. Which would be the best instruction for the nurse to provide? A) Restrict intake of dietary protein B) Increase intake of carbohydrates C) Restrict intake of dietary fat D) Increase intake of fruit juice

A. Restrict intake of dietary protein

When comparing a child's clinical manifestations with that of oligoarthritis versus systemic onset, the health care provider diagnoses this in your 4-year-old child with oligoarthritis based on which of the following clinical findings? A) Right knee is warm and painful when putting it through normal range of motion. B) Faint, red macular rash noted over entire body. C) Rash is diffuse with severe itching. D) Daily has an intermittent elevated temperature.

A. Right knee is warm and painful when putting it through normal range of motion

A client has just undergone a diagnostic cardiac angiogram. As part of their ordered labs, the physician has ordered a thyroid panel. The physiological principle behind ordering this lab tests includes which of the following correlations? Hyperthyroidism can cause: Select all that apply. A) Rise in oxygen consumption B) Sharp decrease in heart rate and blood pressure C) Increase in cardiac output D) Vasoconstriction of all arteries

A. Rise in oxygen consumption C. Increase in cardiac output

A client presents to the pain clinic for a steroid injection into the spine due to increasing pain around the joints. The health care provider tells the client, "You have inflammation where your tendons/ligaments insert into the bone. This injection should help." The nurse assisting with the procedure recognizes this to be characteristic of: A) Sacroiliitis B) Calcinosis C) Excessive bone turnover D) Autoimmune etiology

A. Sacroiliitis

A 23-year-old diagnosed with syphilis presents with palmar rash, sore throat, fever, and a red-brown lesion on the genital area. The symptoms hav been present for 3 weeks. Select the stage of syphilis this client is currently in. A) Secondary B) Primary C) Tertiary D) Latent

A. Secondary

The health care provider has evaluated a client diagnosed with osteoporosis and determined that pharmacological therapy will benefit the client. Select the medication options that the provider may prescribe. Select all that apply. A) Selective estrogen receptor modulators (SERMs) B) RANKL inhibitor denosumab C) Recombinant parathyroid hormone D) Bisphosphonates E) Calcitonin F) Thyroid hormone replacement

A. Selective estrogen receptor modulators (SERMs) B. RANKL inhibitor denosumab C. Recombinant parathyroid hormone D. Biphosphaonates E. Calcintonin

Select the most reliable test to diagnose syphilis. A) Serology B) Urine culture C) Oral swab D) Chest x-ray

A. Serology

An 86-year-old female client has been admitted to the hospital for the treatment of dehydration and hyponatremia after she curtailed her fluid intake to minimize urinary incontinence. The client's admitting laboratory results are suggestive of prerenal failure. The nurse should be assessing this client for which of the following early signs of prerenal injury? A) Sharp decrease in urine output B) Excessive voiding of clear urine C) Acute hypertensive crisis D) Intermittent periods of confusion

A. Sharp decrease in urine output

A family brings their father to his primary care physician for a checkup. Since their last visit, they note their dad has developed a tremor in his hands and feet. He also rolls his fingers like he has a marble in his hand. The primary physician suspects the onset of Parkinson disease when he notes which of the following abnormalities in the client's gait? A) Slow to start walking and has difficulty when asked to "stop" suddenly B) Difficulty putting weight on soles of feet and tends to walk on tiptoes C) Hyperactive leg motions like he just can't stand still D) Takes large, exaggerated strides and swings arms/hands wildly

A. Slow to start walking and has difficulty when asked to "stop" suddenly

A public health nurse is conducting a health promotion campaign under the auspices of the local community center. Which of the following measures that the nurse is promoting are likely to influence the participants' risk of hypercoagulability disorders? Select all that apply. A) Smoking cessation B) Blood glucose screening C) Daily dose of vitamins with minerals D) Cholesterol screening and management E) Eliminating consumption of all alcoholic beverages

A. Smoking cessation B. Blood glucose screening D. Cholesterol screening and management

The nurse is caring for a client who is experiencing an increased level of aldosterone secretion. The nurse anticipates that the client may develop: A) Sodium and water retention B) Potassium and sodium excretion C) Potassium retention and water excretion D) Water and potassium retention

A. Sodium and water retention

When a person is stung on the index finger by a bee, the thalamus interprets the pain as: A) Somewhere on the hand B) A spot on the index finger C) Attributable to a bee stung D) Similar to a previous bee sting

A. Somewhere on the hand

A clinic nurse plans care for a newly diagnosed osteoarthritis client. Which of the following items should this client be provided with educational materials? Select all that apply. A) Splints to protect and rest the involved joint B) Use of heat and cold when appropriate for muscle spasms and pain C) How to use a cane or walker if hip/knees joints are involved D) High dose of daily glucosamine and chondroitin E) Narcotics to help control nighttime pain

A. Splints to protect and rest the involved joint B. Use of heat and cold when appropriate for muscle spasms and pain C. How to use a cane or walker if hip/knees joints are involved

When lecturing about heart attacks (myocardial infarctions), the instructor will emphasize the client may present with: Select all that apply. A) Substernal chest pain B) Neck pain C) Umbilicus pain D) Deep, right-sided abdominal pain E) Pain that radiates to the left arm

A. Substernal chest pain B. Neck pain E. Pain that radiates to the left arm

Select the response that would best describe to a family member why early symptoms of Alzheimer dementia can be difficult to diagnose in the elderly. A) Subtle short-term memory loss that is difficult to differentiate from normal forgetfulness B) Eats two meals a day because of complaints that she has a loss of appetite C) Refusal to drive a motor vehicle to run errands for herself when she is capable D) Inability to walk long distances when she was able to do so without difficulty

A. Subtle short-term memory loss that is difficult to differentiate from normal forgetfulness

While explaining the role of synovial fluid, the instructor rewards which student for identifying an accurate response? Select all that apply. A) Supplies nutrient B) Forces water out of the cartilage matrix C) Supplies O2 to chondrocytes D) Brings cells to phagocytize debris in joints E) Lubricates the joint

A. Supplies nutrient C. Supplies O2 to chondrocytes D. Brings cells to phagocytize debris in joints E. Lubricates the joint

A client with a history of antiphospholipid syndrome presents to the emergency department. Which of the following complaints lead the nurse to suspect the client is experiencing a complication related to the syndrome? Select all that apply. A) Swelling in the right calf with redness and tenderness B) Having respiratory symptoms like shortness of breath and chest pain C) Have developed a cough that produced some bloody sputum D) Developed a headache that has lasted for 3 days now E) Having trouble making "water" and has pain in the lower abdomen and groin

A. Swelling in the right calf with redness and tenderness B. Having respiratory symptoms like shortness of breath and chest pain C. Have developed a cough that produced some bloody sputum

A client has developed the facial appearance that is characteristic of myxedema, along with an enlarged tongue, bradycardia, and voice changes. Which of the following treatment modalities is most likely to benefit this client? A) Synthetic preparations of T 3 or T 4 B) β-Adrenergic blocking drugs C) Corticosteroid replacement therapy D) Oral or parenteral cortisol replacement

A. Synthetic preparations of T 3 or T 4

Following an automobile accident that resulted in a traumatic amputation of the right lower leg, the client complains of feeling tingling, heaviness, and shooting pain in the amputated limb. The health care providers treat phantom limb pain by which of the following interventions? Select all that apply. A) TENS of the large myelinated afferents innervating the area B) Hypnosis C) Relaxation techniques D) Warm, moist compresses E) Use of mirrors to visualize the limb is no longer there

A. TENS of the large myelinated afferents innervating the area B. Hypnosis C. Relaxation techniques

Which of the following clinical manifestations following thyroidectomy would alert the nurse that the client is going into a life-threatening thyroid storm?Select all that apply. A) Temperature of 104.2°F B) Telemetry showing heart rate of 184 C) Unable to close eyelids completely together D) Extremely agitated E) Bruising on knees and feet

A. Temperature of 104.2°F B. Telemetry showing heart rate of 184 D. Extremely agitated

Which of the following physiologic principles would be considered a function of the somatic nervous system? A) The act of typing a report using a computer keyboard B) Withdrawing the hand after touching a hot surface C) The patellar reflex "knee jerk"; activated by tapping the patellar tendon abdominal viscera D) The beginning of depolarization in the cardiac conduction of impulses

A. The act of typing a report using a computer keyboard

The nurse is explaining how vasogenic brain edema occurs to a client's family. The most appropriate information for the nurse to provide would be: A) The blood-brain barrier is disrupted, allowing fluid to escape into the extracellular fluid B) There is an increase in the production of cerebral spinal fluid volume C) Normal physiological circumstances result in decreased adsorption of CSF D) There is a decrease in the amount of fluid volume in the brain

A. The blood-brain barrier is disrupted, allowing fluid to escape into the extracellular fluid

During physiology class, the instructor asks students to explain the pathology behind development of multiple sclerosis. Which student gave the most accurate description? A) The demyelination and subsequent degeneration of nerve fibers and decreased oligodendrocytes, which interfere with nerve conduction B) Muscle necrosis with resultant increase in fat/connective tissue replacing the muscle fibers C) Atherosclerotic destruction of circulation to the brain resulting in lactic acid buildup that affects nerve transmission D) Autoimmune disease where antibody loss of acetylcholine receptors at the neuromuscular junction causes decrease motor response

A. The demyelination and subsequent degeneration of nerve fibers and decreased oligodendrocytes, which interfere with nerve conduction

Which of the following best describes a secondary disorder of endocrine function? A) The disorder occurs when the target organ is normal, but stimulating hormones alter its function B) The disorder occurs in the target gland responsible for producing the hormone C) The disorder occurs when the target organ becomes dysfunctional D) The disorder occurs from hypothalamic dysfunction resulting in under stimulation of the target organ

A. The disorder occurs when the target organ is normal, but stimulating hormones alter its function

A college baseball player has seen his season cut short by a rotator cuff injury. Rotator cuff injuries are frequent because of: A) The inherent instability of the shoulder B) The absence of ligaments at the glenohumeral joint C) The vulnerability of the shoulder menisci D) The large mass of the humeral head

A. The inherent instability of the shoulder

Which of the following joints is classified as a synarthrosis? A) The joint between two vertebrae B) The joint between the femur and the pelvis C) The joint between the humerus and the radius and ulna D) An interphalangeal joint of the hand (knuckle)

A. The joint between two vertebrae

A neuron has been hyperpolarized. How will this affect the excitability of the neuron? A) The neuron will have a membrane potential farther from the threshold. B) The neuron will be more difficult to repolarize after firing. C) The membrane potential of the neuron will be closer to the threshold. D) The neuron's excitability will be significantly increased.

A. The neuron will have a membrane potential farther from the threshold

A 42-year-old male client recently diagnosed with liver cancer is noted as at high risk for bleeding abnormalities. The nurse recognizes this risk as a result of: A) The reduction of clotting factors synthesized in the liver B) Increased amounts of vitamin K produced in the liver C) Weakening of the organ walls as a result of inflammation D) Vitamin C deficiency in the diet

A. The reduction of clotting factors synthesized in the liver

Which of the following messages is most likely to be carried by general somatic afferent (GSA) neurons? A) The sensation of cold when touching ice B) The message to move a finger and thumb C) The message to move the larynx during speech D) Information about the position of a joint

A. The sensation of cold when touching ice

A client with type 2 diabetes has routine lab work, which reveals elevated free fatty acids (FFA). The client asks, "Why is this significant?" The most accurate response would be: Select all that apply. A) This may increase the amount of triglyceride (a form of fat) stored in your liver or around your heart. B) Your pancreas is affected by increased fat (lipotoxicity), which causes beta cell dysfunction, leading to the need for insulin. C) Excess fat in the liver causes a decrease in hepatic glucose production leading to severe hypoglycemia. D) Nonalcoholic fatty liver disease may lead to needing a liver transplant. E) Excess fatty acids may interfere with the way your body responds to an infection.

A. This may increase the amount of triglyceride (a form of fat) stored in your liver or around your heart B. Your pancreas is affected by increased fat (lipotoxicity), which causes beta cell dysfunction, leading to the need for insulin

A client who has just undergone a thyroidectomy is experiencing high fever, tachycardia, and extreme restlessness. The nurse would interpret these manifestations as: A) Thyroid crisis B) Myxedematous coma C) Addisonian crisis D) Hypothyroidism

A. Thyroid crisis

An infant from parents of Mediterranean decent has been diagnosed with a severe form of β-thalassemia anemia. The nurse caring for this infant knows that the infant will likely receive which of the following medical treatments? A) Transfusion therapy B) Iron sulfate supplements C) Stem cell transplant D) Warfarin, a blood thinner to decrease clot formation

A. Transfusion therapy

Hypercoagulability states increase the risk of thrombus formation. A) True B) False

A. True

Platelet disorders are bleeding disorders. A) True B) False

A. True

Type 2 DM is more common than type 1 DM. A) True B) False

A. True

The renal excretion of drugs in children is dependent on which factors? Select all that apply. A) Tubular secretion B) Renal blood flow C) Glomerular filtration rate D) Child's weight E) Urinary output F) Oral fluid intake

A. Tubular secretion B. Renal blood flow C. Glomerular filtration rate

The health care provider suspects a newborn infant may have bilateral developmental dysplasia and is prescribing diagnostic tests to confirm the diagnosis. Select the most appropriate test. A) Ultrasonography B) Barlow maneuver C) Galeazzi sign D) Ortolani maneuver

A. Ultrasonography

Which of the following signs and symptoms in a 2-year-old child should prompt assessment for a urinary tract infection? A) Unexplained fever and anorexia B) Decreased urine output and irritability C) Production of concentrated urine and recurrent nausea D) Frank hematuria

A. Unexplained fever and anorexia

The most damaging effects of urinary obstruction are the result unrelieved obstruction of urine outflow and: A) Urinary stasis B) Concentrated urine C) Kidney hyperplasia D) Renal hypertension

A. Urinary stasis

A child presents with urinary symptoms; however, the urine culture is negative for bacteriuria. Which diagnoses should be further assessed? A) Vaginitis, sexual molestation, or pinworms B) Overflow urge, bladder hypertrophy, or overactive bladder C) Acute abdomen, renal stones, or spastic bladder dysfunction D) Constipation, irritable bowels, or flaccid bladder

A. Vaginitis, sexual molestation, or pinworms

A client with a history of migraine headaches tells the physician that he or she usually experiences an aura before the onset of the headache. The client is most likely experiencing: A) Visual disturbances B) Dizziness C) Lethargy D) Dysphasia

A. Visual disturbances

To form a platelet plug, platelets must adhere to the vessel inner layer. For this to occur, which protein molecule is required? A) Von Willebrand factor B) Plasminogen C) Lipoprotein D) Thromboxane A 2

A. Von Willebrand factor

A client has been diagnosed with a cerebral aneurysm and placed under close observation before treatment commences. Which of the following pathophysiologic conditions has contributed to this client's diagnosis? A) Weakness in the muscular wall of an artery B) Impaired synthesis of clotting factors C) Deficits in the autonomic control of blood pressure D) Increased levels of cerebrospinal fluid

A. Weakness in the muscular wall of an artery

A client is admitted in the ICU with diagnosis of hyperglycemic hyperosmolar state (HHS). The nurse caring for the client knows that the client's elevated serum osmolality has pulled water out of this brain cells based on which of the following assessment findings? Select all that apply. A) Weakness one side of the body B) After the sole of the foot has been firmly stroked, the toes flex and flare out C) Increase in urine output in proportion to the increase in blood glucose D) Unable to respond verbally to questions E) Uncontrollable twitching of a muscle group

A. Weakness one side of the body B. After the sole of the foot has been firmly stroked, the toes flex and flare out D. Unable to respond verbally to questions E. Uncontrollable twitching of a muscle group

A student nurse is taking a test on the endocrine system. From the following list of clinical manifestations, she needs to select the ones she would see in hypothyroidism. Which answers should she select? Select all that apply. A) Weight gain despite loss of appetite B) Nervousness with fine muscle tremors C) Coarse brittle hair D) Heat intolerance E) Puffy face with swollen eyelids

A. Weight gain despite loss of appetite C. Coarse brittle hair E. Puffy face with swollen eyelids

Which of the following clients is most susceptible to experiencing the effects of inadequate erythropoiesis? A client: A) Who has developed renal failure as a result of long-standing hypertension B) Who recently experienced an ischemic stroke and who remains bedridden C) Whose heavy alcohol use has culminated in a diagnosis of pancreatitis D) Whose estimated blood loss during recent surgery was 700 mL

A. Who has developed renal failure as a result of long-standing hypertension

Which of the following clinical manifestations lead the health care worker to suspect the client is at the end-stage expression of hypothyroidism? A client: Select all that apply. A) Who takes analgesics for chronic pain that goes into a coma B) Brought to the emergency department with hypothermia who presents with low serum sodium levels C) In the emergency department presenting with tachycardia and palpitations D) Whose family took him to the health care provider complaining of shortness of breath and heat intolerance E) Who has abnormal retraction of eyelids and infrequent blinking

A. Who takes analgesics for chronic pain that goes into a coma B. Brought to the emergency department with hypothermia who presents with low serum sodium levels

A nurse is caring for a client in the intensive care unit who has sustained severe trauma and now has developed disseminated intravascular coagulation (DIC). The nurse is aware that the client is experiencing: A) Widespread coagulation and bleeding in the vascular compartment B) Bleeding due to structurally weak vessels resulting from vitamin C deficiency C) Impaired platelet function due to vitamin K deficiency D) Thrombocytosis as a result of widespread infection

A. Widespread coagulation and bleeding in the vascular compartment

Unlike disorders of the motor cortex and corticospinal (pyramidal) tract, lesions of the basal ganglia disrupt movement: A) Without causing paralysis B) Posture and muscle tone C) And cortical responses D) Of upper motor neurons

A. Without causing paralysis

Which of the following lab results may be associated with metastatic bone disease? Select all that apply. A) Elevated alkaline phosphatase B) Decreased creatinine levels C) High serum calcium levels D) Lower serum phosphate levels

A. elevated alkaline phosphate C. high serum calcium levels

Which of the following assessment findings on an infant lead the nurse to suspect developmental dysplasia of the hip (DDH), formerly known as congenital hip dislocation? Select all that apply. A) Gluteal fold asymmetry B) Shortening of the thigh, so the knee on affected side is higher C) Joint capsule tightness to the point that there is no normal range of motion D) Delayed crawling on the knees

A. gluteal fold asymmetry B. shortening of the thigh, so the knee on affected side is higher

The client has a fractured tibia. After the cast is applied, he is at high risk for compartment syndrome caused by: A) Inflammation B) Joint immobility C) Muscle atrophy D) Extremity elevation

A. inflammation

The nurse would anticipate the laboratory results of a client experiencing metabolic acidosis to include: A) pH of 7.25 and HCO3 of 18 mEq/L B) pH of 7.45 and HCO3 of 24 mEq/L C) pH of 7.50 and HCO3 of 45 mEq/L D) pH of 7.35 and HCO3 of mEq/L

A. pH of 7.25 and HCO3 of 18 mEq/L

The distinguishing characteristic of chronic osteomyelitis is the presence of: A) Sequestrum bone B) Abscess formation C) Severe bone pain D) External drainage

A. sequestrum bone

A client has experiences severe hemorrhage and is in prerenal failure. The nurse anticipates the laboratory results of the client's BUN and serum creatinine to identify: A) The BUN-to-creatinine ratio is 10:1 B) The BUN-to-creatinine ration is 20:1 C) Creatinine level rises and BUN decreases D) BUN elevates and creatinine decreases

B) The BUN-to-creatinine ration is 20:1

A 35-year-old client is diagnosed with acute kidney injury (AKI) and is started on hemodialysis. The client is concerned with the diagnosis and wants to know what to expect in the progression of this disorder. Which statement best addresses the client's concern? A) "The occurrence of acute kidney injury will always eventually result in chronic renal failure." B) "Acute kidney injury is abrupt in onset and often reversible if recognized early and treated appropriately." C) "Once your condition improves, you can be placed on peritoneal dialysis for the rest of your life." D) "You will need to have a renal transplant to live a productive, healthy life."

B. "Acute kidney injury is abrupt in onset and often reversible if recognized early and treated appropriately."

A client with chronic low back pain presents to the clinic. In addition to a detailed pain assessment, which of the following questions would be appropriate to ask? Select all that apply. A) "Do you have trouble making water?" B) "Can you financially afford your medicine?" C) "What kind of stressors are you experiencing?" D) "Do you consider yourself a good driver?" E) "Are you having trouble sleeping?"

B. "Can you financially afford your medicine?" C. "What kind of stressors are you experiencing?" E. "Are you having trouble sleeping?"

The nurse is scheduled to teach a client experiencing urinary incontinence about Kegel exercises. Which of the following descriptors should the nurse include in this education? A) "Drink at least two glasses of water and then try to hold it for at least 3 hours before going to the bathroom." B) "Contract and relax the pelvic floor muscles at least 10 times every hour while awake." C) "After you have emptied your bladder, continue sitting on the commode and try to forcefully expel more urine." D) "Try to start and stop urination while sitting in a bathtub full of warm soapy water."

B. "Contract and relax the pelvic floor muscles at least 10 times every hour while awake."

A parent tells the nurse he is afraid his child may have muscular dystrophy. The most important question for the nurse to ask would be: A) "Is your child able to use two fingers to hold an object?" B) "Have you noticed your child experiencing frequent falls?" C) "Has your child failed to meet his developmental milestones?" D) "Have you noticed a delayed or slow onset of drawing?"

B. "Have you noticed your child experiencing frequent falls?"

A client diagnosed with type 2 diabetes has been instructed about managing his condition with diet. The nurse determines further teaching is necessary when the client states: A) "I need to limit the amount of foods that I eat that contain trans fats." B) "I must avoid all candies and cookies, but can eat unlimited amounts of pasta and breads." C) "I can drink 8 to 10 glasses of water daily without concern for calories." D) "I need to avoid adding salt to my foods."

B. "I must avoid all candies and cookies, but can eat unlimited amounts of pasta and breads."

The mother of a 2-year-old newly diagnosed with type 1 diabetes asks why insulin has to be given by injection. The best response by the nurse is: A) "When you child gets old enough, you will not have to administer injections." B)"Insulin is destroyed by the stomach contents and has to be administered by injection." C) "Your child is not old enough to swallow the pills needed to treat her diabetes." D) "Insulin needs to go directly into the vein to work best."

B. "Insulin is destroyed by the stomach contents and has to be administered by injection."

A public health nurse is conducting a health promotion class for a group of older adults. Which of the participants' following statements demonstrates an accurate understanding of the risk factors for bladder cancer? A) "I suppose I should listen to my doctor and drink more cranberry juice." B) "More than ever, I guess it would worthwhile for me to quit smoking." C) "I can see that preventing bladder cancer is one more benefit of a healthy diet." D) "I think I should be okay because there's no history of bladder cancer in my family that I'm aware of."

B. "More than ever, I guess it would worthwhile for me to quit smoking."

A nurse is assessing the pain level of an adult with the use of a visual analog scale. Select the response that best indicates the client's rating using this tool. A) "My pain level is at a 6 out of 10." B) "My pain level falls above the 5-cm mark." C) "My pain level looks like the smiley face." D) "My pain is level moderate."

B. "My pain level falls above the 5-cm mark."

A 12-month-old infant is displaying pale skin, rapid heart rate, and increased respiratory rate. The parents bring the child to the clinic. Which statement by the parents points the health care worker to suspect iron deficiency anemia? A) "He has one to two bowel movements/day." B) "We give him regular cow's milk with all his meals and snacks." C) "He doesn't like to eat any vegetables." D) "His grandma feeds him cookies every time she comes to visit."

B. "We give him regular cow's milk with all his meals and snacks."

An 85-year-old male has been brought to the emergency department by his family. Routine lab work reveals low hemoglobin of 8.7 g/dL. While taking a detailed history, which of the following statements by the client/family correlate with this anemia? Select all that apply. A) "I get up to the bathroom two to three times/night." B) "When I go food shopping, I have to sit down and rest after one or two aisles." C) "He seems to get confused once in a while." D) "Some nights I just don't feel like eating a big meal." E) "Every now and then, my big toes gets swollen and hurts real bad."

B. "When I go food shopping, I have to sit down and rest after one or two aisles." C. "He seems to get confused once in a while."

The nurse has just received the lab results of a client's calcium level. The nurse identifies a normal calcium level as: A) 3.5 to 5.3 mg/dL B) 9.0 to 10.5 mg/dL C) 12.0 to 15.0 mg/dL D) 13.5 to 14.5 mg/dL

B. 9.0 to 10.5 mg/dL

A hospital client has been complaining of increasing fatigue for several hours, and his nurse has entered his room to find him unarousable. The nurse immediately checked the client's blood glucose level (and reverified with a second blood glucose meter), which is 22 mg/dL (1.2 mmol/L). The nurse should prepare to administer which of the following? A) A snack that combines simple sugars, protein, and complex carbohydrates B) A 50% glucose solution intravenously C) Infusion of rapid-acting insulin D) An oral solution containing glucagon and simple sugars

B. A 50% glucose solution intravenously

Neurotrophic factors contribute to the maintenance of homeostasis by promoting the growth and survival of neurons. Which of the following clients may be experiencing an alteration in neurotrophin levels? Select all that apply. A) A 92-year-old who fell and fractured the hip getting out of a shower B) A 55-year-old who is exhibiting clinical manifestations of early-onset Alzheimer disease C) A 38-year-old recently diagnosed with multiple sclerosis following initial complaint of vision loss as he was driving to work D) A 44-year-old with a family history of Huntington disease who is exhibiting jerky, uncontrollable movements

B. A 55-year-old who is exhibiting clinical manifestations of early-onset Alzheimer disease D. A 44-year-old with a family history of Huntington disease who is exhibiting jerky, uncontrollable movements

Select the client who is at greatest risk of developing bladder cancer. A) A 40-year-old African American male with a history of prostate enlargement B) A 65-year-old white male with a history of bladder stones C) A 30-year-old African American female with a 2-year history of diabetes D) A 50-year-old white female with a history of two bladder infections

B. A 65-year-old white male with a history of bladder stones

For many clients, the first indication that they have osteoporosis is: A) Bone pain that is not alleviated by rest B) A bone fracture C) Craving high-calcium foods D) Decrease in range of motion in the hip and knee joints

B. A bone fracture

Which of the following clients may be experiencing the effects of neuropathic pain? A) A girl whose playground accident resulted in an arm fracture B) A man with pain secondary to his poorly controlled diabetes C) An elderly woman with a stage III pressure ulcer D) A man whose pain is caused by gastric cancer

B. A man with pain secondary to his poorly controlled diabetes

Which of the following clients presenting to the emergency department would most likely be diagnosed with a pathologic stress fracture? A) A teenager who fell of a ladder and hit the concrete driveway, landing on his hip B) A postmenopausal female who was diagnosed with breast cancer with metastasis to bone C) A competitive volleyball player diving to retrieve a volley and landing on his hip D) A weight lifter who bench-presses 200 kg lost balance and fell to the side, landing on his hip

B. A postmenopausal female who was diagnosed with breast cancer with metastasis to bone

The nurse recognizes the most common cause of acute post infectious glomerulonephritis as: A) Prolonged blockage of the ureter with a stone B) A streptococcal infection 7 to 12 days prior to onset C) Uncontrolled diabetes with increased proteinuria D) Drug-induced damage to the renal glomeruli

B. A streptococcal infection 7 to 12 days prior to onset

Which of the following clients are at risk for developing a platelet clot? Select all that apply. A) A COPD client experiencing an acute exacerbation requiring intravenous steroids B) A trauma client who had a splenectomy following injury and laceration of the liver C) A school-aged child prescribed iron supplements for iron deficiency anemia D) A breast cancer client receiving chemotherapy E) An acute myocardial infarction client with elevated troponin levels

B. A trauma client who had a splenectomy following injury and laceration of the liver C. A school-aged child prescribed iron supplements for iron deficiency anemia D. A breast cancer client receiving chemotherapy E. An acute myocardial infarction client with elevated troponin levels

A diabetic client with a history of hypertension may receive a prescription for which medication to provide a renal protective effect by reducing intraglomerular pressure? Select all that apply. A) Loop diuretics B) ACE inhibitors C) Angiotensin receptor blockers D) Calcium channel blockers E) A digitalis preparation

B. ACE inhibitors C. Angiotensin receptor blockers

Which of the following statements about the use of angiotensin-converting enzyme inhibitor medications and autosomal recessive polycystic kidney disease (ARPKD) is accurate? A) The use of ACE inhibitors will increase the vasopressin levels. B) ACE inhibitors may interrupt the renin-angiotensin-aldosterone system to reduce renal vasoconstriction. C) The ACE inhibitors have been shown to shrink the size of the cysts inside the kidneys. D) ACE inhibitors should be used strictly in those clients who also have an underlying cardiac history.

B. ACE inhibitors may interrupt the renin-angiotensin-aldosterone system to reduce renal vasoconstriction

An elderly client is experiencing pain and is concerned about how the pain will be managed. The nurse is aware that the first line of therapy for managing pain in the elderly would be: A) Allopurinol B) Acetaminophen C) Albuterol D) Amiodarone

B. Acetaminophen

The nurse is caring for a client with sickle cell disease. The most important treatment for the nurse to provide would be: A) Ferrous sulfate and a high-iron diet B) Aggressive intravenous hydration and morphine C) Transfusion of white blood cells D) Iron dextran and a high-carbohydrate diet

B. Aggressive intravenous hydration and morphine

The health care provider is caring for a client who has just been diagnosed with an osteosarcoma. Select the response that best describes the diagnosis. A) The least common malignant bone tumor B) Aggressive; malignant bone tumor C) Slow-growing; malignant bone tumor D) Benign bone tumor

B. Aggressive; malignant bone tumor

The physician suspects that a client with kidney stones has developed magnesium ammonium phosphate (struvite) stones based on which of the following urinalysis results? Select all that apply. A) Elevated uric acid levels B) Alkaline urine pH C) High urine phosphate level D) High bacterial count E) Presence of cystine particles

B. Alkaline urine pH C. High urine phosphate level D. High bacterial count

College students were given various amounts of alcohol within a specified timeframe and then asked to drive an obstacle course. The rationale for poor performance in driving as the amount of alcohol intake increased includes, "The blood-brain barrier: A) Allows more bilirubin to cross the barrier producing brain damage." B) Allows alcohol, a very lipid-soluble molecule to rapidly enter the brain." C) Excludes water-based compounds from crossing the brain with the exception of alcohol." D) Interacts negatively with the potassium-sodium pump, allowing alcohol to freely flow into the capillaries of the brain."

B. Allows alcohol, a very lipid-soluble molecule to rapidly enter the brain."

In contrast to the sympathetic nervous system, the functions of the parasympathetic nervous system include: A) Sweating B) Anabolism C) Pupil dilation D) Vasoconstriction

B. Anabolism

The nurse is teaching a client about appropriate interventions for back pain. Select all that apply. A) Muscle relaxants B) Analgesics C) Bed rest D) Aerobic exercise E) Nonsteroidal anti-inflammatory drugs

B. Analgesics E. Nonsteroidal anti-inflammatory drugs

A nurse is teaching a client newly diagnosed with a seizure disorder about medications. The most important information for the nurse to provide would be: A) Children can build up a tolerance to the medication quickly B) Antiepileptic medications should never be discontinued abruptly C) All anti epileptic medications should be taken with food D) Pregnant women should reduce the dose of medication they are taking

B. Antiepileptic medications should never be discontinued abruptly

A gymnastics student lands awkwardly and hurts her ankle. After MRI scan, it was revealed that she has a torn cartilage. The health care worker states it may take up to 3 to 4 months for this injury to heal. The basic physiologic reason behind the prolonged recovery is due to the fact that cartilage is/has primarily: A) The main secretor of an extracellular matrix B) Avascular C) Low tensile strength D) Lacking minerals

B. Avascular

A client with a history of diabetes presents to the emergency department following several days of polyuria and polydipsia with nausea/vomiting. On admission, the client labs show a blood glucose level of 480 mg/dL and bicarbonate level of 7.8 mEq/dL. The nurse suspects the client has diabetic ketoacidosis (DKA). The priority intervention should include: A) Limit fluid intake to only 250 mL/4 hours. B) Begin a loading dose of IV regular insulin followed by a continuous insulin infusion. C) Give at least 50 units of regular insulin IV stat and recheck blood glucose in 2 hours. D) Push a stat dose of bicarbonate followed by a double-dose (loading) of metformin.

B. Begin a loading dose of IV regular insulin followed by a continuous insulin infusion

A woman has cut her finger while dicing onions in the kitchen, causing her to drop her knife in pain. Which of the following components of this pain signal was transmitted by a third-order neuron? The neurons: A) Between the woman's finger and her spinal cord B) Between the thalamus and the cortex C) Between the CNS and the thalamus D) Of the efferent pathway that causes muscle contraction

B. Between the thalamus and the cortex

Which clinical manifestations would you expect to see in an infant diagnosed with autosomal recessive polycystic kidney disease (ARPKD)? A) Elevated systemic blood pressure and severe pain B) Ascending urinary tract infection and vomiting C) Gross hematuria and massive generalized edema D) Bilateral flank masses and impaired lung development

B. Bilateral flank masses and impaired lung development

A client was involved in an auto accident and suffered massive internal injuries that resulted in a large blood loss. Select the type of anemia the client is at greatest risk to develop. A) Aplastic B) Blood loss C) Hemolytic D) Iron deficiency

B. Blood loss

A client with confirmed low bone density asks the nurse if there is anything she can to decrease the risk fo trauma. The best response would be: A) High-impact aerobic exercise for 1 hour three times per week B) Brisk walking three times per week on a flat surface C) Lawn bowling for 1 hour per week D) Running 1 mil/day with good athletic shoes

B. Brisk walking three times per week on a flat surface

A soccer player has been diagnosed with a brain contusion after being injured in a game. The best explanation of the injury by the nurse would be: A) Tearing of brain tissue occurred B) Bruising on the surface of the brain occurred C) Transient neurogenic dysfunction caused by mechanical force to the brain occurred D) Hypoxia to the brain occurred

B. Bruising on the surface of the brain occurred

The nurse is caring for a spinal cord injury client. Assessment reveals shallow breath sounds with a very weak cough effort. The nurse correlates this with which level of injury on the spinal column? A) C2 B) C5 C) T1 D) T10

B. C5

While discussing hormonal control of bone formation, one should note that which hormone lowers blood calcium levels and decreases bone resorption? A) Vitamin D B) Calcitonin C) Prolactin D) Phosphate

B. Calcitonin

A client may be at risk for the development of hypoxia. Select the conditions that would place a client at risk. Select all that apply. A) Increased atmospheric pressure B) Carbon monoxide poisoning C) Increased oxygenation by the lungs D) Thrombocytopenia E) Age F) Severe anemia

B. Carbon monoxide poisoning C. Increased oxygenation by the lungs F. Severe anemia

The nurse is aware that hospitalized clients are at the greatest risk of developing septicemia as a result of: A) Asymptomatic bacteriuria B) Catheter-associated bacteriuria C) Acute viral cystitis D) Obstructive pyelonephritis

B. Catheter-associated bacteriuria

An elderly client has been brought to his primary care provider by his wife, who is concerned about his recent decrease in coordination. Upon assessment, his primary care provider notes that the client's gait is wide-based, unsteady, and lacking in fluidity, although his muscle tone appears normal. This client requires further assessment for which of the following health problems? A) Muscle atrophy B) Cerebellar disorders C) Impaired spinal reflexes D) Lower motor neuron lesions

B. Cerebellar disorders

The nurse is caring for a client who is being treated for HHS (hyperosmolar hyperglycemic state). The nurse is aware that the client should be assessed for: A) Polyuria B) Cerebral edema C) Dehydration D) Glycosuria

B. Cerebral edema

A client who has had a spinal injury now has sensory changes on the distal forearm and fourth and fifth fingers. The nurse can predict that this client has experienced an injury to the: Select all that apply. A) Cervical (C) 7 B) Cervical (C) 8 C) Thoracic (T) 1 dorsal root D) Thoracic (T) 2 dorsal root

B. Cervical (C) 8 C. Thoracic (T) 1 dorsal root

An increase in the level of RANKL would result in: A) Fusing of the epiphysis and metaphysis in long bones B) Changes in the rate of bone remodeling C) Decreased production of PTH D) Increased vitamin D activation

B. Changes in the rate of bone remodeling

An elderly client asks the nurse why so many older people develop anemia. The best response would be: A) Cor pulmonale B) Chronic disease C) Stress D) Increased erythropoiesis

B. Chronic disease

Polycythemia develops in clients with lung disease as a result of: A) Hyperventilation B) Chronic hypoxia C) Decreased blood viscosity D) Excessive respiratory fluid loss

B. Chronic hypoxia

In men experiencing nonrelaxing external sphincter with associated urine retention, the health care worker should assess for which of the following possible causes? A) Increased intra-abdominal pressure B) Chronic prostatitis C) Chronic stress response D) Pelvic inflammatory disease

B. Chronic prostatitis

Which factor has been implicated in the progression of diabetic nephropathy? A) Increased sodium intake B) Cigarette smoking C) Decreased blood pressure D) Elevated cholesterol

B. Cigarette smoking

A client with a history of chronic pyelonephritis has been admitted several times with recurrent bacterial infection of the urinary tract. The nurse should anticipate educating this client with regard to which common treatment regimen? A) Increase intake of cranberry juice to 2 L/day. B) Continue taking antibiotics for full 10 to 14 days even if symptoms of infection disappear. C) Force micturition every 2 hours while awake. D) Take prescribed diuretics early in the day to avoid having to get up during the night.

B. Continue taking antibiotics for full 10-14 days even if symptoms of infection disappear

Which physiologic change in the elderly population contributes to urinary incontinence? A) Side effects of medication B) Decline in detrusor muscle function C) Bladder capacity increases D) Increased bladder contractility

B. Decline in detrusor muscle function

A chronic kidney disease (CKD) client asks the nurse, "Why do I itch all the time?" The nurse bases her response on which of the following integumentary physiologic factors that causes pruritus? Select all that apply. A) Too harsh of soap while bathing B) Decrease in perspiration C) Limited sodium intake D) Enlarged size of sweat glands E) Elevated serum phosphate levels

B. Decrease in perspiration E. Elevated serum phosphate levels

What is the effect of von Willebrand disease on the platelets? A) Increased platelet adhesion B) Decreased platelet adhesion C) Increased platelet formation D) Decreased platelet formation

B. Decreased platelet adhesion

The nurse is preparing to assess a client who has just been admitted to the hospital with a diagnosis of prerenal failure. Which would the nurse expect the client to manifest? Select all that apply. A) Increased urinary output B) Decreased urinary output C) BUN to serum creatinine ration of 10:1 D) BUN-to-serum creatinine ratio of greater than 20:1 E) Decreased BUN F) Increased BUN

B. Decreased urinary output D. BUN-to-serum creatinine ratio of greater than 20:1 F. Increased BUN

Select the statement that best describes the pathophysiology of Parkinson disease. A) Acetylcholine levels rise causing an inhibition of voluntary movement B) Degeneration of the nigrostriatal dopamine system occurs. C) Failure of the cerebral cortex, which does not allow the use of acetylcholine D) Increase in the number of dopamine receptors by an alternate basal ganglion

B. Degeneration of the nigrostriatal dopamine system occurs.

Manifestations of childhood renal disease are varied and may differ from adult- onset renal failure. A school-aged child with chronic kidney disease may exhibit: A) Low IQ level with borderline retardation B) Developmental delays such as uncoordinated gait and minimal fine motor skills C) Inability to control bladder, resulting in incontinence D) Frequent, uncontrolled rolling of the tongue and opening mouth extremely wide

B. Developmental delays such as uncoordinated gait and minimal fine motor skills

After numerous trips to the physician's office, a client is diagnosed with diffuse scleroderma based on which of the following clinical manifestations? Select all that apply. A) Protruding eyeball with very red, inflamed eyes and associated photophobia. B) Difficulty swallowing resulting in weight loss due to malabsorption. C) Spider veins on the face and chest. D) Hands turn bluish purple when getting items out of the freezer. E) Inability to walk long distances with severe leg cramping in calf muscles.

B. Difficulty swallowing resulting in weight loss due to malabsorption. C. Spider veins on the face and chest. D. Hands turn bluish purple when getting items out of the freezer.

A client is beginning to recover from acute tubular necrosis. The nurse would likely be assessing which of the following manifestations of the recovery phase of ATN? A) Edema B) Diuresis C) Proteinuria D) Hypokalemia

B. Diuresis

Stretch-sensitive receptors in the skin (Ruffini end organs, Pacinian corpuscles, and Merkel cells) help signal postural information and are processed through the: A) Third-order neurons B) Dorsal column-medial lemniscus pathway C) Anterolateral pathway D) Posterior column of the spinal cord

B. Dorsal column-medial lemniscus pathway

Which of the following principles should underlie the pain control strategy in the care of a child with a diagnosis of cancer? A) Opioids should be avoided in order to prevent liver and kidney insult. B) Dosing and timing should aim for a steady serum level of the prescribed drug. C) Doses of analgesia should be given only when the client's pain becomes severe. D) Drugs from numerous classifications should be used to maximize pain control.

B. Dosing and timing should aim for a steady serum level of the prescribed drug

A client is admitted to the hospital with a diagnosis of deep vein thrombosis and started on intravenous heparin therapy. Seven days later, the client's lab values identify a rapid decrease in platelets. The health care provider recognizes this as: A) Thrombotic thrombocytopenia purpura (TTP) B) Drug-induced thrombocytopenia C) Hemolytic-uremic syndrome (HUS) D) Immune thrombocytopenic purpura (ITP)

B. Drug-induced thrombocytopenia

A client with a suspected diagnosis of primary hypothyroidism would most likely demonstrate which of the following serum laboratory values A) Decreased thyroid-stimulating hormone (TSH) related to poor thyroid function B) Elevated thyroid-stimulating hormone (TSH) and decreased thyroxine (T4) C) Elevated thyroid-stimulating hormone (TSH), thyroxine (T4) and T3 D) Decreased thyroid-stimulating hormone (TSH) and thyroxine (T4) and low T3

B. Elevated thyroid-stimulating hormone (TSH) and decreased thyroxine (T4)

The CT scan report identified that a client with a skull fracture has developed a hematoma that resulted from a tour artery. The report would be interpreted as: A) Chronic subdural hematoma B) Epidural hematoma C) Intracranial hematoma D) Subdural hematoma

B. Epidural hematoma

Which of the following pathophysiologic phenomena may result in a diagnosis of Cushing disease? A) Hypopituitarism B) Excess ACTH production by a pituitary tumor C) Autoimmune destruction of the adrenal cortex D) Malfunction of the HPA system

B. Excess ACTH production by a pituitary tumor

A 6-month-old infant was born with a murmur. The pediatrician is recommending valve replacement/repair surgery based on which of the following lab values indicating intravascular hemolysis? A) Increase in the number of red blood cells (polycythemia) B) Excess of hemoglobin in the blood plasma (hemoglobinemia) C) Elevated bilirubin levels D) Decreased number of reticulocytes

B. Excess of hemoglobin in the blood plasma (hemoglobinemia)

Which urinary structure helps to stop micturition when it is occurring and maintains continence under high bladder pressure? A) Internal urethra B) External sphincter C) Detrusor muscle D) Urinary vesicle

B. External sphincter

The pathophysiologic effects of spina bifida are due to: A) Malformation of the brain and spinal column causing spinal fluid to pool at the base of the spine B) Failure of one or more neural arches to close within the vertebral column of the neural tube C) Cystic lesions growing into the dorsal root ganglia D) Hypertrophy of the primary vesicles, which eventually causes the excess fluid to bulge in weakened structures

B. Failure of one or more neural arches to close within the vertebral column of the neural tube

A client who was involved in an automobile accident experienced major trauma and sustained a fractured femur. The nurse notices the following changes 4 hours after admission to the critical care unit: Disorientation Dyspnea Tachycardia Low-grade temperature Diaphoresis Substernal chest pain The nurse suspects: A) Sympathetic nervous system response B) Fat embolism syndrome (FES) C) Parasympathetic nervous system response D) Autonomic hyperreflexia

B. Fat embolism syndrome (FES)

A client who is diagnosed with seizures describes feeling confused after experiencing a seizure. The family members report that the client has been smacking his lips prior to having a seizure. The client most likely experienced which type of seizure? A) Atonic B) Focal C) Clonic D) Myoclonic

B. Focal

The health care provider has ordered the administration of a hypertonic intravenous (IV) solution for a client. The nurse anticipates that the IV fluid will cause water to shift how? A) From the transcellular to intracellular space B) From the intracellular to intravascular space C) From the intravascular to interstitial space D) From the interstitial to transcellular space

B. From the intracellular to intravascular space

A client with significant burns on his lower body has developed sepsis on the 3rd day following his accident.Which of the following manifestations would the nurse anticipate for an ischemic acute tubular necrosis rather than prerenal failure? The client: A) Exhibits pulmonary and peripheral edema B) GFR does not increase after restoration of renal blood flow C) Undergoes emergent hemodialysis that does not result in decreased BUN and creatinine D) Exhibits oliguria and frank hematuria

B. GFR does not increase after restoration of renal blood flow

The health care provider is reviewing lab results of a client. Select the test that is the best measurement of overall kidney function? A) Serum creating levels B) Glomerular filtration rate (GFR) C) Urine albumin levels D) Blood urea nitrogen (BUN)

B. Glomerular filtration rate (GFR)

A 25-year-old female client exhibits exophthalmos of both eyes. The health care provider recognizes this as a manifestation of: A) Acquired hypothyroidism B) Graves disease C) Hashimoto thyroiditis D) Myxedema

B. Graves disease

Hyaline cartilage is a firm but flexible type of connective tissue that is essential for: A) Calcium salt storage B) Growth of long bones C) Bone surface perfusion D) Reduced friction on tendons

B. Growth of long bones

The nurse is assessing a client with hyperkalemia. The nurse is aware that the organ at most risk for this client would be the: A) Lungs B) Heart C) Liver D) Brain

B. Heart

Which of the following client clinical manifestations most clearly suggests a need for diagnostic testing to rule out renal cell carcinoma? A) Urinary urgency B) Hematuria C) Oliguria D) Cloudy urine

B. Hematuria

Diabetic retinopathy, the leading cause of acquired blindness in the United States, is characterized by retinal: A) Glaucoma B) Hemorrhages C) Dehydration D) Infections

B. Hemorrhages

Which factor contributes to the development of polycystic kidney disease? A) A reduction in prerenal blood flow to the kidneys B) Hereditary mutations in polycystic I and II C) Enlargement in the basement membrane of the kidney D) Multiple recurrent urinary tract infections

B. Hereditary mutations in polycystic I and II

The nurse is teaching a client diagnosed with Addison disease about the importance of lifetime oral replacement therapy. Select the pharmacologic agent that would be prescribed. A) Insulin B) Hydrocortisone C) Ketoconazole D) Potassium supplements

B. Hydrocortisone

The nurse is caring for a client diagnosed with hyperthyroidism. Which of the following signs and symptoms are most likely to be seen in this client? Select all that apply. A) Anemia B) Hypertension C) Bradycardia D) Weight gain E) Diarrhea F) Tremor G) Constipation H) Weight loss

B. Hypertension E. Diarrhea F. Tremor H. Weight loss

While working on the med-surg floor, the nurse has a client who is experiencing an insulin reaction. The client is conscious and can follow directions. The most appropriate intervention would be: A) Call the physician and wait for him or her to respond to give you orders of what he or she prefers you do for this client. B) Immediately administer 15 g of glucose (preferably via oral route if the client is alert enough to swallow) and wait for 15 minutes. Then repeat this if necessary. C) Start pushing 50% glucose solution slowly and do not stop pushing until the client's repeat blood glucose level is above 100 mg/dL. D) Skip the oral glucose tablets and go directly to giving intramuscular glucagon. Repeat the glucagon in 15 minutes if the blood glucose level is not within a normal range.

B. Immediately administer 15 g of glucose (preferably via oral route if the client is alert enough to swallow) and wait for 15 minutes. Then repeat this if necessary

An elderly resident of an assisted-living facility has had his mobility and independence significantly impaired by the progression of his rheumatoid arthritis (RA). What is the primary pathophysiologic process that has contributed to this client's decline in health? A) A mismatch between bone resorption and remodeling B) Immunologically mediated joint inflammation C) Excessive collagen production and deposition D) Cytokine release following mechanical joint injury

B. Immunologically mediated joint inflammation

A client has developed global ischemia of the brain. The nurse determines this is: A) Inadequate perfusion to the dominant side of the brain B) Inadequate to meet the metabolic needs of the entire brain C) Inadequate perfusion to the right side of the brain D)Inadequate perfusion fo the non dominant side of the brain

B. Inadequate to meet the metabolic needs of the entire brain

While lecturing to a group of physiology students, the instructor asks, "What metabolic factors cause vasodilation of cerebral vessels thereby increasing cerebral blood flow to the brain?" The student with the best response would be: A) Increased oxygen saturation B) Increased carbon dioxide level C) Decreased serum sodium level D) Decreased hydrogen ion concentration

B. Increased carbon dioxide level

Hemolytic anemia is characterized by excessive red blood cell destruction and compensatory: A) Hypoactive bone marrow B) Increased erythropoiesis C) Iron retention in the body D) Shrinkage of the spleen

B. Increased erythropoiesis

A 55-year-old male client has reported joint pain in his feet. Which of the following blood work results should prompt further testing to rule out primary gout? A) Increased C-reactive protein (CRP) B) Increased serum uric acid C) Increased polymorphonuclear leukocytes D) Increased serum cortisol

B. Increased serum uric acid

A 68-year-old client with an 80 pack/year history of smoking was diagnosed with emphysema 18 months ago. The client's most recent scheduled blood work showed excessive increase in production of red blood cell (erythrocytes), a problem that suggests the need for which of the following interventions? A) Vitamin B 12 supplements B) Increased supplementary oxygen therapy C) Hemodialysis or peritoneal dialysis D) Scheduled erythropoietin injections

B. Increased supplementary oxygen therapy

A client has been diagnosed with metabolic syndrome and growth hormone (GH) deficiency. Physical assessment of the client would identify: A) Increased bone mineral density B) Increased visceral fat C) Enhanced insulin uptake D) Increase in lean body mass

B. Increased visceral fat

The nurse is teaching a client diagnosed with osteomalacia about treatments to improve the condition. The best information for the nurse to provide would be: A) Gaining weight of 5 to 10 pounds B) Increasing dietary consumption of vitamin D C) Decreased exposure to the natural UV radiation of the sun D) A bone marrow transplant

B. Increasing dietary consumption of vitamin D

A client is brought to the emergency department semicomatose and a blood glucose reading of 673. He is diagnosed with diabetic ketoacidosis (DKA). Blood gas results are as follows: serum pH 7.29 and HCO 3 - level 19 mEq/dL; PCO 2 level 32 mm Hg. The nurse should anticipate that which of the following orders may correct this diabetic ketosis? A) Administration of potassium chloride B) Initiating an insulin IV infusion along with fluid replacement C) Administering supplemental oxygen and rebreathing from a paper bag D) Instituting a cough and deep breathing schedule for every hour while awake to improve ventilation

B. Initiating an insulin IV infusion along with fluid replacement

A client has been admitted for immune thrombocytopenic purpura. The client has not responded to corticosteroid treatment. The priority nursing intervention for this client would include which of the following treatment measures? A) Place the client in isolation, so the skin rashes will not spread to other clients. B) Insert an intravenous catheter, so immune globulin can be administered in a timely manner. C) Insert a Foley catheter to monitor hourly urine output. D) Prepare a surgical permit for an emergency splenectomy.

B. Insert an intravenous catheter, so immune globulin can be administered in a timely manner

The nurse is assessing a client diagnosed with anemia and notes that the client's skin and mucous membranes are pale. The nurse interprets this as: A) Presence of systolic murmur B) Insufficient hemoglobin C) Tissue hypoxia to the brain D) Changes in blood viscosity

B. Insufficient hemoglobin

A basketball player fell awkwardly when attempting to claim a rebound, a mishap that resulted in a tear to the anterior cruciate ligament (ACL) of his left knee. The school nurses will anticipate the player to exhibit which of the following clinical manifestations? Select all that apply. A) Immoveable (locked) knee joint B) Intense pain C) Abscess formation D) Edema (swelling) of the knee E) Large hematoma on the anterior knee surface

B. Intense pain D. Edema (swelling) of the knee

Which of the following insulin administration regimens is most likely to result in stable blood glucose levels for a client with a diagnosis of type 1 diabetes? A) One large dose of long-acting insulin each day before breakfast B) Intermediate-acting insulin at 8:00 AM and 8:00 PM with rapid-acting insulin before each meal C) Six to eight small doses of rapid-acting insulin each day, with capillary monitoring before each D) Long-acting insulin twice daily (breakfast and bedtime), with intermediate-acting insulin in the afternoon

B. Intermediate-acting insulin at 8:00 AM and 8:00 PM with rapid-acting insulin before each meal

Which of the following signs and symptoms should prompt a 29-year-old woman's primary care provider to assess for systemic lupus erythematosus (SLE)? A) Chronic nausea and vomiting that is unresponsive to antiemetics B) Joint pain and increased creatinine and blood urea nitrogen C) A history of thromboembolic events and varicose veins D) Dysmenorrhea and recent spontaneous abortion

B. Joint pain and increased creatine and blood urea nitrogen

The health care provider is assessing a child and suspects an inflammatory myopathy. The assessment data include muscle weakness and a characteristic rash, primarily involving the skin and muscle. The health care provider would document this as: A) Juvenile idiopathic arthritis (JIA) B) Juvenile dermatomyositis (JDMS) C) Ankylosing spondylitis D) Systemic lupus erythematous (SLE)

B. Juvenile dermatomyositis (JDMS)

A client is admitted to the emergency department and diagnosed with diabetic keto acidosis(DKA). The client would most likely manifest: A) Respiratory alkalosis B) Ketosis C) Hypervolemia D) Hypoglycemia

B. Ketosis

A client with excessive production of growth hormone level will likely exhibit which clinical manifestations? Select all that apply. A) Short stature with obesity B) Large hands and feet due to increased production of GH C) Excess thirst and urination due to decreased glucose uptake D) Difficulty chewing food E) Tendency to develop asthma

B. Large hands and feet due to increased production of GH C. Excess thirst and urination due to decreased glucose uptake D. Difficulty chewing food

A client with acute tubular necrosis (ATN) exhibits oliguria and edema, and the laboratory results reveal increased levels of urea, potassium, and creatinine. Based on symptoms and lab data, which phase of ATN is this client most likely experiencing? A) Initiation phase B) Maintenance phase C) Recovery phase D) Reparative phase

B. Maintenance phase

Megaloblastic anemias caused by folic acid or vitamin B 12 deficiencies can seriously affect RBC production. Which of the following lab results would correlate with this diagnosis? A) Iron level of 70 μg/dL (normal) B) Mean corpuscular hemoglobin (MCV) 120 fL (high) C) Platelet count 200,000 (normal) D) Reticulocyte count 3.6% (high)

B. Mean corpuscular hemoglobin (MCV) 120 fL (high)

The nurse is reviewing the client's laboratory data showing an increased mean corpuscular volume (MCV) and a normal mean corpuscular hemoglobin concentration (MCHC). The client most likely has which of the following types of anemia? A) Sickle cell anemia B) Megaloblastic anemia C) Iron deficiency anemia D) Hemolytic anemia

B. Megaloblastic anemia

Select the tactile receptors that are sensitive to the movement of very light objects over the surface of the skin. A) Ruffini end organs B) Meissner corpuscles C) Merkel disks D) Free nerve endings

B. Meissner corpuscles

A client sustained a spinal cord injury and is immediately brought to the emergency department. The client asks the physician if there is any medication he can get to possibly help him, regardless of risks. The physician assesses the client and determines the most important medication to administer would be: A) Naloxone B) Methylprednisone C) Tirilazad D) IV opioids

B. Methylprednisone

Reduced glomerular filtration rate (GFR), with a serum creatinine level that remains in the normal range, is associated with aging because elderly persons tend to have reduced: A) Calcium intake B) Muscle mass C) Drug tolerance D) Renal perfusion

B. Muscle mass

The nurse is assessing a client for early manifestations of hyponatremia. The nurse would assess the client for: A) Dry, sticky mucous membranes B) Muscle weakness C) Tachycardia D) Peaked T wave on the EKG

B. Muscle weakness

A client has been brought to the emergency department following an overdose of insulin that resulted in unconsciousness. When explaining the rationale for this to the family, the nurse will emphasize that neurons: A) Store glycogen within the brain cavity B) Must rely on glucose from the blood to meet their energy needs C) Require many amino acids in order to produce enough energy to function properly D) Can cause the liver to convert triglycerides into energy if needed quickly

B. Must rely on glucose from the blood to meet their energy needs

The nurse determines that teaching was effective when a client who has been treated for Chlamydia states the importance of: A) Retesting in 3 to 4 months B) No follow-up is needed C) Repeating a Pap smear D) Repeating antibiotics for 1 week

B. No follow-up is needed

Nociceptors are sensory receptors that are activated by: A) Cortisol B) Noxious stimuli C) Pressure and touch D) Sudden movements

B. Noxious stimuli

A child has been diagnosed with classic growth hormone deficiency. The child may experience: Select all that apply. A) Early puberty B) Obesity C) Short stature D) Delayed skeletal muscle maturation E) Below normal intelligence F) Mature facial features

B. Obesity C. Short stature D. Delayed skeletal muscle maturation

A pediatric client has been diagnosed with juvenile idiopathic arthritis (JIA). The condition has predominantly affected both the ankle and knee joints of the lower extremities. The nurse recognizes this as: A) Systemic-onset disease B) Oligoarticular disorder C) Polyarticular disorder D) Blount disease

B. Oligoatricular disorder

Select the correct statement regarding Herpes simplex virus. A) Herpes simplex virus cannot be transmitted if lesions are not present B) Only two strains of the virus are considered sexually transmittable C) The virus is not contagious D) The virus will not reactivate

B. Only two strains of the virus are considered sexually transmittable

In the ICU, a postsurgical client has developed sepsis and is being treated with multiple medications. During the mid-morning assessment, which finding leads the nurse to suspect the client may be developing a complication called disseminated intravascular coagulation (DIC)? Select all that apply. A) Headaches associated with light sensitivity B) Oozing from all previous puncture and intravenous sites C) Decreased O 2 saturation and diminished breath sounds in lower lobes D) Hemorrhage from the surgical site requiring deep pressure dressings E) Urine from the Foley catheter is bloody

B. Oozing from all previous puncture and intravenous sites D. Hemorrhage from the surgical site requiring deep pressure dressings

The health care provider reviews the following assessment data of a client: Pain in the front of the knee that is associated with inflammation and thickening of the patellar tendon Pain during specific activities such as kneeling, running, bicycle riding, or stair climbing Swelling, tenderness, and increased prominence of the tibial tubercle Based on these findings, the provider determines that the most likely diagnosis would be: A) Slipped capital femoral epiphysis B) Osgood-Schlatter disease C) Legg-Calve-Perhtes disease D) Blount disease

B. Osgood-Schlatter disease

Water movement from the side of the membrane having a lesser number of particles and greater concentration of water to the side having a greater number of particles and lesser concentration of water is termed: A) Active transport B) Osmosis C) Filtration D) Diffusion

B. Osmosis

An elderly client who experiences chronic pain takes opioid analgesics on a regular basis, a practice that has resulted in frequent constipation and occasional bowel obstructions. Which of the following problems may directly result from these gastrointestinal disorders? A) Urinary tract infections B) Overflow urinary incontinence C) Bladder cancer D) Neurogenic bladder

B. Overflow urinary incontinence

A client who has experienced a spinal cord injury still has use of the arms and has impaired motor and sensory function of the trunk, legs, and pelvic organs. The injury would be classified as: A) Brown-Sequard syndrome B) Paraplegia C) Tetraplegia D) Quadriplegia

B. Paralegia

The perception of "where" a stimulus is in space and in relation to body parts is a function of the: A) Occipital lobe B) Parietal lobe C) Hypothalamus D) Prefrontal cortex

B. Parietal lobe

Common manifestations of acute meningococcal meningitis, a highly contagious and lethal form of meningitis, include: A) Diplopia B) Petechiae C) Papilledema D) Focal paralysis

B. Petechiae

A client with thrombocytopenia is most likely to exhibit: A) Swollen, painful calf B) Pinpoint hemorrhages (petechiae) on the legs C) Platelet level of 450,000/ul D) Ecchymosis of the abdomen measuring 7 cm across

B. Pinpoint hemorrhages (petechiae) on the legs

A client's recent computed tomography (CT) scan has revealed the presence of hydrocephalus. Which of the following treatment measures is most likely to resolve this health problem? A) Aggressive diuresis B) Placement of a shunt C) Administration of hypertonic intravenous solution D) Lumbar puncture

B. Placement of a shunt

Magnetic resonance imaging of a client's knee has revealed the presence of bursitis. The nurse should anticipate performing which intervention for bursitis? A) Applying Buck's traction with 10-pound weights B) Placing an ice pack on the knee to decrease swelling C) Administering an antihistamine like Benadryl to minimize inflammation D) Obtaining a surgical permit to repair the bursae

B. Placing an ice pack on the knee to decrease swelling

Select the most important treatment for thrombotic thrombocytopenic purpura (TTP). A) Transfusion with factor VIII B) Plasmapheresis C) Heparin infusion D) Intravenous administration of vitamin C

B. Plasmapheresis

In light of the presence of numerous risk factors for coronary artery disease, a client's primary care provider has recommended that he take low-dose (81 mg) aspirin once daily. Doing so will reduce the client's risk of myocardial infarction by altering which of the following stages of hemostasis? A) Vessel spasm B) Platelet plug formation C) Blood coagulation D) Clot lysis

B. Platelet plug formation

A client has been experiencing elevated blood glucose levels. The nurse anticipates that the client assessment data would include: A) Hypertension B) Polydipsia C) Moist mucous membranes D) Fasting blood sugar (FBS) 120 mg/dL

B. Polydipsia

A 43-year-old female has recently been diagnosed with systemic lupus erythematosus (SLE) glomerulonephritis. She has presented to the out-client department to have a renal biopsy. Knowing the usual treatment options, the nurse should anticipate educating the client (who has a positive biopsy result) on which of the following medications being prescribed? Select all that apply. A) Lasix, a diuretic B) Prednisone, a corticosteroid C) Captopril, an ACE inhibitor D) Ampicillin, an antibiotic

B. Prednisone, a corticosteroid C. Captorpril, an ACE inhibitor

A client complains of general malaise and fatigue and has a mild fever. The nurse would evaluate this stage of disease as the: A) Incubation stage B) Prodromal stage C) Acute stage D) Convalescent stage

B. Prodromal stage

A client with laryngeal dystonia has gotten to the point that people on the telephone cannot understand her. She has heard about getting Botox injections into her vocal cords. The nurse will teach about the actions of Botox. Which is the most accurate description? This drug: A) Will slow the decline in muscle strength and function B) Produces paralysis of the larynx muscles by blocking acetylcholine release C) Prevents the depolarizing effect of the neurotransmitters D) Inhibits the peripheral metabolism of dopamine

B. Produces paralysis of the larynx muscles by blocking acetylcholine release

Which of the following physiologic processes is a direct effect of the release of growth hormone by the anterior pituitary? A) Development of cartilage and bone B) Production of insulin-like growth factors (IGFs) by the liver C) Increase in overall metabolic rate and cardiovascular function D) Positive feedback of the hypothalamic-pituitary-thyroid feedback system

B. Production of insulin-like growth factors (IGFs) by the liver

A client has been admitted for deterioration of her renal function due to chronic renal failure. Her admission K + level is 7.8 mEq/L. The nurse would expect to see which of the following abnormalities on her telemetry (ECG) strip? Select all that apply. A) Tachycardia (fast rate) with frequent early ventricular beats (PVCs) B) Prolonged PR interval with widening of the QRS complex C) Ventricular fibrillation D) Atrial flutter with a 2:1 conduction ratio

B. Prolonged PR interval with widening of the QRS complex C. Ventricular fibrillation

A hospital client with a diagnosis of type 1 diabetes has been administered a scheduled dose of regular insulin. Which of the following effects will result from the action of insulin? A) Promotion of fat breakdown B) Promotion of glucose uptake by target cells C) Promotion of gluconeogenesis D) Initiation of glycogenolysis

B. Promotion of glucose uptake by target cells

While taking an exam on disorders of hemostasis, the students were asked to identify endothelial mediators that inhibit platelet aggregation and also vasodilate vessels. Which of the following would be considered a correct answer? Select all that apply. A) Adenosine diphosphate B) Prostaglandin I 2 C) Thromboxane A 2 (TXA 2 ) D) Nitric oxide E) Plasminogen

B. Prostoglandin I2 D. Nitric oxide

Which of the following assessment findings would lead the nurse to suspect the client has nephrotic syndrome? A) Hematuria and anemia B) Proteinuria and generalized edema C) Renal colic and increased serum sodium D) Increased creatinine with normal blood urea nitrogen

B. Proteinuria and generalized edema

A client is experiencing deep somatic pain. The client would manifest: Select all that apply. A) Pain that occurs superficially B) Radiation of pain C) Sharp pain with a burning quality and may be abrupt or slow in onset D) Localized pain E) Pain reproduced by stimuli

B. Radiation of pain E. Pain reproduced by stimuli

The nurse is assessing a client who has a history of untreated chlamydial infection. The nurse is aware that the client is at risk for: A) HIV encephalopathy B) Reiter syndrome C) Cervical cancer D) Early-onset dementia

B. Reiter syndrome

While taking a client history, which of the following assessments lead the nurse to suspect the client may have polycystic kidney disease? Select all that apply. A) Massive proteinuria on dipstick urine specimen B) Renal colic with flank pain C) Bright red blood in urine sample D) Elevated blood pressure of 180/94 E) Shortness of breath (SOB) with loud rhonchi and wheezes heard on auscultation

B. Renal colic with flank pain C. Bright red blood in urine sample D. Elevated blood pressure of 180-94

A client with CKD is being treated for hyperphosphatemia and hypocalcemia. Select the most appropriate intervention. Select all that apply. A) Oral phosphate supplements B) Restriction of foods high in phosphate C) Increased daily consumption of milk D) Phosphate-binding antacids E) Activated vitamin D

B. Restriction of foods high in phosphate D. Phosphate-binding antacids E. Activated vitamin D

Cerebellar tremor is exhibited by which symptom? A) Slow, slurred speech of continuous, varying loudness B) Rhythmic movement of the finger or toe that worsens as a target is approached C) Constant conjugate readjustment of eye position D) Over- or under reaching for a target, followed by overcorrection

B. Rhythmic movement of the finger or toe that worsens as a target is approached

The nurse is assessing a client and notes the client is now displaying decerebrate posturing. The position would be documented as: A) Active range of motion with increased strength in the upper extremities when painful stimulation applied B) Rigidity of the arms with palm of the hands turned away from the body and with stiffly extended legs and plantar flexion of the feet C) Flexion of the arms, wrists, and fingers, with abduction of the upper extremities, internal rotation, and plantar flexion of the lower extremities D) Prone position with arms placed above the head and legs elevated, deep tendon reflexes showing hyperreflexia

B. Rigidity of the arms with palms of the hands turned away from the body and with stiffly extended legs and plantar flexion of the feet

The health care provider is obtaining a pain history from a client. The most important information to obtain would be: Select all that apply A) Gender B) Self-report C) Self image D) Pain onset E) Intensity F) Localization G) Quality

B. Self-report D. Pain onset E. Intensity F. Localization G. Quality

The health care provider is explaining the course of treatment to the parents of a 2-day-old infant born with clubfeet. The most appropriate information to provide would be: A) Immediate corrective surgery B) Serial manipulations and casting of the affected extremity C) Chemotherapy D) Bilateral hip spica cast

B. Serial manipulations and casting of the affected extremities

A client is experiencing an episode of gout. The nurse recognizes that the condition is a result of: A) Serum calcitonin levels B) Serum uric acid levels C) Serum blood urea nitrogen (BUN) D) Serum calcium levels

B. Serum uric acid levels

A pregnant client asks the nurse if she is at risk for developing gestational diabetes. The nurse reviews the risk factors with the client. The most important information for the nurse to provide would be: Select all that apply. A) Family history of hypertension B) Severe obesity C) Glycosuria D) Polycystic ovary disease E) Previous low birth weight baby

B. Severe obesity C. Glycosuria D. Polycystic ovary disease

The most recent blood work of a client with a diagnosis of acute myelogenous leukemia (AML) reveals thrombocytopenia. Where is the client most likely to experience abnormal bleeding as a result of low platelets? A) Inside the brain B) Skin and mucous membranes C) Sclerae of the eyes D) Nephrons and ureters

B. Skin and mucous membranes

A nurse caring for a client with a diagnosis of diabetes insipidus (DI) should prioritize the close monitoring of which of the following electrolyte levels? A) Potassium B) Sodium C) Magnesium D) Calcium

B. Sodium

The nurse is assessing a female client with a diagnosis of primary adrenal cortical insufficiency. The nurse anticipates the client will manifest: A) Weight gain and fluid retention B) Sparse axillary and pubic hair C) Anorexia and increased excitability D) Pale skin and increased hunger

B. Sparse axillary and pubic hair

The health care provider is assessing a client with a history of ankylosing spondylitis, to note progression of the disease. The most important area for the provider to assess would be: A) Ankles B) Spine C) All joints (polyarticular) D) Elbows

B. Spine

A physiology instructor asks the class, "What role do osteoblasts play in the physiology of bone tissue?" The student with the best answer would be: A) Differentiation into mature bone cells B) Synthesis and secretion of the bone matrix C) Maintenance of calcium balance D) Resorption of the bone matrix

B. Synthesis and secretion of the bone matrix

A client was involved in an auto accident and is brought to the hospital ambulance. The admitting diagnosis is right hip dislocation. Select the most appropriate intervention for this client: A) The fracture will be reduced in 12 hours, and the client can immediately apply weight to the extremity B) The client will need emergency surgery intervention to reduce the fracture and prevent complications C) The client can be discharged and make an appointment in a few days to see her primary care doctor D) The client will be allowed to walk on the extremity for a few days before any treatment will be done

B. The client will need emergency intervention to reduce the fracture and prevent complications

Which of the following events would suggest that an individual's physiologic response to an obstruction has progressed beyond the compensatory stage and is now in the decompensatory stage? A) The bladder muscle hypertrophies. B) The detrusor loses its power of contraction. C) The ability to suppress urination is diminished. D) The individual experiences urgency.

B. The detrusor loses its power of contraction

The health care provider is assessing the functional integrity of all spinal nerves utilizing a pinpoint pressed against the skin. A normal response would be interpreted as: A) No response when the pin is pressed B) The withdrawal reflex is activated C) Verifying the intensity of the pin's force of compression D) Saying "Ouch" when the pin is pressed

B. The withdrawal reflex is activated

The nurse is preparing to administer medication to a client. Select the medication that the nurse would administer to prevent platelet aggregation. A) Vitamin K B) Ticlopidine (Ticlid) C) Warfarin D) Tirofiban

B. Ticlopidine (Ticlid)

The health care provider is assessing a client for carpal tunnel syndrome. The health care provider performs light percussion over the median nerve at the wrist. This assessment is known as: A) Chvostek sign B) Tinel sign C) Trousseau sign D) Phalen maneuver

B. Tinel sign

A client newly diagnosed with rheumatoid arthritis is crying and tells the nurse she does not know how to manage the disease. Select the most important information for the nurse to provide. Select all that apply. A) Refraining from medication B) Treatment goals C) Posture D) Physical rest E) Education F) Therapeutic use of heat and cold G) Increasing impact exercises

B. Treatment goals C. Posture D. Physical rest E. Education F. Therapeutic use of heat and cold

A client diagnosed with autosomal dominant von Willebrand factor disease (vWF) is experiencing mild to moderate bleeding. The health care provider would classify the diagnosis as: A) Type 1 B) Type 2 C) Type 4 D) Type 3

B. Type 2

While playing outside in the snow, a young child complained of painful fingertips since he would not keep his gloves on. In the emergency department, the nurse knows this painful sensation is a result of which transmission of proprioceptive somatosensory information? A) Reflexive networks B) Type C dorsal root ganglion neurons C) Anterolateral pathway D) Myelinated type B trigeminal sensory neurons

B. Type C dorsal root ganglion neurons

The body compensates for obstructed urine outflow up to a certain point. Which of the following signs/symptoms lead the nurse to suspect decompensatory changes are occurring? Select all that apply. A) Reports of renal colic B) Urinary frequency noted C) High residual volume up to 1000 mL D) Postvoid residual volume less than 50 mL E) Must strain to initiate the stream of urine

B. Urinary frequency noted C. High residual volume up to 1000 mL E. Must strain to initiate the stream of urine

The nurse has just completed an assessment on a client admitted with Guillain-Barre syndrome. The nurse determines that a priority of care will be: A) Emotional support B) Ventilatory assessment and support C) Feeding the client D) Administration of antibiotics

B. Ventilatory assessment and support

The nurse is caring for a client who is a strict vegetarian; the client is at greatest risk for the development of: A) Folic acid deficiency anemia B) Vitamin B12 deficiency anemia C) Microcytic anemia D) Blood loss anemia

B. Vitamin B12 deficiency anemia

A client diagnosed with schizophrenia has been admitted to the emergency department (ED) after ingesting more than 2 gallons of water in one sitting. Which of the following pathophysiologic processes may result from the sudden water gain? A) Hypernatremia B) Water movement from the extracellular to the intracellular compartment C) Syndrome of inappropriate secretion of ADH (SIADH) D) Isotonic fluid excess in the extracellular fluid compartment

B. Water movement from the extracellular to the intracellular compartment

Prior to undergoing diagnostic testing with contrast, it is recommended that older adult clients have their creatinine level checked. The rationale for this is to ensure the client: A) Is not allergic to shell fish or iodine B) Will not undergo an acute kidney injury by decreasing renal blood flow C) Does not have a kidney stone obstructing the urethra D) Is in good enough health to withstand a walking on a treadmill

B. Will not undergo an acute kidney injury by decreasing renal blood flow

A female teenager has experienced three uncomplicated urinary tract infections in the last 3 months. Knowing the anatomical location of the urethra, the nurse should educate this teenager about: A) Proper handwashing to decrease amount of Pseudomonas growing on the hands B) Wiping from front to back to prevent Escherichia coli contamination of the urethra C) Wearing gloves when wiping perineum after defecation to prevent Staphylococcus aureus infection D) Washing hands prior to inserting a tampon to minimize the risk of group B Streptococcus

B. Wiping from front to back to prevent Escherichia coli contamination of the urethra

A client with trigeminal neuralgia usually complains of excruciating pain. Which of the following activities may trigger an acute pain attack? Select all that may apply. A) Brushing the teeth with lukewarm water B) Working in the office that has an air duct located directly overhead C) One's significant other lightly stroking their face D) Applying lipstick E) Walking outside on a windy day

B. Working in the office that has an air duct located directly overhead C. One's significant other lightly stroking their face E. Walking outside on a windy day

A client is experiencing severe pain in his back to the point of being immobile and running a temperature. The client also has swelling in his lower back (vertebrae). Following biopsy, the results show spinal tuberculosis. The nurse will anticipate explaining which priority intervention to this client? A) Drain the abscess in the lower back B) Administer the four-drug antimicrobial medications C) Locate a chiropractor for pain control D) Log-roll the client while he is on bedrest

B. administer the four-drug antimicrobial medications

While explaining to the parents of a child diagnosed with postural scoliosis, the nurse will emphasize which of the following concepts? This form of scoliosis: A) Compresses vertebrae, causing nerve damage in distal limbs B) Can be corrected with bending and active/passive exercises C) Is congenital in nature and responds poorly to treatment measures D) Is often a long, C-shaped curve from the cervical area to sacral region and is very painful

B. can be corrected with bending and active/passive exercises

Assessment of a newborn infant reveals the presence of developmental dysplasia of the hip (DDH) that is currently demonstrated by subluxation of the baby's hip joint and a general laxity in the baby's ligaments. What measures should be emphasized in this infant's treatment? A) Corticosteroid therapy B) Close observation C) Open reduction D) Joint reconstruction

B. close observation

A child has been hospitalized for the treatment of hematogenous osteomyelitis. The defining characteristic of this type of osteomyelitis is: A) The presence of dead bone tissue B) Introduction of microorganisms from the bloodstream C) Bacterial proliferation in the absence of the classic signs of infection D) Destruction of the vascular network in the endosteum

B. introduction of microorganisms from the bloodstream

While explaining to a group of nursing students the difference between benign and malignant bone tumors, the instructor will emphasize that a benign tumor: Select all that apply. A) Is associated with constant, deep aching pain that does not go away with rest B) May be asymptomatic and detected incidentally C) Is a painful mass that is detected on a long bone and seems to be increasing in size D) May cause the client to have a pathological fracture

B. may be asymptomatic and detected incidentally D. may cause the client to have a pathological fracture

Following a knee replacement surgery, a nurse's next door neighbor asks, "I don't think I am healing right. Can you come look at my knee?" Upon assessment, the nurse notices the client is warm to touch and has a fever; the incision is inflamed and not well approximated with foul-smelling drainage around the incision line. At this point, the nurse tells the client she needs to go see her surgeon because the client may have: A) Contaminated the wound with MRSA B) Osteomyelitis C) An abscess in the pocket of the incision D) Potential bone cancer

B. osteomyelitis

Which of the following factors may adversely affect bone healing and therefore place the client at risk for long-term problems? Select all that apply. A) Immobilization due to skeletal traction B) Walking with a walker with minimal weight bearing on affected side C) Delayed union at the fracture site within a period considered the normal healing time D) History of uncontrolled diabetes mellitus with associated circulatory problems

C) Delayed union at the fracture site within a period considered the normal healing time D) History of uncontrolled diabetes mellitus with associated circulatory problems

The father of a 2-year-old boy recently diagnosed with hemophilia A asks the nurse how to prevent complications for his son. The best response would be: A) "Hemophilia A will no longer be a concern once treated with a blood transfusion." B) "Do not allow the child to play outside or with other children." C) "Avoid administering aspirin and non steroidal anti-inflammatory drugs." D) "Small bleeding in the joints is to be expected and can be treated at home."

C. "Avoid administering aspirin and nonsteroidal anti-inflammatory drugs."

A client diagnosed with type 2 diabetes has been instructed about managing his condition with diet. The nurse determines further teaching is necessary when the client states: A) "I can drink 8-10 glasses of water daily without concern for calories." B) "I need to avoid adding salt to my foods." C) "I must avoid all candies and cookies, but can eat unlimited amounts of pasta and breads." D) "I need to limit the amount of foods that I eat that contain trans fats."

C. "I must avoid all candies and cookies, but can eat unlimited amounts of pasta and breads."

The nurse is providing discharge instructions to a client who experienced a ligamentous sprain. The nurse determines that teaching was effective when the client states: A) "I will return tomorrow for the extremity to be casted." B) "Constant heat will improve circulation to the area and it will feel better." C) "I should rest, apply ice, compression, and elevation to the extremity." D) "I have no limitations to my activity."

C. "I should rest, apply ice, compression, and elevation to the extremity."

An adult client has been diagnosed with polycystic kidney disease. Which of the client's following statements demonstrates an accurate understanding of this diagnosis? A) "I suppose I really should have paid more attention to my blood pressure." B) "I've always been prone to getting UTIs, and now I know why." C) "I suppose I should be tested to see if my children might inherit this." D) "I had a feeling that I was taking too many medications, and now I know the damage they can do."

C. "I suppose I should be tested to see if my children might inherit this."

A 23-year-old female client has been diagnosed with von Willebrand disease following a long history of "heavy periods" and occasional nosebleeds. Which of the client's following statements demonstrates a sound understanding of her new diagnosis? A) "I'm really disappointed that I won't be able to do sports anymore." B) "I read on a website that I might have to get blood transfusions from time to time." C) "I'll make sure to take Tylenol instead of aspirin when I get aches and pains." D) "I hope my insurance covers the injections that I'll need to help my blood clot."

C. "I'll make sure to take Tylenol instead of aspirin when I get aches and pains."

The mother of a newborn infant questions why her baby needs vitamin K injection immediately after birth. The best response by the nurse would be: A) "The infant was exposed to high levels of heparin upon birth." B) "Infants have a higher body fat content, which prohibits the absorption of vitamin K." C) "Infants are not born with the normal intestinal bacteria that synthesize vitamin K for clotting." D) "It is hospital policy to administer the injection to newborns."

C. "Infants are not born with the normal intestinal bacteria that synthesize vitamin K for clotting."

A client diagnosed with Paget disease asks the health care provider how the disease developed. The best response would be: A) "It is a result of a sedentary lifestyle." B) "It results from a chromosomal disorder." C) "It is thought to have a probable association with a viral infection: paramyxovirus." D) "It can result from a deficiency in calcium."

C. "It is thought to have a probable association with a viral infection: paramyxovirus."

A 44-year-old woman has sought care for the treatment of headaches that have been increasing in severity and frequency and has been subsequently diagnosed with migraines. Which of the following teaching points should her care provider emphasize? A) "Weight loss and exercise are very important components of your treatment." B) "Stopping all of your current medications, even temporarily, should provide some relief." C) "It would be helpful for you to take control of your diet, sleep schedule, and stress levels." D) "Your headaches are likely a result of a nerve disorder and, unfortunately, cannot be treated successfully."

C. "It would be helpful for you to take control of your diet, sleep schedule, and stress levels."

A client who has had recurrent UTIs asks the nurse about the old wise tale of drinking cranberry juice daily. The nurse can respond: A) "There is no research on this topic, so I don't think it will help you." B) "Studies on this are based on a person drinking at least 1 gallon of juice/day." C) "Research suggests cranberry juice will reduce bacterial adherence to the lining of the urinary tract." D) "Beer is probably more effective at killing bacteria than cranberry juice."

C. "Research suggests cranberry juice will reduce bacterial adherence to the lining of the urinary tract."

In anatomy class, the instructor asks, "Explain how urine is expelled from the bladder during voiding." The student with the most accurate response would be: A) "The urothelium acts as a barrier to prevent urine from seeping into capillaries." B) "The beginning of micturition occurs when neurons send messages down to the pudendal nerve." C) "The detrusor muscle contract down on the urine and the ureteral orifices are forced shut. The external sphincter relaxes as urine moves out of the bladder." D) "It's really the external sphincter muscle that controls urination. The somatic nervous system innervates the muscles of the external sphincter and the pelvic floor muscles that together control the outflow of urine."

C. "The detrusor muscle contract down on the urine and the ureteral orifices are forced to shut. The external sphincter relaxes as urine moves out of the bladder."

A 38-year-old female is considering the use of oral contraceptives as a method of birth control. She asks her health care provider if any risks would be involved. The best response would be: A) "Why do you want the pill?" B) "Have you talked about this with your partner?" C) "Yes, risks increase with age." D) "There are no risks involved."

C. "Yes, risks increase with age."

A client experienced asymptomatic UTIs while pregnant. The client asks the nurse if this places her at any risk for complications. Which would be the best response by the nurse? A) "You are at risk for having a baby weighing over 10 pounds at birth." B) "You are at risk for a prolonged labor experience." C) "You are at risk for developing acute pyelonephritis." D) "You are at irks for delivering your baby beyond your due date."

C. "You are at risk for developing acute pyelonephritis."

A client has undergone a kidney transplant and voices concerns about organ rejection to the nurse. The most appropriate response by the nurse would be: A) "Your new kidney will continue to work fine as long as you do not drink any alcohol." B) "Kidney transplantation is 100% successful, and there is no need for you to worry." C) "You will be given medication to decrease your immune systems from attacking your new kidney." D) "There is no need to worry about rejection because the occurrence of rejection is low with kidney transplant."

C. "You will be given medication to decrease your immune systems from attacking your new kidney."

Mature red blood cells have a life span of approximately: A) 10 days B) 90 days C) 120 days D) 30 days

C. 120 days

Which of the following individuals likely faces the highest risk of megaloblastic anemia? A) A 69-year-old woman who takes ASA four times daily to treat her arthritis B) A 44-year-old man who lost approximately 500 mL of blood in a workplace accident C) A 21-year-old college student who lives a vegan lifestyle D) An infant who is exclusively fed commercial baby formula

C. A 21-year-old college student who lives a vegan lifestyle

Which of the following individuals has the highest chance of having a medulloblastoma? A) An 88-year-old man who has begun displaying signs and symptoms of increased ICP B) A 60-year-old woman who is soon to begin radiation therapy for the treatment of breast cancer C) A 4-year-old child who has become uncoordinated in recent months D) A 68-year-old man who is a smoker and has a family history of cancer

C. A 4-year-old child who has become uncoordinated in recent months

The nurse is reviewing assessment data on four clients. Select the client at highest risk for developing type 2 diabetes. A) A 60-year-old female with a history of gestational diabetes B) A 10-year-old male whose grandmother has type 2 diabetes C) A 45-year-old obese female with a sedentary lifestyle D) A 40-year-old male who has an active lifestyle

C. A 45-year-old obese female with a sedentary lifestyle

Which of the following clients is likely at the greatest risk of developing a urinary tract infection? A) A pregnant woman who has been experiencing urinary frequency B) A client with a diagnosis of chronic kidney disease who requires regular hemodialysis C) A 79-year-old client with an indwelling catheter for urinary incontinence D) A confused, 81-year-old client who is incontinent of urine

C. A 79-year-old client with an indwelling catheter for urinary incontinence

The health care provider has determined that a client has developed rheumatoid arthritis. The provider is aware that the client may experience joint inflammation that involves of immunologic mediation triggered by: A) A macrophage-mediated response that attacks self B) An exposure to a virus that triggers a B-cell response C) A T-cell-mediated response to an immunologic trigger, such as a microbial agent D) A decreased production of inflammatory mediations and cytokines

C. A T-cell-mediated response to an immunologic trigger, such as a microbial agent

The primary care provider for a newly admitted hospital client has added the glomerular filtration rate (GFR) to the blood work scheduled for this morning. The client's GFR results return as 50 mL/minute/1.73 m 2 . The nurse explains to the client that this result represents: A) A need to increase water intake B) The kidneys are functioning normally C) A loss of over half the client's normal kidney function D) Concentrated urine

C. A loss of over half the client's normal kidney function

The health care provider is reviewing diurnal variation pattern in adrenocorticotropic (ACTH) levels. Select the typical diurnal variation pattern in adrenocorticotropic (ACTH) levels. A) ACTH maintains a consist level regardless of the time of day B) ACTH peaks in correspondence with food intake C) ACTH peaks in the morning and declines throughout the day D) ACTH increases in the morning and peaks again in the evening hours

C. ACTH peaks in the morning and declines throughout the day

The nurse is caring for a 42-year-old male client who is admitted for treatment of heart failure. He has abnormally large hands and feet and a broad face with a protruding jaw. Based on these signs and symptoms, the nurse identifies which of the following endocrine disturbances as the most likely cause for these physical changes? A) Hyperthyroidism B) Cushing syndrome C) Acromegaly D) Myxedema

C. Acromegaly

Blood coagulation is initiated by either the intrinsic or extrinsic pathway. The final step in both pathways would be: A) Activation of factor XII B) Deactivation of fibrinogen, preventing the conversion to fibrin C) Activation of factor X, leading to conversion of prothrombin II to thrombin IIa D) Activation of factors VII, X, V, and II

C. Activation of factor X, leading to conversion of prothrombin II to thrombin IIa

A patient is being evaluated for kidney stones. The nurse anticipates the patient will manifest: A) A visible abdominal mass and abdominal pain B) Increased thirst and increased urinary output C) Acute onset of colicky or dull and achy intermittent flank pain D) Elevation in systemic blood pressure and frequent urination

C. Acute onset of colicky or dull and achy intermittent flank pain

A child is recovering from a bout with group A β-hemolytic Streptococcus infection. They return to the clinic a week later complaining of decrease in urine output with puffiness and edema noted in the face and hands. The health care provider suspects the child has developed: A) Autosomal recessive polycystic kidney disease B) Adult-onset medullary cystic disease C) Acute postinfectious glomerulonephritis D) Acute nephritic syndrome

C. Acute post infectious glomerulonephritis

A client is brought to the emergency department and is diagnosed with an ischemic stroke confirmed by CT scan. The most important treatment for this client would be to: A) Prepare the client for emergency surgery B) Administer analgesics for the relief of pain C) Administer IV tissue-type plasmin activator (tPA) D) Monitor vital signs closely for improvement

C. Administer IV tissue-type plasmin activator (tPA)

A 48-year-old male client diagnosed with type 2 diabetes presents with the following: Blood glucose level of 46 mg/dL Very lethargic Has cool, clammy skin Select the most appropriate intervention. A) Place surgared breath mints in the client's mouth B) Administer scheduled dose of insulin C) Administer injectable glucagon D) Encourage the client to sit up and drink 12 ounces of orange juice

C. Administer injectable glucagon

A 48-year-old male client diagnosed with type 2 diabetes presents with the following: Blood glucose level of 46 mg/dL Very lethargic Has cool, clammy skin Select the most appropriate intervention A) Encourage the client to sit up and drink 12 ounces of orange juice B) Administer scheduled dose of insulin C) Administer injectable glucagon D) Place sugared breath mints in the client's mouth

C. Administer injectable glucagon

An otherwise healthy client has been referred to a pain clinic because she claims to experience exquisite pain from the friction of her clothes on her torso. This client is likely to be diagnosed with which of the following health problems? A) Visceral pain B) Hypoalgesia C) Allodynia D) Primary hyperalgesia

C. Allondynia

Which of the following clients are at risk of developing folic acid deficiency anemia? Select all that apply. A) A vegetarian who consumes no meats but gets protein from nuts/legumes B) A HIV-positive client who consumes "green juice" at least twice/day made with organic fruits and vegetables C) An elderly male with poor dietary habits who drinks approximately five alcoholic beverages/day D) A pregnant client with prolonged morning sickness lasting most of the day with nausea and frequent vomiting noted E) A toddler who wants to eat chicken nuggets and French fries but refuses to eat anything green

C. An elderly male with poor dietary habits who drinks approximately five alcoholic beverages/day D. A pregnant client with prolonged morning sickness lasting most of the day with nausea and frequent vomiting noted

A client presents with signs and symptoms of candidiasis and is concerned about why this occurs after taking antibiotics. The most appropriate information for the nurse to provide would be: A) The bacterium is found in the vagina and can overgrow due to poor vaginal hygiene. B) Antibiotics cause an increase in vaginal glycogen, which promotes candidiasis C) Antibiotics suppress the normal protective vaginal flora allowing overgrowth of C. albicans D) The inappropriate use of antibiotics will cause overgrowth of candida.

C. Antibiotics suppress the normal protective vaginal flora allowing overgrowth of C. albicans

Glomerulonephritis is usually caused by: A) Vesicoureteral reflux B) Catheter-induced infection C) Antigen-antibody complexes D) Glomerular membrane viruses

C. Antigen-antibody complexes

The nurse is teaching a client with diabetes about medications that will increase the blood glucose level. The most important information for the nurse to provide would be: Select all that apply. A) Insulin B) Vitamins C) Antipsychotics D) Oral contraceptives E) Total parenteral nutrition F) Loop diuretics

C. Antipsychotics D. Oral contraceptives E. Total parenteral nutrition F. Loop diuretics

A 33-year-old client has been admitted to the hospital for the treatment of Graves disease. Which of the following assessments should the client's care team prioritize? A) Assessment of the client's level of consciousness and neurologic status B) Assessment of the client's peripheral vascular system for thromboembolism C) Assessment of the client's vision and oculomotor function D) Cardiac monitoring and assessment of peripheral perfusion

C. Assessment of the client's vision and oculomotor function

While talking about their migraine headaches, two women have found that they have some common triggers for their migraines, which may include: Select all that apply. A) Nonpharmacologic treatments like yoga B) Piercings of their nose and cheeks C) At the time of their menstrual cycle D) When drinking white wine E) Consuming chocolate

C. At the time of their menstrual cycle E. Consuming chocolate

In immune thrombocytopenia purpura (ITP), the client has what type of disorder that primarily destroys which blood component? A) Allergic; fibrinogen B) Alloimmune; factor VIII C) Autoimmune; platelets D) Immunoglobulin; B cells

C. Autoimmune; platelets

The nurse reviews the lab results of a client who has a thrombocyte count of 60,000/ul. The client is at risk for: A) Disseminated intravascular coagulation B) Deep vein thrombosis C) Bleeding D) Hypercoagulation

C. Bleeding

The nurse provides prophylactic treatment for a neonate to prevent conjunctival gonorrhea. The nurse is aware that the intervention will reduce the risk of: A) Pruritus B) Genital infection C) Blindness D) Urethritis

C. Blindness

Hypoparathyroidism causes hypocalcemia by: A) Increasing serum magnesium B) Increasing phosphate excretion C) Blocking release of calcium from bone D) Blocking action of intestinal vitamin D

C. Blocking release of calcium from bone

The most reliable method for measuring body water or fluid volume increase is by assessing: A) Tissue turgor B) Intake and output C) Body weight change D) Serum sodium levels

C. Body weight change

The nurse is assessing a client who has developed hypocalcemia. The nurse anticipates the assessment data to include: Select all that apply. A) Negative Trousseau sign B) Signs of kidney stones C) Bone pain D) Tetany E) Lethargy F) Serum calcium less that 8.5mg/dL

C. Bone pain D. Tetany F. Serum calcium less than 8.5mg/dL

Which of the following statements about bone mass and the elderly is accurate? A) Bone mass is increased with aging due to all the wear and tear of the joints. B) Once skeletal growth has been attained, there is no longer any replacement or repair. C) Bone resorption and formation are no longer perfectly coupled, and bone mass is lost. D) Hormones, like PTH, are much slower to be released as one ages.

C. Bone resorption and formation are no longer perfectly coupled, and bone mass is lost

The health care provider is teaching a client about the metabolic effect of cortisol. The most appropriate information to provide would be: A) Regulation of behavior and emotion B) Suppression the immune response to invasive pathogens C) Breakdown of proteins and fats D) Regulation of anti-inflammatory reactions

C. Breakdown of proteins and fats

Laboratory analysis of a vaginal discharge identifies hyphae on microscopic exam and pH of 4.2. The client most likely has: A) C. trachomatis B) N. gonorrhea C) C. albicans D) T. vaginalis

C. C. albicans

Which of the following measures should a public health nurse recommend to middle-aged women to reduce their chances of developing osteoporosis later in life? A) Weight control and daily use of low-dose corticosteroids B) Genetic testing and range-of-motion exercises C) Calcium supplementation and regular physical activity D) Increased fluid intake and use of vitamin D supplements

C. Calcium supplementation and regular physical activity

A female athlete has been diagnosed with amenorrhea due to intense training for a spot on the Olympic swimming team. As a health care provider, which of the following should be implemented to prevent premature osteoporosis? A) Encourage a minimum of 10 hours of sleep/night. B) Increase dietary intake of protein and iron. C) Calcium/vitamin D supplements to support BMD. D) Watch sodium intake and eat a carb-consistent diet with lots of fruits.

C. Calcium/ vitamin D supplements to support BMD

Which of the following assessments should be prioritized in the care of a client who is being treated for a serum potassium level of 2.7 mEq/L? A) Detailed fluid balance monitoring checking for pitting edema B) Arterial blood gases looking for respiratory alkalosis C) Cardiac monitoring looking for prolonged PR interval and flattening of the T wave D) Monitoring of hemoglobin levels and oxygen saturation

C. Cardiac monitoring looking for prolonged PR interval and flattening of the T wave

An elderly male client has been brought to the emergency department after experiencing stroke-like symptoms a few hours ago, and has been subsequently diagnosed with an ischemic stroke. The care team is eager to restore cerebral perfusion despite the likely death of the brain cells directly affected by the stroke. What is the rationale for the care team's emphasis on restoring circulation? A) Failure to restore blood flow creates a severe risk for future transient ischemic attacks. B) Necrosis will continue unabated throughout the brain unless blood flow is restored. C) Cells of the penumbra may be saved from hypoxic damage if blood flow is promptly restored. D) Unless blood flow is restored, the client faces the risk of progressing to hemorrhagic stroke.

C. Cells of the penumbra may be saved from hypoxic damage if blood flow is promptly restored

Drugs like diazepam (Valium), a benzodiazepine, exert their action on ion channels. These drugs do not open the GABA-operated ion channel, but they: A) Modulate the release from axon terminals B) Modulate the peripheral sympathetic nerves and can have both a transmitter and modulator function C) Change the effect that GABA has when it binds to the channel at the same time as the drug D) Play a necessary role in the long-term survival of presynaptic neurons

C. Change the effect that GABA has when it binds to the channel at the same time as the drug

The client's health history includes the diagnosis of a metatarsus adductus. The nurse would interpret this as: A) Condition of outtoeing B) Internal femoral anteversion C) Congenital deformity characterized by forefoot adduction D) Congenital dislocation at the hip

C. Congenital deformity characterized by forefoot adduction

Which cranial nerves that have their origin in the pons assist with the manipulation of the jaw during chewing and speech? Select all that apply. A) Cranial nerve I B) Cranial nerve III C) Cranial nerve VI D) Cranial nerve VII E) Cranial nerve VIII

C. Cranial nerve VI D. Cranial nerve VII E. Cranial nerve VIII

Select the laboratory blood test that would be a used to suggest a diagnosis of muscular dystrophy (MD). A) Serum glucose B) Rheumatoid factor C) Creatine kinase D) Amylase levels

C. Creatine kinase

A client has been diagnosed with condylomata acuminate. The nurse recognizes that the most likely treatment will be: A) Radiation therapy B) Antibiotics C) Cryotherapy D) Antifungal cream

C. Cryotherapy

Conditions that predispose to sickling of hemoglobin in persons with sickle cell anemia include: A) Impaired red blood cell maturation B) Increased iron content of blood C) Decreased oxygen saturation D) Increased intravascular volume

C. Decreased oxygen saturation

Warfarin has been prescribed for a client. The nurse expects that the client will have: A) Binding of antithrombin III, which prevents other clotting mechanisms to occur B) Inhibited activation of factor X C) Decreased prothrombin and other pro coagulation factors D) Suppressed formation of fibrin

C. Decreased prothrombin and other procoagulation factors

Client and family education regarding peritoneal dialysis should include assessing the client for: A) Bleeding around the arteriovenous fistula or an external arteriovenous shunt B) Signs and symptoms of hypoglycemia such as weakness, irritability, and shakiness C) Dehydration that may appear as dry mucous membranes or poor skin turgor D) Muscle cramps associated with hypoparathyroidism

C. Dehydration that may appear as dry mucous membranes or poor skin turgor

The client has a traumatic complete spinal cord transection at the C5 level. Based on this injury, the health care worker can expect the client to have control of which body function/part? A) Bladder B) Finger flexion C) Diaphragm D) Trunk muscle

C. Diaphragm

A pregnant woman who is beginning her third trimester has been diagnosed with a urinary tract infection (UTI). Which of the following factors most likely predisposed this client to the development of a UTI? A) Increased urine alkalinity during pregnancy B) Hypertrophy of the bladder wall C) Dilation of the upper urinary structures D) Spastic peristalsis of the ureters

C. Dilation of the upper urinary structures

Which of the following catecholamines may have a vasodilator effect on the renal blood vessels and be prescribed for clients in shock who are experiencing renal insufficiency manifestations? A) Enkephalin B) Norepinephrine C) Dopamine D) Acetylcholine

C. Dopamine

A 70-year-old female client tells the health care provider she is concerned because she is getting shorter and her posture is not as good as it used to be. The provider completes an assessment of her spine and notes a loss of height in the vertebral column and kyphosis. This assessment would be recognized as: A) Scoliosis B) Lordosis C) Dowager hump D) Degenerative joint disease

C. Dowager hump

Regardless of the cause, chronic kidney disease results in progressive permanent loss of nephrons and glomerular filtration, and renal: A) Tubule dysplasia B) Vascular pressure C) Endocrine functions D) Hypophosphatemia

C. Endocrine function

A young adult has been diagnosed with a tumor of the femur, and the health care provider suspects a malignancy. Upon further assessment, the client states that he experiences bone pain, limitation of movement, and tenderness over the involved bone area. Select the most likely diagnosis. A) Osteonecrosis B) Osteosarcoma C) Ewing sarcoma D) Chondrasarcoma

C. Ewing sarcoma

A client has been diagnosed with inherited hyper coagulability. Select the most likely cause. A) Hyperestrogenic state B) Myocardial infarction C) Factor V gene mutation D) Prolonged immobility

C. Factor V gene mutation

Following surgery for a large malignant brain tumor, the nurse should anticipate discussing which further treatment option with the family that may ensure that any remaining cancer cells will be killed? A) Chemotherapy B) Immunotherapy C) Gamma knife radiation D) Stem cell transplant

C. Gamma knife radiation

A 22-year-old female client states she has vaginal itching with pain and small vesicles that appear on her vagina when she is stressed. The client is most likely experiencing. A) Genital warts B) Molluscum contagiosa infection C) Genital herpes D) Chlamydia trachomatis infection

C. Genital herpes

A client tells the health care provider that he has been very compliant over the last 2 months in the management of his diabetes .The best diagnostic indicator that would support the client's response would be: A) Capillary blood glucose sample B) Urine test C) Glycosylated hemoglobin, hemoglobin A1C (HbA1C) D) Fasting blood glucose level

C. Glycoslylated hemoglobin, hemoglobin A1C (HbA1c)

A client who has been diagnosed with gonorrhea asks the nurse if anyone has to know about it. The best response by the nurse is: A) If the client abstains from sexual intercourse, it does not need to be reported. B) It is only reported if a client is married C) Gonorrhea is a reportable sexually transmitted infection D) It is only reported after the second infection

C. Gonorrhea is a reportable sexually transmitted infection

Which of the following assessment findings of a male client constitutes a criterion for a diagnosis of metabolic syndrome? The client: A) States that he does less than 30 minutes of strenuous physical activity each week B) Has a resting heart rate between 85 and 95 beats/minute C) Has blood pressure that is consistently in the range of 150/92 mm Hg D) Has a fasting triglyceride level of 100 mg/dL

C. Has blood pressure that is consistency in the range of 150/92 mm Hg

Select the option that best describes the pathogenesis of the anemia in a client with beta-thalassemia. A) Increased levels of hemoglobin (hyperchomia) occur B) Premature destruction fo cells occurs due to riding, nodeformable membranes C) Heinz bodies impair DNA synthesis and damage the red cell membrane. D) HbS becomes sickled when deoxygenation occurs.

C. Heinz bodies impair DNA synthesis and damage the red cell membrane

Which signs and symptoms would you expect to see in a patient diagnosed with acute nephritic syndrome? A) Generalized edema and hypotension B) Proteinuria and hyperlipidemia C) Hematuria and azotemia D) Hypoalbuminemia and lipiduria

C. Hematuria and azotemia

A female client has been diagnosed with anti phospholipid syndrome. The nurse would anticipate the client to have a health history that may include: Select all that apply. A) Large birth weight babies B) Multiple births C) Hypertension during pregnancy D) Pregnancy loss before the 10th week E) Recurrent fetal loss F) Uteroplacental insufficiency

C. Hypertension during pregnancy E. Recurrent fetal loss F. Uteroplacental insufficiency

The nurse is providing care for a client who has a diagnosis of kidney failure. Which of the following laboratory findings is consistent with this client's diagnosis? A) Elevation in vitamin D levels B) Hypophosphatemia C) Hypocalcemia D) Hypokalemia

C. Hypocalcemia

A family brings a client to the emergency department with increasing lethargy and disorientation. They think the client had a seizure on the drive over to the hospital. The client has been sick with a "cold virus" for the last few days. On admission, the client' temperature is 102°F. Which other clinical manifestations may lead to the diagnosis of encephalitis? A) Petechia over entire body B) BP 100/72 C) Impaired neck flexion resulting from muscle spasm D) Appearance of red-purple discolorations on the skin that do not blanch on applying pressure

C. Impaired neck flexion resulting from muscle spasm

While trying to explain the physiology behind type 2 diabetes to a group of nursing students, the instructor will mention which of the following accurate information? A) The destruction of beta cells and absolute lack of insulin in people with type 2 diabetes means that they are particularly prone to the development of diabetic complication. B) Because of the loss of insulin response, all people with type 2 diabetes require exogenous insulin replacement to control blood glucose levels. C) In skeletal muscle, insulin resistance prompts decreased uptake of glucose. Following meals (postprandial), glucose levels are higher due to diminished efficiency of glucose clearance. D) They have increased predisposition to other autoimmune disorders such as Graves disease, rheumatoid arthritis, and Addison disease.

C. In skeletal muscle, insulin resistance prompts decreased uptake of glucose. Following meals (postprandial), glucose levels are higher due to diminished efficiency of glucose clearance

A client reports that she frequently suffers from UTIs after engaging in sexual intercourse. Which would be the best information for the nurse to provide? A) Decrease fluid intake before intercourse B) Delay voiding for 5 hours after intercourse C) Increase fluid intake before intercourse D) Refrain from intercourse

C. Increase fluid intake before intercourse

A nurse is teaching a client newly diagnosed with diabetes about the action of insulin. The most appropriate information for the nurse to provide would be that it: A) Decreases the storage of glucose B) Promotes breakdown of stored triglycerides C) Increases protein synthesis D) Promotes gluconeogenesis

C. Increases protein synthesis

During the assessment of a client with heart failure, the nurse uses finger pressure to determine if edema is present in the lower extremities. When would the nurse document pitting edema? A) The skin is thickened and hard. B) Indentation is not present. C) Indentation remains after the finger has been removed. D) the area is firm and discolored.

C. Indentation remains after the finger has been removed.

Select the most appropriate intervention for the nurse to teach a client diagnosed with distal symmetric neuropathy related to diabetes. A) Rotate insulin injection sites once a week. B) Wear comfortable, open-toe shoes C) Inspect the feet for blisters daily D) Decrease daily walking activity

C. Inspect the feet for blisters daily

The diagnosis of type 1 diabetes would be confirmed by: A) Insulin is present in large amounts for use by the body B) Insulin is produced but unavailable for use in the body C) Insulin is not available for use by the body D) Small amounts of insulin are produced daily

C. Insulin is not available for use by the body

Among the treatments for multiple sclerosis (MS), which medication will reduce the exacerbation of relapsing-remitting MS? A) Long-term corticosteroid administration B) Mitoxantrone, an antineoplastic agent C) Interferon-β, a cytokine injection D) Baclofen, a muscle relaxer

C. Interferon-B, a cytokine injection

A 72-year-old woman with complaints of increasing fatigue has completed a series of fecal occult blood tests that indicate the presence of blood in her stool. Which of the following health problems is likely to accompany this client's gastrointestinal bleed? A) Hemolytic anemia B) Aplastic anemia C) Iron deficiency anemia D) Megaloblastic anemia

C. Iron deficiency anemia

A client's emergency magnetic resonance imaging (MRI) has been examined by the physician and tissue plasminogen activator (tPA) has been administered to the client. What was this client's most likely diagnosis? A) Status epilepticus B) Subarachnoid hemorrhage C) Ischemic stroke D) Encephalitis

C. Ischemic stroke

A client is admitted to the emergency department and diagnosed with diabetic keto acidosis (DKA). The client would most likely manifest: A) Hypervolemia B) Respiratory alkalosis C) Ketosis D) Hypoglycemia

C. Ketosis

The nurse has just completed teaching a client newly diagnosed with type 1 diabetes about rapid-acting insulin. The nurse determines that teaching was effective when the client selects: A) Regular B) NPH C) Lispro D) Glargine

C. Lispro

A client has developed spinal shock. The most important assessment for the nurse to perform would be: A) Vasoconstriction causing hypertension B) Paresthesia of the extremities below the injury C) Loss of tendon reflexes D) Inability to void or defecate

C. Loss of tendon reflexes

Sue is fatigued, and some blood tests are done. Her results include Hct 40%; Hgb 8g/dL; WBC 8000; and platelets 175,000. The nurse should interpret Sue's blood work as indicative of: A) High platelets/thrombocytosis B) Low WBC count/granulocytopenia C) Low hemoglobin/anemia D) High hematocrit/polycythemia

C. Low hemoglobin/anemia

A client is brought to the emergency department with a suspected diagnosis of DKA (diabetic keto acidosis). Select the assessment/diagnostic data to confirm the diagnosis. Select all that apply. A) Hypoglycemia B) High arterial pH C) Low serum bicarbonate D) Negative serum ketones E) Positive urine ketones

C. Low serum bicarbonate E. Positive urine ketones

A client tells the nurse that the client has been taking Alka-Seltzer (bicarbonate-antacid) four times a day for the past 2 weeks for an upset stomach. Upon assessment of the client, the nurse notes hyperactive reflexes, tetany, and mental confusion. Arterial blood gases reveal pH 7.55; serum HCO3- 37. The nurse suspects the client may be experiencing: A) Metabolic acidosis B) Respiratory acidosis C) Metabolic alkalosis D) Respiratory alkalosis

C. Metabolic alkalosis

What role do the basal ganglia play in cognition? A) Make it possible for the motor neurons to innervate a few to thousand of muscle fibers and provide gross motor movement B) Provide central control over the gamma motor neurons to increase or decrease muscle tone in anticipation fo changes in muscle force C) Monitor sensory information coming into the brain and apply it to information stored in the memory D) Initiate movement to avoid hazardous situations, while the stretch receptors serve to integrate motor movement, so they function in a coordinated manner

C. Monitor sensory information coming into the brain and apply t to information stored in the memory

A 2-week-old infant (full-term at birth) is admitted to the pediatrics unit with "spitting up large amounts of formula" and diarrhea. The infant has developed a weak suck reflex. Which of the following statements about total body water (TBW) is accurate in this situation? A) About 52% of the infants' weight accounts for the amount of water in their body. B) Because of the infants' higher fat ratio, one should anticipate an increased TBW to as high as 90%. C) Most full-term infants have a TBW of approximately 75% due to their high metabolic rate. D) Most of an infant's TBW remains in the ICF compartment, so they should be able to transfer needed water into the ECF space.

C. Most full-term infants have a TBW of approximately 75% due to their high metabolic rate

A child with a diagnosis of Legg-Calvé-Perthes disease will exhibit: A) Defective synthesis of type I collagen B) Congenital dislocation of the acetabulofemoral joint C) Necrosis of the proximal femoral head D) Intoeing due to metatarsus adductus

C. Necrosis of the proximal femoral head

A 60-year-old woman has been recently diagnosed with multiple sclerosis, a disease in which the oligodendrocytes of the client's central nervous system (CNS) are progressively destroyed. Which physiologic process within the neurologic system is most likely to be affected by this disease process? A) Oxygen metabolism B) Neurotransmitter synthesis C) Nerve conduction D) Production of cerebrospinal fluid

C. Nerve conduction

Which stresses can cause injury to the kidney glomerulus? Select all that apply. A) Arthritis B) Asthma C) Nonimmunologic D) Immunologic E) Diabetes F) Heredity

C. Nonimmunologic D. Immunologic E. Diabetes F. Heredity

Which condition increases a patient's chance of developing an ascending urinary tract infection? A) Seeding of the kidney in the course of infective endocarditis B) Transfer of pathogens in the course of septicemia C) Obstruction of urine outflow from the kidneys D) The presence of high levels of glucose in the urine

C. Obstruction of urine outflow from the kidney

The most common cause of thyrotoxicosis is Graves disease. When assessing this client, the nurse should put priority on which of the following signs/symptoms? A) Complaints of muscle fatigue B) Facial myxedema with puffy eyelids C) Ophthalmopathy D) Pulse rate of 64 beats/minute

C. Ophthalmology

A client asks the nurse there is anything that would greatly increase her risk of acquiring human papilloma virus (HPV). The best response by the nurse would be: A) Cigarette smoking B) First intercourse after age 30 C) Oral-genital sex D) Age greater than 35 years

C. Oral-genital sex

An adult female client visits with her health care provider about pain in her hand. She describes it as an audible grinding and cracking sound, especially in her thumb. "I had to buy an automatic jar opener...I just can't grasp and open a jar...it just hurts too badly." The health care provider suspects the client has a degenerative form of joint disease that is often evidenced by: A) Rheumatoid arthritis B) Systemic lupus erythematosus C) Osteoarthritis D) Ankylosing spondylitis

C. Osteoarthritis

A female client presents to the orthopedic clinic for evaluation. The primary care provider told her she has a "spur" on her joint. She asks, "Why did this happen? I guess I just didn't exercise enough." The nurse recognizes this to be a later structural change of osteoarthritis (OA), where the client no longer has a "shock absorber," culminating in: A) Osteonecrosis and loss of synovial fluid B) Formation of tophi in the synovial space C) Osteophyte formation and erosion of cartilage D) Separation of the epiphyseal plate

C. Osteophyte formation and erosion of cartilage

How are the oxygenation needs of the articulating areas in a synovial joint met? A) Capillaries release oxygenated blood into the synovial cavity at a controlled rate. B) The cartilage of the articulating areas uses anaerobic metabolism exclusively. C) Oxygen is provided to the articulating areas indirectly by synovial fluid. D) The epiphyses of long bones provide oxygen to the articulating areas.

C. Oxygen is provided to the articulating areas indirectly by synovial fluid

Select the most common symptoms of diabetes. Select all that apply. A) Polyhydramnios B) Polycythemia C) Polyphagia D) Polydipsia E) Polyuria

C. Polyphagia D. Polydipsia E. Polyuria

A young adult male client presents to the orthopedic clinic complaining of "stiffening of the spine." The health care provider orders some diagnostic lab work. Which lab result leads the health care worker to diagnose ankylosing spondylitis? A) Elevated serum calcium level of 15.1 mg/dL B) Severe decrease in red blood cells associated with decreased iron levels C) Presence of HLA-B27 allele marker D) Elevated serum uric acid level

C. Presence of HLA-B27 allele marker

One of the first signs that indicates an infant may have congenital hypothyroidism is: A) No passage of meconium within the first 72 hours after birth B) Palpable mass in the neck region C) Prolonged period of physiologic jaundice D) Full, bounding fontanels

C. Prolonged period of physiologic jaundice

A client who has a spinal cord injury exhibits pallor and goose pimples, and the blood pressure is 170/100. The most important action for the nurse to take would be to: A) Reposition the client supine and increase the rate of the IV fluids B) Take the client's blood pressure every 15 minutes until the physician arrives C) Raise the head of the bed and empty the bladder D) Turn the client to the left side and insert an indwelling urinary catheter

C. Raise the head of the bed and empty the bladder

An elderly female client has been hospitalized for the treatment of acute pyelonephritis. Which of the following characteristics of the client is most likely implicated in the etiology of her current health problem? The client: A) Has been diagnosed with type 2 diabetes several years earlier B) Takes a diuretic and an ACE inhibitor each day for the treatment of hypertension C) Recently had a urinary tract infection D) Has peripheral vascular disease

C. Recently had a urinary tract infection

A client is experiencing chest pain that radiates to the left arm and neck. The nurse would interpret this pain as: A) Cutaneous B) Somatic C) Referred D) Visceral

C. Referred

While lifting weights during football season, a player hears a loud "pop." This is primarily associated with a loss of which function? A) Autonomic nerve fibers B) Supporting bursa sacs C) Reflexive proprioception D) Elastic articular cartilage

C. Reflexive proprioception

Removal of a client's peripheral intravenous catheter resulted in brief bleeding and the loss of a small amount of blood. Which of the following processes occurred during the formation of the platelet plug that helped to stop blood flow? A) Activation of factor X B) Conversion of prothrombin to thrombin C) Release of von Willebrand factor from the epithelium D) Conversion of fibrinogen to fibrin threads

C. Release of Von Willebrand factor from the epithelium

A client with a diagnosis of myasthenia gravis has required a mastectomy for the treatment of breast cancer. The surgery has been deemed a success, but the client has gone into a myasthenic crisis on postoperative day 1. Which of the following measures should the care team prioritize in this client's immediate care? A) Positioning the client to minimize hypertonia and muscle rigidity B) Seizure precautions with padded side rails and bed in lowest height C) Respiratory support and protection of the client's airway D) Monitoring the client for painful dyskinesias

C. Respiratory support and protection of the client's airway

A client is devastated to receive a diagnosis of amyotrophic lateral sclerosis (ALS). The symptomatology of this disease is a result of its effects on upper and lower motor neurons. The health care provider caring for this client will focus on which priority intervention for this client? A) Ability to turn from side to side, thereby preventing skin breakdown B) Ability to empty bladder completely, thereby preventing autonomic dysreflexia C) Respiratory ventilation assessment and prevention of aspiration pneumonia D) Assessment of lower extremities to prevent deep vein thrombosis

C. Respiratory ventilation assessment and prevention of aspiration pneumonia

Primary adrenal insufficiency is manifested by: A) Truncal obesity and 3+ pitting edema in lower legs B) Potassium level of 2.8 mEq/L and weight gain of 3 pounds overnight C) Serum sodium level of 120 mmol/L (low) and blood glucose level of 48 mg/dL (low) D) Hypopigmentation over neck and BP greater than 150/90

C. Serum sodium level 120 mmol/L (low) and blood glucose level of 48 mg/dL (low)

A client is experiencing a cluster headache. The client would most likely manifest: A) Nausea with vomiting B) Sensitivity to light C) Severe pain behind the eye D) Symptoms aggravated by physical activity

C. Severe pain behind the eye

In contrast to synarthrosis joints, which synovial joints allow for movement in all directions? A) Knee joints B) Elbow joints C) Shoulder joints D) Vertebrocostal joints

C. Shoulder joint

When providing discharge teaching related to some newly prescribed medications, the client who received a head injury on the left temporal lobe area will need for the health care provider to: A) Print materials using extra large font sizes, so the client will be able to see the instructions B) Bring one of the pills into the room during teaching, so the client can recognize the color and shape C) Sit on the right side and speak slowly and clearly during the education D) Draw the shades on the window to minimize glare from bright lights

C. Sit on the right side and speak slowly and clearly during the education

The nurse is caring for a client diagnosed with diabetes mellitus who is reporting burning pain of his feet. The nurse would interpret this as: A) Intermittent claudication B) Autonomic neuropathy C) Somatic neuropathy D) Peripheral vascular disease

C. Somatic neuropathy

The health care provider is assessing a client to differentiate a herniated disk from other causes of acute back pain. The most important test for the provider to perform would be: A) Apgar test B) Deep tendon reflex test C) Straight-leg test D) Pain rating scale

C. Straight-leg test

A client presents to the emergency department after suffering an injury while playing sports. The client is diagnosed with a sprain. Select the best explanation of the injury. A) Hyperextension of a muscle B) unusual muscle contraction C) Tear of a ligamentous structure D) Excessive stretch of a muscle

C. Tear of a ligamentous structure

Intercellular collagen fibers provide what function for tendons and ligaments? A) Elastic recoil B) Shape flexibility C) Tensile strength D) Inorganic calcium

C. Tensile strength

A client with a diagnosis of lung cancer has developed bone metastases resulting in severe and protracted pain. Which of the following assessment components should the nurse prioritize when assessing the client's pain? A) The appearance of grimacing, guarding, or wincing B) The presence of changes in vital signs that correspond to pain C) The client's subjective report of the character and severity of pain D) The results of a detailed neurologic assessment

C. The client's subjective report of the character and severity of pain

The nurse is caring for a 2-day-old newborn infant who appears lethargic and has a yellowish tint to the skin. Select the most likely cause of this newborn's signs and symptoms. A) A decrease in the breakdown of red blood cells B) The inability of a newborn to produce bilirubin C) The inability of the immature liver to conjugate bilirubin D) Cirrhosis of the liver

C. The inability of the immature liver to conjugate bilirubin

A client with long-standing type 2 diabetes is surprised at his high blood sugar readings while recovering from an emergency surgery. Which of the following factors may have contributed to the client's inordinately elevated blood glucose levels? A) The tissue trauma of surgery resulted in gluconeogenesis. B) Illness inhibited the release and uptake of glucagon. C) The stress of the event caused the release of cortisol. D) Sleep disruption in the hospital precipitated the dawn effect.

C. The stress of the event caused the release of cortisol

A parent asks the nurse what signs and symptoms a child would display if the child had a UTI. Which is the best response by the nurse? A) The same symptoms as an adult. B) Foul-smelling urine is not a sing of infection C) They do not present with typical symptoms D) They will not have afever

C. They do not present with typical symptoms

A nurse is assisting a client with self-care and observes the following: Areas of bruising (purpura) on the client't arms and legs Pinpoint hemorrhages (petechiae) on the legs Pinpoint hemorrhages (petechiae) in the mouth Platelets level of 90,000/ul The nurse recognizes these data as a manifestation of: A) Neutropenia B) Leukopenia C) Thrombocytopenia D) Thrombocytosis

C. Thombocytopenia

A client had excessive blood loss and prolonged hypotension during surgery. His postoperative urine output is sharply decreased, and his blood urea nitrogen (BUN) is elevated. The most likely cause for the change is acute: A) Prerenal inflammation B) Bladder outlet obstruction C) Tubular necrosis D) Intrarenal nephrotoxicity

C. Tubular necrosis

A 40-year-old female with the diagnosis of multiple sclerosis has been experiencing severe bladder spasms along with less bladder volume. This is likely due to: Select all that apply. A) Bladder atonia B) Autonomic hyperreflexia C) Uninhibited bladder D) Neurogenic detrusor overactivity

C. Uninhibited bladder D. Neurogenic detrusor overactivity

A client tells the nurse that he is experiencing involuntary loss of urine associated with a strong desire to void (urgency). The nurse would recognize this as: A) Overflow incontinence B) Transient incontinence C) Urge incontinence D) Stress incontinence

C. Urge incontinence

An obese, male client with a history of gout and a sedentary lifestyle has been advised by his primary care provider to avoid organ meats, certain fish, and other foods that are high in purines. The care provider is demonstrating an awareness of the client's susceptibility to which of the following types of kidney stones? A) Calcium stones B) Magnesium ammonium phosphate stones C) Uric acid stones D) Cystine stones

C. Uric acid stones

A client with a long history of cigarette smoking and poorly controlled hypertension has experienced recent psychomotor deficits as a result of hemorrhagic brain damage. The client's psychomotor deficits are likely the result of: A) Alzheimer disease B) Frontotemporal dementia (FTD) C) Vascular dementia D) Wernicke-Korsakoff syndrome

C. Vascular dementia

Select the first stage of hemostasis. A) Clot reaction B) Clot dissolution C) Vessel spasm D) Formation of the platelet plug

C. Vessel spasm

The nurse is assessing a client with fluid volume excess. The nurse anticipates the client would manifest: A) Increased BUN B) Weak, rapid pulse C) Weight gain D) Decreased blood pressure

C. Weight gain

A female client asks, "Why do I leak urine every time I cough or sneeze?" The health care worker's response is based on which physiologic principle? A) Involuntary bladder continence during filling B) A pressure difference between the urethra and bladder C) When intravesical pressure exceeds maximal urethral closure pressure D) A decrease in bladder distensibility

C. When intravesical pressure exceeds maximal urethral closure pressure

A client with a diagnosis of small cell lung carcinoma has developed bone metastases, a finding that has prompted a series of new interventions. What are the primary goals of the treatment regimen for this client's bone cancer? A) Prevention of brain metastasis and early identification of osteonecrosis B) Promotion of bone remodeling at tumor sites through calcium and vitamin D supplements C) Prevention of pathologic fractures and maximization of function D) Pain management and prevention of osteomyelitis

C. prevention of pathologic fractures and maximization of function

The parents of a child diagnosed with rheumatic disease are shocked by the diagnosis and tell the nurse that they did not think children could acquire the disease. The best response would be: A) "Do not worry; a course of antibiotic therapy will cure the child." B) "Children who acquire rheumatic diseases are more likely to have a higher intelligence level." C) "Your child will not be restricted with any activities and will have a normal lifestyle." D) "Children can be affected with almost all of the rheumatic diseases the occur in adults."

D. "Children can be affected with almost all of the rheumatic diseases that occur in adults."

A nursing student asks her instructor, "I don't understand this coagulation system. When we donate blood, what keeps it from clotting in the bag?" The instructor responds: A) "Calcium gluconate is added to the bag to keep it from clotting." B) "If the blood is used within 24 hours from retrieval, there will be no clot formation yet." C) "The blood bank adds heparin to every bag prior to actually collecting the blood from the donor." D) "Citrate is added to the blood bag, which prevents it from clotting."

D. "Citrate is added to the blood bag, which prevents it from clotting."

A neighbor is complaining to a friend (who happens to be a nurse) about several changes in their body. Which of the following complaints raises a "red flag" because it could be a sign of epithelial cell bladder cancer? A) "Seems like I'm holding onto more water these days." B) "Every now and then, I have urine leak when I cough." C) "Sometimes I get a sharp pain in my side while exercising." D) "I noticed my urine is pinkish red, but I'm not having any pain when I pee."

D. "I noticed my urine is pinkish red, but I'm not having any pain when I pee."

The mother of a 2-year-old newly diagnosed with type 1 diabetes asks why insulin has to be given by injection. The best response by the nurse is: A) "Your child is not old enough to swallow the pills needed to treat her diabetes." B) "Insulin needs to go directly into the vein to work best." C) "When your child gets old enough, you will not have to administer injections." D) "Insulin is destroyed by the stomach contents and has to be administered by injection."

D. "Insulin is destroyed by the stomach contents and has to be administered by injection."

A diabetic client presents to a clinic for routine visit. Blood work reveals a HbA1C of 11.0% (high)? Which response by the patient may account for this abnormal lab result? A) "I've had more periods of hypoglycemia than usual over the past few months." B) "I've been doing great. I haven't needed much insulin coverage before meals." C) "To tell you the truth, my blood glucose levels have been pretty normal for me." D) "My meter broke so I have not been checking my blood glucose levels for a while."

D. "My meter broke so I have not been checking my blood glucose levels for a while."

A client is very much concerned about the harmful effect that all microorganisms may have. The best response by the nurse would be: A) "All microorganisms are harmful and you should be worried." B) "Daily use of antibiotics as a preventative measure will prevent harm." C) "Only the outside of the body may have potentially harmful microorganisms." D) "Not all interactions between microorganisms and humans are harmful."

D. "Not all interactions between microorganisms and humans are harmful."

A client asks if pain threshold and pain tolerance are the same. The best response by the health care provider would be: A) "Pain threshold is not varied for person to person." B) "Pain tolerance is not affected by physiological, familial, cultural, and environmental factors." C) "Pain tolerance is the minimum intensity of pain a client will endure." D) "Pain threshold is the point at which a stimulus is perceived as painful."

D. "Pain threshold is the point at which a stimulus is perceived as painful."

The health care provider has prescribed Repaglinide 2 mg for a client diagnosed with type 2 diabetes. The most important information for the nurse to give the client would be: A) "Use a different injection site for each injection" B) "Keep the medication refrigerated until used." C) "Adjust the dosing based on recent blood glucose readings." D) "Take the medication 15 to 30 minutes before each meal."

D. "Take the medication 15 to 30 minutes before each meal"

The parents of a 6-year-old child diagnosed with bilateral genu valium (knock knees) ask the health care provider when treatment will begin to correct the problem. The best response would be: A) "Since the angle is greater than 15 degrees and unilateral, and the child is short, intervention is not necessary." B) "The child's angle is less than 15 degrees and will have permanent damage even if we intervene now." C) "You child is tall and needs immediate surgery." D) "The child's measurements fit within the acceptable assessment period; treatment is not necessary at this time."

D. "The child's measurements fit within the acceptable assessment period; treatment is not necessary at the time."

A hospital client has been reluctant to accept morphine sulfate despite visible signs of pain. Upon questioning, the client reveals that he is afraid of becoming addicted to the drug. How can a member of the care team best respond to the client's concern? A) "You might become addicted, but there are excellent resources available in the hospital to deal with that development." B) "You should likely prioritize the control of your pain over any fears of addiction that you have." C) "If you start needing higher doses to control your pain, then we'll address those concerns." D) "There's only a minute chance that you will become addicted to these painkillers."

D. "There's only a minute chance that you will become addicted to these painkillers."

A young child develops type 1A diabetes. The parents ask, "They tell us this is genetic. Does that mean our other children will get diabetes?" The best response by the health care provider would be: A) "Probably not since genetically your other children have a different cellular makeup, they just might not become diabetic." B) "If you put all your children on a low-carbohydrate diet, maybe they won't get diabetes." C) "We don't know what causes diabetes, so we will just have to wait and see." D) "This autoimmune disorder causes destruction of the beta cells, placing your children at high risk of developing diabetes."

D. "This autoimmune disorder causes destruction of the bet cells, placing your children at high risk for developing diabetes"

A client tells the nurse that he has recently begun to take over-the-counter (OTC) calcium supplements to ensure that his blood will clot. The best response by the nurse would be: A) "It will not cause problems, and it will keep your bones strong." B) "Take the supplement twice a day with milk to increase absorption." C) "It is a good idea; you will prevent bleeding." D) "This is not necessary, unless it has been prescribed by your health care provider."

D. "This is not necessary, unless it has been prescribed by your health care provider

The health care provider is comparing results of a client's recent GFR measurement. Which of the following results would be interpreted as normal? A) 100 to 110 mL/minute B) 135 to 145 mL/minute C) 70 to 114 mL/minute D) 120 to 130 mL/minute

D. 120 to 130 mL/minute

The nurse is caring for the following group of clients. Select the client most likely to be diagnosed with respiratory alkalosis. A) A 63-year-old male with a 40-year history of smoking and chronic lung disease B) A 45-year-old male with pneumothorax after a car accident C) A 18-year-old female who has overdosed on narcotics D) A 26-year-old female with anxiety who has been hyperventilating

D. A 26-year-old female with anxiety who has been hyperventilating

The nurse is caring for four clients. Select the client at greatest risk for developing Alzheimer disease. A) A 65-year-old with a history of substance abuse B) A 20-year-old with a history of head trauma C) A 75-year-old with no significant family history or injury D) A 56-year-old with a history of Down Syndrome

D. A 56-year-old with a history of Down syndrome

The nurse is reviewing the assessment data of four clients. Select the client who would be diagnosed with precocious puberty. A) A 13-year-old female with menarche and a recent growth spurt B) A 16-year-old male with genital enlargement and pubic hair growth C) A 12-year-old male with advanced growth D) A 7-year-old female with early menarche

D. A 7-year-old female with early menarche

The nursing is reviewing assessment data of four clients. Select the client who is at greatest risk for developing pernicious anemia. A) A client of Italian heritage B) A client who is recovering from a minor surgical procedure C) A client diagnosed with a neoplastic disorder D) A client who has undergone partial gastrectomy

D. A client who has undergone partial gastrectomy

Which of the following individuals likely faces the greatest risk for the development of chronic kidney disease? A) A first-time mother who recently lost 1.5 L of blood during a postpartum hemorrhage B) A client whose diagnosis of thyroid cancer necessitated a thyroidectomy C) A client who experienced a hemorrhagic stroke and now has sensory and motor deficits D) A client with a recent diagnosis of type 2 diabetes who does not monitor his blood sugars or control his diet

D. A client wit ha recent diagnosis of type 2 diabetes who does not monitor his blood sugars or control his diet

What happens during the depolarization phase of nerve cells? A) It is an undisturbed period of the action potential during which the nerve is not transmitting impulses. B) The cell membrane decreases its permeability to sodium. C) The neurons are stimulated to fire. D) A rapid change in polarity to one that is positive on the inside and the membrane becomes open to sodium.

D. A rapid change in polarity to one that is positive on the inside and the membrane becomes open to sodium

The health care provider is evaluating a client with a suspected benign bone tumor. The provider confirms the diagnosis after evaluation of the client's x-ray results. Select the best description of the results. A) Abnormal bone merging with surrounding normal bone tissue B) An area of partially destroyed bone adjacent to completely normal areas C) A mouth-eaten pattern of bone destruction D) A uniform and well-defined area with well-demarcated edges

D. A uniform and well-defined area with well-demarcated edges

A client who experienced a traumatic head injury from a severe blow to the back of his head now lives with numerous function deficits, including an inability to maintain steady posture while he is in a standing position, although he is steadier when walking. Which of the following disorders most likely resulted from his injury? A) Cerebellar dystaxia B) Cerebellar tremor C) A lower motor neuron lesion D) A vestibulocerebellar disorder

D. A vestibulocerebellar disorder

A lung cancer client with small cell carcinoma may secrete an excess of which hormone causing an ectopic form of Cushing syndrome due to a nonpituitary tumor? A) GH B) TSH C) DHEA D) ACTH

D. ACTH

A client has been given the diagnosis of diffuse glomerulonephritis. They ask the nurse what diffuse means. The nurse responds: A) Only some of the glomeruli are affected. B) Only one segment of each glomerulus is involved. C) That the mesangial cells are being affected. D) All glomeruli and all parts of the glomeruli are involved.

D. All glomeruli and all parts of the glomeruli are involved

Which of the following clients would be considered high risk for falling and fracturing a hip? A) A 54-year-old male with obesity and short stature B) A 36-year-old female whose diet consists of excessive sugar intake C) A 77-year-old male with hearing impairment and corrective eye lenses D) An 81-year-old female taking medication for chronic osteoporosis

D. An 81-year-old female taking medication for chronic osteoporosis

Pain assessment is likely to be most challenging when providing care for which of the following older adult clients? A) A 90-year-old client who takes multiple medications for cardiac and respiratory conditions B) A 77-year-old man who has sustained burns on the lower part of his body C) An 82-year-old woman who has been diagnosed with diabetes and an anxiety disorder D) An 87-year-old man with vascular dementia and other health problems like heart failure

D. An 87-year-old man with vascular dementia and other health problems like heart failure

A long bone, such as the humerus of the upper arm, has which of the following structural characteristics? A) A perichondrium that overlies most of the bone surface B) A durable outer shell made of cancellous bone C) A diaphysis at each end D) An endosteum composed of osteogenic cells

D. An endosteum composed of osteogenic cells

A client has just been diagnosed with multiple sclerosis (MS). The nurse recognizes that the client's condition is a result of: A) Circulating antibodies attacking the postsynaptic acetylcholine levels B) Degeneration of the lower cell bodies of the lower motor neurons in the gray matter C) Degenerative changes in the musculoskeletal system D) An immune-mediated response that is caused by the demyelination fo the myelin sheath of the white matter of the brain, spinal cord, and optic nerve

D. An immune-mediated response that is caused by the demyelination of the myelin sheath of the white matter of the brain, spinal cord, and optic nerve

Which additional physical finding would you anticipate seeing in a child suspected of having a Wilms tumor? A) Massive generalized edema B) Presence of a palpable flank mass C) A visible fie red rash on the body D) Anomalies of the genitourinary system

D. Anomalies of the genitourinary system

A diabetic client has developed diabetic neuropathy and is prescribed pharmacologic intervention. The medication most likely to be prescribed would be a: A) Antihistamine B) Anti-infective C) Anti-anxiety D) Anti-seizure

D. Anti-seizure

The health care provider is evaluating a client who is at risk for the development of osteonecrosis. The client is at risk for: A) An acute infection of the bone B) Development of an unspecified mass or growth on the bone C) A chronic infection of the bone D) Aseptic necrosis of a bone segment caused by interruption of the blood supply

D. Aseptic necrosis of a bone segment caused by interruption of the blood supply

The nurse is assessing a client with a suspected diagnosis of multiple sclerosis. The nurse would anticipate the assessment data to include: A) Hyperactive reflexes B) Depression and sadness C) Heat intolerance D) Ataxia and paresthesias

D. Ataxia and paresthesias

A family brings their elderly mom to the emergency department. The client had a short period of time where she was confused, had slurred speech and appeared to have a weak arm. Now she is back to her normal self. Suspecting a transient ischemic attacks (TIAs), the health care provider will order diagnostic testing looking for: A) Aneurysm leakage B) Cause of minor residual deficits C) Diffuse cerebral electrical malfunctions D) Atherosclerotic lesions in cerebral vessels

D. Atherosclerotic lesions in cerebral vessels

A client has been diagnosed with chlamydial infection. Select the pharmacological treatment of choice. A) Acyclovir B) Ceclor C) Bactrim D) Azithromycin

D. Azithromycin

As other mechanisms prepare to respond to a pH imbalance, immediate buffering is a result of increased: A) Intracellular albumin B) Hydrogen/potassium binding C) Sodium/phosphate anion absorption D) Bicarbonate/carbonic acid regulation

D. Bicarbonate/carbonic acid regulation

A client with a long-standing diagnosis of chronic kidney disease has been experiencing increasing fatigue, lethargy, and activity intolerance in recent weeks. His care team has established that his GFR remains at a low, but stable, level. Which of the following assessments is most likely to inform a differential diagnosis? A) Blood work for white cells and differential B) Cystoscopy and ureteroscopy C) Assessment of pancreatic exocrine and endocrine function D) Blood work for hemoglobin, red blood cells, and hematocrit

D. Blood work for hemoglobin, red blood cells, and hematocrit

A client has sustained a severe, diffuse brain injury that resulted in seriously compromised brain function. The client is at greatest risk for: A) Paraplegia B) Confusion C) Amnesia D) Brian death

D. Brain death

Which of the following serves as a communicating channel for exchange of nutrients and metabolites between the osteocytes and the blood vessels on the surface of the bone layer? A) Woven bone B) Volkmann canals C) Osteons D) Canaliculi

D. Canaluculi

A client is experiencing pain, tingling, and numbness of the thumb and first, second, third, and half of the fourth digits of the hand. She states that she has pain in the wrist and hand, which worsens at night, and she has noticed that they have become clumsy. The nurse recognizes these manifestations as: A) Disuse syndrome B) Lou Gehrig disease C) Muscular dystrophy D) Carpal tunnel syndrome

D. Carpal tunnel syndrome

A client with type 1 diabetes has started a new exercise routine. Knowing there may be some increase risks associated with exercise, the health care provider should encourage the client to: A) Watch for too rapid weight loss B) Monitor for respiratory disorders C) Be careful that you're not experiencing a rebound hyperglycemia D) Carry a snack with carbs to prevent profound hypoglycemia

D. Carry a snack with carbs to prevent profound hypoglycemia

When trying to explain the difference between vasogenic versus cytotoxic cerebral edema, the physiology instructor mentions that cytotoxic edema displays which of the following functions in the brain? Select all that apply. A) Impaired blood-brain barrier that allows water/proteins to leave vessels and go into the interstitial space B) Mainly allows edema to form in the white matter of the brain C) May cause herniation by displacing a cerebral hemisphere D) Causes ischemia to build up lactic acid due to anaerobic metabolism E) Allows cells to increase volume to the point of rupture, damaging neighboring cells

D. Causes ischemia to build up lactic acid due to anaerobic metabolism E. Allows cells to increase volume to the point of rupture, damaging neighboring cells

The intracranial volume that is most capable of compensating for increasing intracranial pressure is the: A) Brain cell tissue B) Intravascular blood C) Surface sulci fluid D) Cerebrospinal fluid

D. Cerebrospinal fluid

A client with a diagnosis of epilepsy has required surgical removal of part of her prefrontal cortex. Which of the following effects should her family and care team anticipate? A) Lapses in balance and coordination B) Deficits in regulation of the endocrine system C) Sensory losses D) Changes in behavior and judgment

D. Changes in behavior and judgement

A client has suffered a stroke that has affected his speech. The physician has identified the client as having expressive aphasia. Later in the day, the family asks the nurse to explain what this means. The most accurate response would be aphasia that is: A) Characterized by an inability to comprehend the speech of others or to comprehend written material B) Nearly normal speech except for difficulty with finding singular words C) Manifested as impaired repetition and speech riddled with letter substitutions, despite good comprehension and fluency D) Characterized by an inability to easily communicate spontaneously or translate thoughts or ideas into meaningful speech or writing

D. Characterized by an inability to easily communicate spontaneously or translate thoughts or ideas into meaningful speech or writing

Impaired and delayed healing in a person with diabetes is caused by long-term complications that include: A) Ketoacidosis B) Somogyi effect C) Fluid imbalances D) Chronic neuropathies

D. Chronic neuropathies

A 16-year-old male football player suffered a fracture a fracture of his radius and ulna. Assessment of the fracture includes the following data: the fracture does not protrude through the skin, and both bones broke completely through into two pieces. Select the best description of the injury. A) Thromboemboli B) Open spiral fracture C) Closed greenstick fracture D) Closed comminuted fracture

D. Closed comminuted fracture

A client presents to the emergency department following an accident where he fell off a chair. He reports hip numbness, increasing pain, and muscle cramping. The triage nurse suspects: A) Dislocated joint B) Osteosarcoma C) Osteomyelitis D) Closed hip fracture

D. Closed hip fracture

The client asks the health care provider to explain what the purpose is for the application of cold to a sprained ankle. The best response would be: A) Dilates the blood vessels and can be left on for as long as needed B) Provides alternation vasoconstriction and dilatation when applied locally C) Causes extravasation of blood into the damaged area D) Cold provides pain relief and suppresses the release of products from tissue damage

D. Cold provides pain relief and suppresses he release of products from tissue damage

A client develops interstitial edema as a result of decreased: A) Vascular volume B) Hydrostatic pressure C) Capillary permeability D) Colloidal osmotic pressure

D. Colloidal osmotic pressure

The nurse is assessing a client who sustained a fractured radius. A cast was applied to the extremity approximately 1 hour ago, and the client is now complaining of increased pain and numbness to the finger tips. The client is most likely experiencing: A) Fracture blisters B) Thromboemboli C) Complex regional pain syndrome D) Compartment syndrome

D. Compartment syndrome

When describing the covering on bones to the students, the instructor asks, "Why is periosteum an important part of the bone covering?" The student responses should mention which of the following? It: A) Is composed of a single layer of osteoprogenitor cells B) Is the site of red blood cell development C) Supplies yellow bone marrow to assist with adipose cell production D) Contains blood vessels that assist with providing nutrition to bone tissue

D. Contains blood vessels that assist with providing nutrition to bone tissue

A client has undergone surgery for cancer of the bladder and now has an alternative reservoir called an ileal loop. The nurse recognizes that the client had a: A) Segmental surgical resection B) Diathermy C) Endoscopic resection D) Cystectomy

D. Cystectomy

Magnesium is important for the overall function of the body because of its direct role in: A) Cell membrane permeability B) Somatic cell growth control C) Sodium and tonicity regulation D) DNA replication and transcription

D. DNA replication and transcription

A client tells his health care provider that his body is changing. It used to be normal for his blood glucose to be higher during the latter part of the morning. However, now his fasting blood glucose level is elevated in the early AM (07:00). The health care provider recognizes the client may be experiencing: A) Possible stress-related hypoglycemia B) Somogyi effect C) Hyperglycemic hyperosmolar state (HHS) D) Dawn phenomenon

D. Dawn phenomenon

The nurse is assessing a client who is experiencing hyperventilation. The nurse is aware that the client is at risk for: A) Increased H2CO3 B) Decreased pH C) Increased PCO2 D) Decreased PCO2

D. Decreased PCO2

During a client's admission assessment prior to reduction mammoplasty surgery, the nurse notes a reference to a Leiden mutation in the client's history. The nurse assesses this client for an increased risk of developing which of the following postsurgical complications? A) Hemorrhage B) Myocardial infarction C) Hemophilia A or B D) Deep vein thrombosis

D. Deep vein thrombosis

A client who experienced a traumatic head injury reports extreme thirst. The nurse notes that the client is consuming 10 to 15 L of ice water daily and is experiencing polyuria. Which does the nurse suspect? A) Diabetes mellitus B) Syndrome of inappropriate antidiuretic hormone (SIADH) C) Psychogenic polydipsia D) Diabetes insipidus (DI)

D. Diabetes insipidus (DI)

A diabetic client's most recent blood work indicated a decreased glomerular filtration rate and urine testing revealed + microalbuminuria. Which priority self-care measures should the client's care team prescribe for this client? A) Use of over-the-counter herbal products for natural diuretic properties B) Increased fluid intake to at least 2000 mL/day C) Decreased oral sugar intake to less than 5 tsp/day D) Diet, exercise, and prescriptions to lower blood pressure below 140/80 mm Hg

D. Diet, exercise, and prescriptions to lower blood pressure below 140/80 mm Hg

Complex regional pain syndrome is characterized by: A) Repetitious dermatome pain attacks B) Trigeminal neuralgia with facial tics C) Severe limb pain after amputation D) Disproportionate pain with mobility

D. Disproportionate pain with mobility

Which laboratory findings should you expect to see in a patient diagnosed with nephritic syndrome? A) Low triglycerides and elevated sodium levels B) Decreased high-density lipoproteins and increased iron levels C) Abnormal blood clotting and elevated blood pressure D) Elevated urine protein and hypoalbuminemia

D. Elevated urine protein and hypoalbuminemia

A young child has been diagnosed with Wilms tumor after his mother discovered an unusual mass, prompting a diagnostic workup. Which of the following characteristics is typical of Wilms tumor? The tumor is usually: A) Asymptomatic B) Self-limiting C) A secondary neoplasm D) Encapsulated

D. Encapsulated

Which of the following trends in the hematologic status of a 6-week-old infant (born at 32 weeks' gestation) most clearly warrants medical intervention? A) Decreasing red blood cell counts B) Increasing HgA levels C) Decreasing mean corpuscular volume (MCV) D) Extremely low hematocrit

D. Extremely low hematocrit

The nurse notes that the client is experiencing visible squirming and twitching movements that can be seen as flickers under the skin. The nurse would recognize this as: A) Myoclonus B) Tics C) Tremors D) Fasciculations

D. Fasciculations

A client with a neurogenic bladder has a lesion at the level of sacral reflexes/peripheral nerves that innervate the bladder. The nurse anticipates the client will experience: A) Detrusor sphincter dyssynergia B) Spastic bladder dysfunction C) Bladder paralysis dysfunction D) Flaccid bladder dysfunction

D. Flaccid bladder dysfunction

A client with a history of a seizure disorder has been observed suddenly and repetitively patting his knee. After stopping this repetitive action, the client appears confused but is oriented to person and place but not time. What type of seizure did this client most likely experience? A) Tonic-clonic seizure B) Atonic seizure C) Myoclonic seizure D) Focal seizure with impairment to consciousness

D. Focal seizure with impairment to consciousness

Knowing that she is a carrier for Duchene muscular dystrophy (DMD), a pregnant woman arranged for prenatal genetic testing, during which her child was diagnosed with DMD. As her son develops, the woman should watch for which of the following early signs that the disease is progressing? A) Impaired sensory perception and frequent wounds B) Spasticity and hypertonic reflexes C) Muscle atrophy with decreased coordination D) Frequent falls and increased muscle size

D. Frequent falls and increased muscle size

The best lab test to monitor how well a diabetic's blood sugar levels have been managed over time is: A) Testing for glucose in the urine B) Fasting blood glucose level C) Spot checking blood glucose levels D) Glycosylated hemoglobin (HgbA1C)

D. Glycosylated hemoglobin (HgbA1C)

A client tells the health care provider that he has been very compliant over the last 2 months in the management of his diabetes. The best diagnostic indicator that would support the clients response would be: A) Fasting blood glucose level B) Urine test C) Capillary blood glucose sample D) Glycosylated hemoglobin, hemoglobin A1C (HbA1C)

D. Glycosylated hemoglobin, hemoglobin A1C (HbA1c)

The nurse is aware that the major role of the kidneys in regulating acid-base balance is to increase the production of: A) H+ B) H2CO3 C) HCl D) HCO3-

D. HCO3-

A client tells the nurse she is very concerned because the health care provider informed her that her fractured arm is healing with malunion. The nurse recognizes this as: A) An expected outcome as the client is obese B) Failure of a fracture to unite within the normal period C) Failure to produce union and cessation of the processes of bone D) Healing with deformity, angulation, or rotation that is visible on x-ray films

D. Healing with deformity, angulation, or rotation that is visible on x-ray films

The client is an average-sized adult and has abnormal microcytic hypochromic red blood cells due to a long-term, chronic disease. Which of the following complete blood count (CBC) results is characteristic of her type of anemia? A) Hematocrit 44% B) Reticulocytes 1.5% C) Band cells 3000/mL D) Hemoglobin 8 g/dL

D. Hemoglobin 8 g/dL

A late indicator of increased intracranial pressure is: A) Tachycardia B) Right-sided heart failure C) Narrow pulse pressure D) High mean arterial pressure

D. High mean arterial pressure

A 77-year-old woman has been brought to the emergency department by her daughter because of a sudden and unprecedented onset of confusion. The client admits to ingesting large amounts of baking soda this morning to treat some "indigestion." How will the woman's body attempt to resolve this disruption in acid-base balance? A) Increase the depth of inspiration B) Increasing renal H + excretion C) Increased renal HCO 3 - reabsorption D) Hypoventilation

D. Hypoventilation

A client diagnosed with CKD has begun to experience periods of epistaxis and developed bruising of skin and subcutaneous tissues. The nurse recognizes these manifestations as: A) Increased erythropoietin B) Increased platelet production C) Decreased erythropoietin D) Impaired platelet function

D. Impaired platelet function

Impaired skin integrity and skin manifestations are common in persons with chronic kidney disease. Pale skin and subcutaneous bruising are often present as a result of: A) Thrombocytopenia B) Anticoagulant therapy C) Decreased vascular volume D) Impaired platelet function

D. Impaired platelet function

The nurse is assessing a client diagnosed with myasthenia graves (MG). The nurse would expect the assessment to include: A) Impaired ability to move tongue laterally B) Absent deep tendon reflexes C) Loss of hearing on the affected side D) Inability to move eyes in multiple directions

D. Inability to move eyes in multiple directions

Following a head injury on the football field, the medical team is assessing the player for injury. One of the earliest signs of decreased level of consciousness to assess for would be: A) Stupor B) Lethargy C) Delirium D) Inattention

D. Inattention

A client has been receiving intravenous normal saline at a rate of 125 mL/hour since her surgery 2 days earlier. As a result, she has developed an increase in vascular volume and edema. Which of the following phenomena accounts for this client's edema? A) Obstruction of lymph flow B) Increased capillary permeability C) Decreased capillary colloidal osmotic pressure D) Increased capillary filtration pressure

D. Increased capillary filtration pressure

The nurse is assessing lab values of a client with disseminated intravascular coagulation (DIC). The nurse would anticipate the results to include: A) Decreased fibrin degradation product B) Decreased partial thromboplastin time (PTT) C) Increased fibrinogen D) Increased prothrombin time (PT)

D. Increased prothrombin time (PT)

A nurse is teaching a client newly diagnosed with diabetes about the action of insulin. The most appropriate information for the nurse to provide would be that it: A) Promotes breakdown of stored triglycerides B) Decreases the storage of glucose C) Promotes glucogenesis D) Increases protein synthesis

D. Increases protein synthesis

Which of the following pathophysiologic processes occurs in cases of bacterial meningitis? A) Infection in the cerebrospinal fluid causes vasoconstriction and cerebral hypoxia. B) Trauma introduces skin-borne pathogens to the cerebrospinal fluid. C) Infection in the cerebrospinal fluid causes spinal cord compression and neurologic deficits. D) Inflammation allows pathogens to cross into the cerebrospinal fluid.

D. Inflammation allows pathogens to cross into the cerebrospinal fluid

The immune suppressive and anti-inflammatory effects of cortisol cause: A) Moderate insulin resistance B) Increased capillary permeability C) Increased cell-mediated immunity D) Inhibition of prostaglandin synthesis

D. Inhibition of prostaglandin synthesis

Following a lecture on hemostasis, a nursing student accidentally cuts her hand while preparing supper for her family. She watches the laceration very closely. Sure enough, the first thing she notes is: A) Small hair-like strands form a blood (fibrin) clot. B) Limited bleeding initially as a response to the small vessel walls being sealed by a platelet plug. C) Clot retraction by pushing serum out of the clot and joining the edges of the broken vessel. D) Initially, it takes a few seconds for blood to appear as a result of vessel spasm.

D. Initially, it takes a few seconds for blood to appear as a result of vessel spasm

Select the most appropriate intervention for the nurse to teach a client diagnosed with distal symmetric neuropathy related to diabetes. A) Rotate insulin injection sites once a week B) Decrease daily walking activity C) Wear comfortable, open-toe shoes D) Inspect the feet for blisters daily

D. Inspect the feet for blisters daily

A female client with a history of chronic renal failure has a total serum calcium level of 7.9 mg/dL. While performing an assessment, the nurse should focus on which of the following clinical manifestations associated with this calcium level? A) Complaints of shortness of breath on exertion with decreased oxygen saturation levels B) Difficulty arousing the client and noticing she is disoriented to time and place C) Heart rate of 120 beats/minute associated with diaphoresis (sweaty) D) Intermittent muscle spasms and complaints of numbness around her mouth

D. Intermittent muscle spasms and complaints of numbness around her mouth

A daughter is concerned because her elderly parent has been diagnosed with osteomalacia. The daughter asks the nurse why this happened. The best response would be that: A) Absorption of too much vitamin D B) Consumption of a diet high in calcium C) Not using any sunscreens to help with absorption D) Intestinal absorption slows as a natural aging process

D. Intestinal absorption slows as a natural aging process

Which of the following pregnant women likely faces the greatest risk of developing gestational diabetes? A client who: A) Was diagnosed with placenta previa early in her pregnancy B) Is gravida five (in her fifth pregnancy) C) Has BP of 130/85 mm Hg and pulse rate of 90 beats/minute D) Is morbidly obese defined as greater than 100 pounds over ideal weight

D. Is morbidly obese defined as greater than 100 pounds over ideal weight

The nurse is providing dietary instruction for a client with chronic kidney disease who is on hemodialysis. Which would the nurse encourage the client to restrict? A) Whole grains B) Fresh apples C) Raw carrots D) Lean meats

D. Lean meats

A 6-year-old child has developed idiopathic osteonecrotic disease of the proximal (capital) femoral epiphysis. The health care provider would recognize this as: A) Osgood-Schlatter disease B) Sever disease C) Blount disease D) Legg-Calve-Perthes disease

D. Legg-Calve-Perthes disease

The nurse has just completed teaching a client newly diagnosed with type 1 diabetes about rapid-acting insulin. The nurse determines that teaching was effective when the client selects: A) Regular B) NPH C) Glargine D) Lispro

D. Lispro

The syndrome of inappropriate antidiuretic hormone (SIADH) is characterized by: A) Increased osmolality level of 360 mOsm/kg B) Excessive thirst with fluid intake of 7000 mL/day C) Copious dilute urination with output of 5000 mL/day D) Low serum sodium level of 122 mEq/L

D. Low serum sodium level of 122 mEq/L

The emergency room doctor suspects a client may have bacterial meningitis. The most important diagnostic test to perform would be: A) Blood cultures B) CT of the head C) Sputum culture D) Lumbar puncture

D. Lumbar puncture

A 40-year-old client is admitted to the hospital after experiencing 3 days of extreme vomiting. The doctor reviews the lab results and notes that the hemoglobin and hematocrit are elevated. This is interpreted as: A) Increased production of red cells by the body B) Extreme blood loss C) Normal consequence of aging D) Manifestation of dehydration

D. Manifestation of dehydration

After reviewing the 24-hour intake and output of a hospital client, the nurse suspects that the client may be experiencing flaccid bladder dysfunction. Which of the following diagnostic methods is most likely to confirm or rule out whether the client is retaining urine? A) Blood test for creatinine, blood urea nitrogen, and glomerular filtration rate B) Urine test for culture and sensitivity C) Routine urinalysis D) Measurement of postvoid residual (PVR) by ultrasound

D. Measurement of postvoid residual (PVR) by ultrasound

The nurse determines further teaching is necessary when a client diagnosed with osteoarthritis states: A) The most frequently affected joints are the hips, knees, and lumbar and cervical vertebrae B) Age is one of the strongest risk factors for osteoarthritis of all joints C) The pathogenesis of osteoarthritis resides in the homeostatic mechanisms that maintain the articular cartilage D) Men are more likely to have osteoarthritis than women

D. Men are more likely to have osteoarthritis than women

The nurse is reviewing the following lab results of a client diagnosed with renal failure: pH: 7.24 PCO2: 38 mm Hg HCO3: 18 mEq/L The nurse would interpret this as: A) Respiratory alkalosis B) Metabolic alkalosis C) Respiratory acidosis D) Metabolic acidosis

D. Metabolic acidosis

A sign of early diabetic nephropathy is: A) Anuria B) Glycosuria C) Hypertension D) Microalbuminuria

D. Microalbuminuria

A nurse is explaining the clinical manifestations of diabetic nephropathy (diabetic glomerulosclerosis) to a patient. Which would be the most important information for the nurse to provide? A) A decrease in GFR will occur with early alterations B) It is not necessary to stop smoking C) Blood glucose control has no impact on GFR D) Microalbuminuria is a predictor of future nephropathies

D. Microalbuminuria is a predictor of future nephropathies

Since catecholamines can be degraded by enzymes, the medication category usually prescribed to treat a Parkinson disease client, thereby controlling this interaction, is: A) Dopamine B) β-Adrenergic blocker C) Epinephrine D) Monoamine oxidase (MAO) inhibitor

D. Monoamine oxidase (MAO) inhibitor

A 45-year-old client with chronic kidney disease (CKD) voices concern about her dialysis treatment. The client would like to work and spend time with her family. Which type of dialysis will best fit this client's lifestyle? A) Continuous ambulatory peritoneal dialysis B) Continuous cyclic peritoneal dialysis C) Hemodialysis D) Nocturnal intermittent peritoneal dialysis (NIPD)

D. Nocturnal intermittent peritoneal dialysis (NIPD)

The condition that contributes to the pathology of all metabolic bone diseases is: A) Impaired vitamin D synthesis B) Osteosarcoma C) Infection D) Osteopenia

D. Osteopenia

A 77-year-old woman has been admitted to the geriatric medical unit of the hospital for the treatment of pneumonia. The nurse providing care for the client notes the presence of nasal calcitonin, vitamin D, and calcium chloride on the client's medication administration record. The nurse should conclude that this client likely has a history of: A) Scleroderma B) Osteoarthritis C) Rheumatoid arthritis D) Osteoporosis

D. Osteoporosis

A client presents to a health care clinic reporting sudden onset of deep, localized pain and swelling in her proximal femur. The client is currently undergoing diagnostic workup for suspected lung cancer. The nurse suspects the client may have developed which of the following neoplasms of the skeletal system? A) Exostosis B) Osteochondroma C) Endochondroma D) Osteosarcoma

D. Osteosarcoma

A client has developed hydrocephalus and asks the nurse what may have caused this to occur. The best response would be: A) Deficient cerebrospinal fluid B) Increased reabsorption of cerebrospinal fluid C) Excessive CSF flow in the ventricular system D) Overproduction of cerebrospinal fluid

D. Overproduction of cerebrospinal fluid

Opioid receptors are highly concentrated in which region of the CNS and produce pain relief through the release of endogenous opioids? A) The enkephalins B) Primary afferent neurons C) Thalamus D) Periaqueductal gray (PAG) region

D. Periaqueductal gray (PAG) region

The physician has ordered an anion gap as a laboratory test for a client experiencing metabolic acidosis. The nurse recognizes that the test will measure: A) Chloride and bicarbonate B) Base excess or deficit C) Ratio of hydrochloric acid and bicarbonate D) Phosphates, sulfates, and proteins

D. Phosphates, sulfates, and proteins

A client tells the nurse that the doctor told her she has too many red blood cells accompanied by elevated white cells and platelet counts. The nurse recognizes this as: A) Hemolytic anemia B) Aplastic anemia C) Pernicious anemia D) Polycythemia vera

D. Polycythemia vera

The nurse is reviewing lab results of a client diagnosed with metabolic acidosis. The most important electrolyte for the nurse to assess would be: A) Sodium (Na+) B) Calcium (Ca2+) C) Magnesium (Mg2+) D) Potassium (K+)

D. Potassium (K+)

When an Rh-negative mother has been sensitized and is pregnant with an Rh-positive fetus, what happens to the fetus? A) Bilirubin deficiency B) Nothing, this is normal C) Plasma volume depletion D) Profound red cell hemolysis

D. Profound red cell hemolysis

In isotonic fluid volume deficit, changes in total body water are accompanied by: A) Intravascular hypotonicity B) Increased intravascular water C) Increases in intracellular sodium D) Proportionate losses of sodium

D. Proportionate losses of sodium

The nurse is reviewing the diagnosis of four male clients. Select the diagnosis that places the clients at risk for developing post renal kidney failure A) Severe hypovolemia B) Intratubular obstruction C) Acute pyelonephritis D) Prostatic hyperplasia

D. Prostatic hyperplasia

A 26-year-old woman has sought care for increasing pain at the back of her ankle and the bottom of her foot over the past 2 weeks. The client states that she is generally in good health, although she completed a course of antibiotics for a chlamydial infection 6 weeks earlier. This client's recent history suggests the possibility of: A) Systemic sclerosis B) Ankylosing spondylitis C) Osteoarthritis D) Reactive arthritis

D. Reactive arthritis

A client was diagnosed with Salmonella 1 week ago and is now experiencing joints that are warm, swollen, and tender. The client also reports experiencing fever and weight loss. The health care provider would interpret these manifestations as: A) Rheumatoid arthritis B) Ankylosing spondylitis C) Oseoarthritis D) Reactive arthritis

D. Reactive arthritis

The nurse is caring for a client diagnosed with sickle cell disease. Select the most important factor for the nurse to be aware of that may cause the cells to sickle. A) Rapid administration of intravenous fluids B) Acute chest syndrome C) Presence of pain in the client's joints D) Reduced oxygen tension while the client sleeps

D. Reduced oxygen tension while the client sleeps

A client has been diagnosed with aplastic anemia. The nurse is aware that the client's lab results will identify: A) Increased hemoglobin counts B) Decreased inflammatory cytokines C) Relative polycythemia D) Reduction of white blood cells

D. Reduction of white blood cells

A client who has been diagnosed with temporomandibular joint (TMJ) pain is concerned about the focus of initial therapy. The nurse describes the initial focus as: A) Nutritional intervention B) Surgical intervention C) Physical therapy D) Relief of pain

D. Relief of pain

A client has started having uncontrolled seizures that are not responding to usual medications. Nursing working with the client must pay special attention to which of the following priority aspects of this clients care? Assessment of: A) ECG for arrhythmias B) Urine output and continence C) Ability to grasp hands and squeeze on command D) Respiratory status and oxygen saturation

D. Respiratory status and oxygen saturation

An infant has been diagnosed with autosomal recessive polycystic kidney disease (ARPKD). Which of the following treatment goals would be considered the priority in the care of this child? A) Rehydration therapy B) Total parenteral nutrition C) Prophylactic antibiotics D) Respiratory support

D. Respiratory support

While teaching a class of nursing students about spinal cord injury, the instructor mentions that male SCI clients will be able to have a sexual response if their injury is at which level on the spinal column? A) T12 B) S1 C) L2 D) S4

D. S4

A recently injured (3 months ago) client with a spinal cord injury at T4 to T5 is experiencing a complication. He looks extremely ill. The nurse recognizes this as autonomic dysreflexia (autonomic hyperreflexia). His BP is 210/108; skin very pale; gooseflesh noted on arms. The priority nursing intervention would be to: A) Check the mouth/throat for pustules and redness B) Check the jugular vein for distention C) Assess calves of legs for redness, warmth, or edema D) Scan his bladder to make sure it is empty

D. Scan his bladder to make sure it is empty

The nurse is caring for a client with chronic lung disease who has a history of smoking 2 packs of cigarettes per day for 25 years. The client is at high risk for the development of: A) Chronic anemia B) Thalassemia C) Acquired hemolytic anemia D) Secondary polycythemia

D. Secondary polycythemia

The nurse is caring for a client diagnosed with diabetes mellitus who is reporting burning pain of his feet. The nurse would interpret this as: A) Intermittent claudication B) Autonomic neuropathy C) Peripheral vascular disease D) Somatic neuropathy

D. Somatic neuropathy

A 33-year-old female client who presents to the doctor's office with pronounced enlargement of her hands and facial features is diagnosed with acromegaly. The acromegaly is most likely the result of: A) Decreased growth hormone B) Renal carcinomas C) Increased growth hormone D) Somatrope adenoma

D. Somatrope adenoma

An adolescent boy asks the nurse what increases the secretion of growth hormone (GH). The best response would be: A) Obesity B) Elevated glucose levels C) increased levels of cortisol D) Starvation

D. Starvation

A client who has been taking 80 mg of prednisone, a glucocorticoid, each day has been warned by his primary care provider to carefully follow a plan for the gradual reduction of the dose rather than stopping the drug suddenly. What is the rationale for this directive? A) Sudden changes in glucocorticoid dosing may reverse the therapeutic effects of the drug. B) Stopping the drug suddenly may "shock" the HPA axis into overactivity. C) Sudden cessation of a glucocorticoid can result in adrenal gland necrosis. D) Stopping the drug suddenly may cause acute adrenal insufficiency.

D. Stopping the drug suddenly may cause acute adrenal insufficiency

A client is experiencing acute pain. The nurse would anticipate the client to manifest: A) Bradypnea B) Hyperthermia C) Hypotension D) Tachycardia

D. Tachycardia

When discussing the interior quality of bones, the science instructor mentions that spongy bone has which of the following unique qualities? Select all that apply. A) Rigidity B) Thickness C) A growth plate D) Tensile strength E) Weight-bearing properties

D. Tensile strength E. Weight-bearing properties

A nurse is caring for a client who has just experienced a spinal cord injury. Which of the following bladder dysfunctions will the client experience? A) External sphincter relaxes B) Internal sphincter contracts C) The bladder becomes hyperactive D) The bladder becomes atonic

D. The bladder becomes atonic

The nurse is teaching a client with diabetes and the family about the signs and symptoms of hypoglycemia. The client asks what produces signs and symptoms of headache, disturbed behavior, coma, and seizures. The best response would be: A) Hepatic glycogenolysis B) Hypoglycemia causes ketone breakdown C) Insulin produces a rebound hyperglycemia D) The brain relies on blood glucose as its main energy source

D. The brain relies on blood glucose as its main energy source

The nurse is teaching a client with diabetes and the family about the signs and symptoms of hypoglycemia. The client asks what produces signs and symptoms of headache, disturbed behavior, coma, and seizures. The best response would be: A) Hypoglycemia causes ketone breakdown B) Insulin produces a rebound hyperglycemia C) Hepatic glycogenolysis D) The brain relies on blood glucose as its main energy source

D. The brain relies on blood glucose as its main energy source

A client is managing his diabetes with exercise and diet. The health care provider reviews the client's most recent lab results: fasting blood sugar level at 80 mg/dL and a hemoglobin A1C of 5%. Select the response that best identifies the client. A) The client is at risk for an insulin reaction B) The client needs to modify his diet related to the low readings C) The client is at risk of developing hyperglycemia D) The client is achieving normal glycemic control

D. The client is achieving normal glycemic control

Select the statement that best describes systemic lupus erythematous (SLE). SLE is characterized by which of the following? A) Exposure to UV light will aid in decreasing symptoms B) An onset that is always sudden and presence of symptoms are constant and consistent. C) T-cell-mediated response to an immunologic trigger D) The formation of autoantibodies and immune complexes (type III hypersensitivity)

D. The formation of autoantibodies and immune complexes (type III hypersensitivity)

A nurse is discussing how vascular dementia differs from Alzheimer disease with a client. Select the most appropriate information. A) Alzheimer disease most often presents with a rapid onset of memory loss followed by slowly progressive dementia that has a course of several years B) Vascular dementia develops between the ages of 35 and 45 C) Vascular dementia is caused by neuritic (senile) plaques D) The main presenting symptom of vascular dementia is slow psychomotor function

D. The main presenting symptom of vascular dementia is slow psychomotor function

A client has an impaired platelet function that may have developed from inheritance, drugs, disease, or extracorporeal circulation. The health care provider would document this as: A) Disseminated intravascular coagulation B) Fibrinolysis C) Plasmapheresis D) Thrombocytopathia

D. Thrombocytopathia

A client was started on a protocol for the prevention of deep vein thrombosis shortly after admission and has been receiving 5000 units of heparin twice daily for the last week. An immune response to this treatment may increase the client's chance of developing which health problem? A) Antiphospholipid syndrome B) Disseminated intravascular coagulation (DIC) C) Von Willebrand disease D) Thrombocytopenia

D. Thrombocytopenia

A 48-year-old male client, who normally enjoys good health, has been admitted to the hospital for the treatment of polycythemia vera. The nurse who is providing care for the client should prioritize assessments aimed at the early identification of which of the following health problems? A) Orthostatic hypotension B) Hyperventilation C) Vasculitis D) Thromboembolism

D. Thromboembolism

A client who is being seen in the outpatient clinic reports a single episode of unilateral arm and leg weakness and blurred vision that lasted approximately 45 minutes. The client is most likely experiencing: A) Lacunar infarct B) Thrombotic stroke C) Cardiogenic embolic stroke D) Transient ischemic attack

D. Transient ischemic attack

Select the condition that contributes to the risk of infertility in males. A) Genital warts B) Chancroid C) Herpes simplex D) Trichomoniasis

D. Trichomoniasis

Many factors contribute to the incontinence that is common among the elderly. A major factor is increased: A) Detrusor muscle function B) Intake of liquids and water C) Urethral closing pressure D) Use of multiple medications

D. Use of multiple medications

Which of the following characteristics differentiates a migraine with aura from a migraine without aura? A) Gastrointestinal involvement in the hours leading up to the headache B) A decrease in mood and affect prior to the headache C) Lack of response to nonpharmacologic treatments D) Visual symptoms that precede the headache

D. Visual symptoms that precede the headache

Although the distal portion of the urethra often contains pathogens, the urine formed in the kidney and found in the bladder is sterile because of the: A) Alkaline urine B) Glomerular filtering C) Warm temperature D) Washout phenomenon

D. Washout phenomenon

A client has a history of chronic alcoholism. The health care provider is aware that the client is at greatest risk for: A) Vascular dementia B) Creutzfeldt-Jakob disease C) Frontotemporal dementia D) Wernicke-Korsakoff syndrome

D. Wernicke-Korsakoff syndrome

Select the option that best describes the sensory process involved in the inhibition of micturition. A) Neural influences from the subconical centers in the basal ganglia modulate the contractile response during filling and then modulate the expulsive activity of the bladder to facility complete emptying B) As bladder filling occurs, ascending spinal afferent neuron relays this information to the micturition center to produce coordinated inhibition or relaxation of the external sphincter C) The bladder neck is widened and shortened as it is pulled up by the globular muscles in the bladder fundus; the resistance of the internal sphincter in the bladder neck is decreased, and the external sphincter relaxes D) When the bladder is distended to 150 to 250 mL in the adult, the sensation fo fullness is transmitted to the spinal cord and then to the cerebral cortex, allowing for conscious inhibition of the micturition reflex

D. When the bladder is distended to 150 to 250 mL in the adult, the sensation fo fullness is transmitted to the spinal cord and then to the cerebral cortex, allowing for conscious inhibition of the micturition reflex

Which of the following individuals is experiencing the effects of a primary endocrine disorder? A client: A) With adrenal cortical insufficiency due to pituitary hyposecretion of ACTH B) Who has hypothyroidism as a result of low TSH production C) Whose dysfunctional hypothalamus has resulted in endocrine imbalances D) Who has low calcium levels because of the loss of his parathyroid gland

D. Who has low calcium levels because of the loss of his parathyroid gland

Which of the following processes is most likely to occur as a result of a spinal reflex? A) Peristalsis of the small and large bowel B) Control of oculomotor function in changing light levels C) Pain sensation from a potentially damaging knee movement D) Withdrawal of a hand from a hot stove element

D. Withdrawal of hand from a hot stove element

While taking a client history, which of the following findings may lead the nurse to suspect the client is at risk for development of osteonecrosis? A) Previous stress fracture in the hips B) Bacterial infection in the knee following total knee replacement C) Synovial inflammation with painful swelling and warm to touch D) Bone marrow ischemia due to radiation therapy for cancer

D. bone marrow ischemia due to radiation therapy for cancer

The nurse is reviewing a client's arterial blood gas (ABG) results. The nurse interprets the client's pH level as normal when the results identify a: A) pH of 7.00 to 7.25 B) pH 6.55 to 7.12 C) pH of 7.55 to 7.8 D) pH of 7.35 to 7.45

D. pH 7.35 to 7.45

A client's clavicular fracture has healed in the weeks following a bicycle accident. Which of the following events takes place in the remodeling stage of bone healing? A) Formation of granulation tissue B) Development of fibrocartilage that resembles the appearance of the original bone C) Deposition of mineral salts into the callus D) Reduction in the size of the callus

D. reduction in the seize of the callus

An infant is born with a clubfoot. The nurse should anticipate which of the following treatment measures may be instituted? A) Surgery prior to leaving the hospital B) Administration of muscle-relaxing medications C) Skeletal traction with pins inserted into affected bones D) Serial manipulations and casting

D. serial manipulations and casting


Related study sets

Chronic Past Questions (Final Study)

View Set

AP Psychology AP classroom questions

View Set

BUAD 201 Chapter 2 Practice Quiz

View Set

Chapter 8 Alcohol and Tobacco Use and Abuse

View Set

Civil Liberties and Civil Rights Study Guide

View Set